Download as docx, pdf, or txt
Download as docx, pdf, or txt
You are on page 1of 123

1.

A 50-year-old man presents to the office with fatigue, malaise, and disabling joint pain in
his fingers, wrists, shoulder, hips, knees, and ankles. His pain is severe and associated
with a mild degree of morning stiffness for 10-15 minutes. He occasionally takes
acetaminophen and ibuprofen for this pain. He has a 10-pack-year smoking history. He
does not drink alcohol. Family history includes an uncle who died of liver cancer. On
examination, there is grayish skin pigmentation, most prominent on the exposed parts.
Abdominal examination is significant for liver enlargement 2 cm below the costal margin.
Laboratory studies reveal the following: Hemoglobin 13.0 g/L, Leukocyte count
5,500/mm3, Serum creatinine 0.8 mg/dl, Blood glucose 218mg/dl, Aspartate
aminotransferase (SGOT) 128 U/L, Alanine aminotransferase (SGPT) 155 U/L, Alkaline
phosphatase 120 U/L , Serum iron 450 mol/L (50-170g/dL), Transferrin saturation of iron
62% (22-47%), Serum Ferritin 3000ng/L (15-200 ng/ml, males). X-ray of the joints shows
narrowing of joint spaces and diffuses demineralization. This patient's condition makes
him more vulnerable to which of the following infections?

o . Epstein Barr virus


o . Escherichia coli
o . Streptococcus pneumoniae
o . Chlamydia psittaci
o . Listeria monocytogenes

The above patient is suffering from hereditary hemochromatosis. Patients with


hemochromatosis and cirrhosis are at increased risk of infection with Listeria monocytogenes.
Possible explanations include increased bacterial virulence in the presence of high serum iron
and impaired phagocytosis due to iron overload in reticuloendothelial cells. Iron overload is also
a risk factor for infection with Yersinia enterocolitica and septicemia from Vibrio vulnificus, both
of which are iron-loving bacteria.
(Choices B, C, D and E) Other than Listeria, none of the above-mentioned agents are more likely
to cause infection in patients with hemochromatosis.
Educational Objective:
Patients with hemochromatosis are vulnerable to Listeria monocytogenes infections. Iron
overload is also a risk factor for infection with Yersinia enterocolitica and septicemia from Vibrio
vulnificus, both of which are iron-loving bacteria.
The correct answer is: . Listeria monocytogenes

2. A 34-year-old obese Caucasian female complains of periodic visual obscurations. She has
episodes during which she "goes blind" for several seconds when standing up or
stooping forward abruptly. She also describes frequent morning headaches over the last
two months for which she has had to take ibuprofen or aspirin almost every morning.
She takes no other medications. Past medical history is insignificant aside from one
uncomplicated vaginal delivery. She denies use of alcohol, tobacco, or illicit drugs. She is
afebrile with a blood pressure of 138/88 mmHg and pulse of 93/min. Visual field testing
shows enlarged blind spots. There are no other significant findings on neurologic
examination. Which of the following is the most likely cause of this patient's symptoms?
Select one:
o . Glaucoma
o . Cataract
o . Amaurosis fugax
o . Papilledema
o . Optic neuritis

This patient's presentation is concerning for increased intracranial pressure (ICP) resulting in
papilledema. When intracranial pressure is increased, the pressure is transmitted to the optic
nerve sheath resulting in swelling of the optic nerve head, which can be visualized as
papilledema on ophthalmologic examination. Papilledema can cause visual syrnptorns such as
rnornentary loss of vision that varies according to changes in head positioning. Norrnal
individuals have a blind spot in their visual fields at the location of the optic nerve head, but this
blind spot enlarges in patients with papilledema. Although headaches can be due to a variety of
etiologies, those secondary to increased intracranial pressure are often worse in the rnorning,
which is the case in this patient. There is no definite cause of this patient's elevated ICP based on
the vignette, but given her fairly young age, obesity, and lack of additional cornorbidities, one
likely cause would be idiopathic intracranial hypertension (also known as pseudoturnor cerebri).
However, a CT or MRI of the brain would still be indicated to exclude an underlying rnass lesion.
(Choice A) Optic neuritis is cornrnonly associated with rnultiple sclerosis, but can be seen with
other disorders as well. It typically presents as unilateral eye pain and visual loss with an
associated afferent pupillary defect.
(Choice B) Glaucorna is caused by increased intraocular pressure. Patients rnay have peripheral
visual field deficits, but they should not have intermittent loss of vision with changes in head
position.
(Choice C) Cataracts typically present with gradual loss of vision as opposed to the intermittent
syrnptorns experienced by this patient.
(Choice E) Amaurosis fugax is transient monocular blindness that lasts only a few rninutes and is
usually vascular in origin. It would be unlikely in this patient given her young age and low risk of
atherosclerosis.
(Choice F) Uveitis can present with eye pain and redness if it is in the anterior charnber, or
painless visual changes secondary to floaters or reduced visual acuity if the posterior charnber is
affected.
Educational objective:
Papilledema usually presents with transient loss of vision lasting a few seconds with changes in
head position and can be confirmed with ophthalmologic examination. Papilledema is caused by
increased intracranial pressure, which has its own associated syrnptorns such as rnorning
headaches or changes in headache intensity with position.
The correct answer is: . Papilledema

3. A 29-year-old man returns home to Colorado after a diving trip to Honduras in Central
America. He was gone for 6 days. The day he returns, he starts to develop diarrhea,
abdominal cramping, and nausea. There is no mucus and blood in the stool. He has no
other medical problems and does not take medications. He does not use tobacco,
alcohol or drugs. His temperature is 37.2°C (98.9°F), blood pressure is 120/74 mm Hg,
pulse is 80/min, and respirations are 15/min. There is no lymphadenopathy. Chest is clear
to auscultation. Abdomen is soft and non-tender. There is no organomegaly. Bowel
sounds are increased. Stool is negative for leukocytes and fecal occult blood. Which of
the following is the most likely pathogen responsible for his symptoms?
Select one:
o . Escherichia coli
o . Giardia
o . Vibrio cholerae
o . Salmonella
o . Cyclospora

Diarrhea in travelers is most commonly due to contaminated food and water. Although a variety
of agents (e.g., bacteria, viruses, parasites) are possible, enterotoxigenic E coli is the most
frequent cause of traveler's diarrhea.
(Choices A and B) Parasites may cause diarrhea in travelers, but do so rarely. Some regions
arehyperendemic for certain parasites. For instance, Nepal is hyperendemic for Giardia and
Cyclospora. Certain mountainous areas of the northern and western USA are also hyperendemic
for Giardia. Thus traveler's diarrhea is more likely to be parasitic in these locations.
(Choices D and E) Salmonella and Vibrio cholerae are less common causes of traveler's diarrhea.
Educational objective:
Enterotoxigenic E coli is the most frequent cause of traveler's diarrhea. Travelers with abdominal
cramps, diarrhea, and malaise are likely to be infected with this organism.
The correct answer is: . Escherichia coli

4. A 74-year-old man presents with fatigue, short- ness of breath on exertion, and back and
rib pain, which is made worse with movement. Investigations reveal he is anemic,
calcium, urea, and creatinine are elevated. X-rays reveal multiple lytic lesions in the long
bones and ribs, and protein electrophoresis is positive for an immunoglobulin G (IgG)
paraprotein. Which of the following is the most likely mechanism for the renal injury?
Select one:
o uric acid crystals
o tubular damage by light chains
o glomerular injury
o vascular injury by light chains
o plasma cell infiltrates
In multiple myeloma, tubular damage by light chains is almost always present. The injury is a
direct toxic effect of the light chains or indirectly from the inflammatory response. Infiltration by
plasma cells and glomerular injury is rare. Hypercalcemia may produce transient or irreversible
renal damage as do amyloid and myeloma cell infiltrates. (Kasper, p. 658)
The correct answer is: tubular damage by light chains

5. A 65-year-old Caucasian male presents to your office complaining of an episode of


slurred speech and clumsiness of his right hand. The episode lasted 15 minutes and
resolved spontaneously. He had a similar episode one week ago. His past medical history
is significant for moderate hypertension, diabetes mellitus (OM) type 2 and osteoarthritis
of the right knee. He has smoked one pack of cigarettes daily for 35 years, and drinks 1-2
glasses of wine daily. His current medications include metoprolol, glyburide and
naproxen. His blood pressure is 160/95 mmHg, pulse is 65/min, respirations are 16/min,
and temperature is 36.7°C (98°F). The physical findings are within normal limits. The lab
studies show: Fasting blood glucose 200 mg/dl, Total cholesterol 240 mg/dl, LDL
cholesterol 140 mg/dl, HDL cholesterol 76 mg/dl. What is the most important risk factor
for a stroke in this patient?
Select one:
o . Smoking
o . Alcohol consumption
o . Diabetes mellitus
o . Elevated cholesterol level
o . Hypertension

Of all the abovementioned traditional risk factors for cardiovascular disease, hypertension has
the strongest association with strokes. Hypertension increases the risk of all types of stroke.
Furthermore, multiple observational studies have demonstrated that patients with hypertension
have approximately four times the risk of stroke when compared to non-hypertensive subjects.
(Choices B and E) Smoking and diabetes mellitus are established risk factors for strokes, but
their association with strokes is not as strong as hypertension.
(Choice C) Hypercholesterolemia is a strong risk factor for coronary heart disease, but its
association with strokes is marginal at best
(Choice D) Moderate alcohol consumption has a protective effect, and explains the increased
HDL cholesterol level of this patient
Educational Objective:
Multiple observational studies have demonstrated that patients with hypertension have
approximately four times the risk of stroke when compared to non-hypertensive subjects.
*Know the concept of multiple risk factors. Know the association between hypertension and
strokes. This is a very common scenario for USMLE Steps 2 & 3
The correct answer is: . Hypertension

6. A 74-year-old nursing home resident is brought to the ER with a low-grade fever, cough
and shortness of breath for the last two days. The cough is productive of small amounts
of greenish sputum. His past medical history is significant for hypertension, diabetes
mellitus type 2, COPD, hypercholesterolemia and mild dementia. His blood pressure is
152/78 mmHg and his heart rate is 89/min, regular. Physical examination reveals
decreased breath sounds, coarse rhonchi, and increased fremitus over the lower left lung
field. His oxygen saturation is 92% on room air when lying on his right side but drops to
84% when he lies on his left. Which of the following best explains this finding?
Select one:
o . Effort-dependent hypoventilation
o . Decreased cardiac output
o . Increased arterio-venous shunting
o . Increased dead space ventilation
o . Decreased oxygen diffusion capacity

Proper oxygenation of blood requires a balance between ventilation and perfusion. Under
normal circumstances in an upright individual, both ventilation and perfusion are greatest at the
lung bases (the most dependent regions of the lungs) and least at the apices. When lying on
one's right or left side, the dependent lung gets the greatest ventilation and perfusion.
This patient has signs and symptoms of a left lower lobe (LLL) pneumonia. Decreased breath
sounds and increased fremitus over the LLL indicate consolidation, and rhonchi indicate
narrowed conducting airways (possibly due to mucous plugging). In consolidated lung
segments the alveoli are filled with exudate and do not participate in pulmonary gas exchange,
so their ventilation is essentially zero, i.e., blood perfusing consolidated areas is not oxygenated.
Positions that increase the proportion of pulmonary blood flow passing through such segments
decrease the patient's oxygen saturation (right-to-left shunt). When this patient lies on his right
side, the consolidated LLL is at a higher vertical height (less dependent) and therefore receives a
reduced proportion of the pulmonary blood flow, minimizing the shunt caused by the
consolidated LLL. Conversely, when this patient is on his left side, the consolidated LLL is more
dependent, receives an increased proportion of the pulmonary blood flow, and worsens the
shunt.
(Choice A) Dead space refers to the volume of inhaled air that is not available for gas exchange.
As a consolidated lobe cannot fill with inhaled air, it does not significantly increase the
physiologic dead space. An example of a situation that would cause hypoxia due to increased
dead space would be a pulmonary embolism, where the lung is properly ventilated, but
segments of lung are not perfused.
(Choice B) If anything, lying on the left side should increase this patient's cardiac output, as a
reflex attempt to compensate for impaired arterial oxygenation.
(Choice C) Oxygen diffusion capacity does not change with position.
(Choice E) This patient's total minute ventilation would not be expected to change with position.
Educational objective:
Placing a consolidated lung segment in the dependent position can increase the right-to-left
shunt in a patient with pneumonia.
The correct answer is: . Increased arterio-venous shunting

7. A 43-year-old HIV-positive male presents to your office with several exophytic purple
skin masses on his lower abdomen. Physical examination reveals tender hepatomegaly
and an abdominal CT scan shows nodular, contrast-enhanced intrahepatic lesions of
variable size. Liver biopsy is attempted but severe hemorrhage results. Which of the
following is the most likely cause of this patient's condition?
Select one:
o . Mycobacteria
o . Spirochetes
o . Brucella
o . Bartonella
o . Clostridia

This is the typical presentation of bacillary angiomatosis (BA), a condition caused by Barlonella
henselae and Barlonella quintana. BA generally affects immunosuppressed patients (e.g. those
with AIDS, hematologic malignancies, undergoing chemotherapy, organ transplant recipients).
BA causes nonspecific constitutional symptoms like fever, weight loss, malaise and abdominal
pain in addition to characteristic lesions of the skin and viscera. The classic cutaneous lesion in
BA - a large, pedunculated exophytic papule with a cellarette of scale - resembles a large
pyogenic granuloma or cherry angioma. Diagnosis is made via tissue biopsy and microscopic
identification of organisms and the characteristic angiomatous histology. Extreme caution must
be exercised in biopsying these lesions because they are prone to hemorrhage. BA can be
treated with a variety of antibiotics which lead to involution of the lesions.
Educational Objective:
Barlonella henselae and Barlonella quintana cause bacillary angiomatosis in
immunocompromised individuals. Patients present with cutaneous and visceral angioma-like
blood vessel growths. Antibiotic treatment causes lesion regression.
The correct answer is: . Bartonella

8. A 52-year-old alcoholic man presents to the emergency department because of anxiety


and tremors. His last drink of alcohol was 2 days ago. His initial electrolyte panel is:
Sodium 132 mEq/L, Potassium 2.9 mEq/L, Chloride 100 mEq/L, Bicarbonate 25 mEq/L. He
is treated for alcohol withdrawal, and given aggressive intravenous and oral potassium
supplementation. Three days later, his electrolyte panel is: Sodium 135 mEq/L, Potassium
3.1 mEq/L, Chloride 102 mEq/L, Bicarbonate 28 mEq/L. Which of the following explains
why this patient's potassium level is so difficult to correct?
Select one:
o . Alcohol withdrawal
o . Hypophosphatemia
o . Poor oral absorption
o . Hypomagnesemia
o . Thiamine deficiency

Chronic alcoholism eventually results in multiple electrolyte abnormalities such as


hypomagnesemia, hypokalemia and hypophosphatemia. In this vignette, therefore, it is likely
that the patient also has low magnesium levels. Hypomagnesemia is notorious for causing
refractory hypokalemia. Mg is an important cofactor for potassium uptake and maintenance of
intracellular potassium levels. For these reasons, magnesium levels in alcoholics must also be
checked and corrected to successfully correct the coexisting hypokalemia. Another cause of
hypomagnesemia is intake of diuretics.
(Choice A} Poor oral absorption may result in hypokalemia refractory to oral replacement.
Nevertheless, it is not the cause of refractory hypokalemia in this patient since he remained
hypokalemic despite aggressive IV potassium supplementation.
(Choice B) Hypophosphatemia can result in weakness, rhabdomyolysis, paresthesias, and
respiratory failure in severe cases. It does not directly cause hypokalemia.
(Choice C) Alcohol withdrawal, by itself, does not cause refractory hypokalemia.
(Choice E) Thiamine deficiency is also seen in alcoholics; however, it does not cause refractory
hypokalemia.
(Choice F) Hypoalbuminemia is commonly seen in alcoholics. It may indicate poor nutrition,
decreased hepatic synthetic function, or proteinuria. It may result in spurious hypocalcemia,
since calcium is largely albumin-bound. It does not cause hypokalemia.
(Choice G) Type I or II renal tubular acidosis (RTA} can cause hypokalemia; however, this patient
cannot have RTA because he is not acidotic (i.e., the bicarbonate level is normal}.
Educational Objective:
Chronic alcoholics typically present with multiple electrolyte abnormalities such as hypokalemia,
hypomagnesemia and hypophosphatemia. Hypomagnesemia, causes refractory hypokalemia;
therefore it is important to correct the magnesium along with the potassium levels to be able to
correct the electrolyte abnormalities of such patients.
The correct answer is: . Hypomagnesemia

9. A 56-year-old man is having intermittent fevers and malaise for the past 2 weeks. He has
no other localizing symptoms. Two months ago, he had valve replacement surgery for a
bicuspid aortic valve. A mechanical valve was inserted and his postoperative course was
uncomplicated. On examination, his temperature is 38°C, blood pressure 124/80 mm Hg,
pulse 72/min, and head and neck are normal. There is a 3/6 systolic ejection murmur, the
second heart sound is mechanical, and a 2/6 early diastolic murmur is heard. The lungs
are clear and the skin examination is normal. Three sets of blood cultures are drawn and
an urgent echocardiogram is ordered. Which of the following is the most likely causative
organism?
Select one:
o S. epidermidis
o enterococci
o Candida
o Staphylococcus aureus
o S. viridans

S. epidermidis is still the leading cause of prosthetic valve endocarditis in the early postoperative
period (usually defined <2 months). It is usually the consequence of intraoperative
contamination or postoperative bacteremia. Endocarditis occurring 12 months after surgery is
usually due to the same organisms that cause native valve endocarditis. (Kasper, p. 736)
The correct answer is: S. epidermidis

10. A 14-year-old boy is brought by his mother because she noticed a change in his voice.
He has been having frequent nosebleeds for the last month, and feels that his "left nose"
is always congested. There is no history of trauma. He admits to using marijuana, in the
absence of his mother. He is otherwise well, and does not take any medications. He
actively participates in the school basketball tournaments. Physical examination reveals
an intact nasal septum with a visible mass at the back of the left nostril. CT scan reveals
an erosion of the adjacent bone. What is the most likely reason of this patient's
nosebleeds?
Select one:
o . Chondroma of nasal cartilage
o . Angiofibroma
o . Reactive nasal polyps
o . Bleeding disorder
o . Cocaine abuse

Any adolescent who presents with a nasal obstruction, visible nasal mass, and frequent
nosebleeds (epistaxis) is considered to have a juvenile angiofibroma (JNA), unless proven
otherwise. This is typically found in the back of the nose or upper throat (nasopharynx) of
adolescent boys. It is a benign growth, but is capable of eroding and locally invading. JNAs are
deemed potentially very dangerous because these are composed of many blood vessels which
may bleed readily. In addition, its common areas of occurrence are difficult to access surgically.
For these reasons, such tumors should only be touched by a specialist. In some cases, no
treatment is necessary. Treatment is required if the angiofibroma is enlarging, obstructing the
airway, or causing chronic nosebleeds. Surgical treatment includes removal of the tumor.
Removal is often difficult because the tumor is unencapsulated and may be deeply invasive.
Recurrence of the tumor after surgical resection is common.
(Choice A) Although cocaine abuse can be a cause of nasal bleeding, it does not present as a
nasal mass and bone erosion.
(Choice C) An underlying bleeding disorder is possible but less likely, given this patient's CT
findings.
(Choice D) Reactive nasal polyps normally do not cause bony erosions. These are usually
associated with chronic infections or allergies, and the main complaint is obstruction rather than
bleeding.
(Choice E) Chondroma of the nasal cartilage is very rare, and is not seen in young adults.
Educational Objective:
Any male adolescent who presents with epistaxis, a localized mass, and a bony erosion on the
back of the nose has an angiofibroma until proven otherwise.
The correct answer is: . Angiofibroma

11. A 23-year-old woman with no other past medical history was diagnosed with
hypertension 6 months ago. She was initially treated with hydrochlorothiazide, followed
by the addition of lisinopril, followed by high doses of a beta-blocker, but her blood
pressure has not been well controlled. She assures the provider that she is taking all of
her medicines. On examination her blood pressure is 165/105 mm Hg in each arm, and
168/105 mm Hg when checked by large cuff in the lower extremities. Her pulse is 60.
Cardiac examination reveals an S4 gallop but no murmurs. She has a soft mid-abdominal
bruit. Distal pulses are intact and equal. She does not have hyperpigmentation, hirsutism,
genital abnormalities, or unusual distribution of fat. Her sodium is 140, potassium 4.0,
HCO3 22, BUN 15, and creatinine 1.5. Which of the following is the most likely cause of
her difficult-to-control hypertension?
Select one:
o . Primary hyperaldosteronism (Conn syndrome)
o . Cushing syndrome
o . Congenital adrenal hyperplasia
o . Coarctation of the aorta
o . Fibromuscular dysplasia

(Fauci, p 1570) T his patient is young to have developed hypertension, and the finding of renal
bruits is highly suggestive of a secondary cause of the condition: renal artery stenosis caused by
fibromuscular dysplasia (FMD). FMD is more common in young females (85%-90% of cases are
in females in some series). T he exact etiology of the condition is unknown, but renal artery
stenosis causing hypertension is a common presentation. Digital subtraction angiography is the
diagnostic modality of choice though duplex ultrasonography, CT angiography, and MR
angiography can also be utilized. Etiologies that can mimic fibromuscular dysplasia include
atherosclerosis and vasculitis. T he patient has no physical findings to make one suspect Cushing
syndrome (abnormal fat distribution, ecchymoses, hirsutism, etc), congenital adrenal hyperplasia
(virilization), or coarctation of the aorta (BP lower in legs than in arm). She does not have the
metabolic alkalosis and hypokalemia of primary hyperaldosteronism.
The correct answer is: . Fibromuscular dysplasia
12. A 76-year-old woman with a history of congestive heart failure, coronary artery disease,
and an “irregular heart beat” is brought to the ED by her family. She has been
complaining of increasing abdominal pain over the past several days. She denies nausea
or vomiting and bowel movements remain unchanged. Vitals are HR of 114 beats per
minute, BP 110/75 mm Hg, and temperature 98°F. On cardiac examination, her HR is
irregularly irregular with no murmur detected. The abdomen is soft, nontender, and
nondistended. The stool is heme-positive. This patient is at high risk for which of the
following conditions?
Select one:
o . Sigmoid volvulus
o . Acute cholecystitis
o . Mesenteric ischemia
o . Perforated gastric ulcer
o . Diverticulitis

(Rosen, pp 1188-1192.) Patients with coronary artery disease, valvular heart disease, and
arrhythmias, particularly atrial fibrillation, are at high risk for mesenteric ischemia. In addition,
age greater than 50 years, congestive heart failure, recent myocardial infarction, critically ill
patients with sepsis or hypotension, use of diuretics or vasoconstrictive medications, and
hypercoagulable states place patients at higher risk. The most common cause of acute
mesenteric ischemia is arterial embolus, which accounts for 50% of cases. The classic finding is
“pain out of proportion to examination findings”; that is, a patient complains of severe pain but
is not particularly tender on examination. A high degree of suspicion for mesenteric ischemia in
an elderly patient with abdominal pain is warranted.
(a) Perforated gastric ulcer presents with acute onset of severe epigastric pain and bleeding,
generally in someone with PUD. (b) Diverticulitis presents as LLQ pain that is usually described
as dull and constant. (c) Acute cholecystitis occurs with an obstruction of the cystic duct with
gallstones and is often accompanied by fever, chills, nausea, and a positive Murphy sign. It is the
most common surgical emergency in elderly patients. (e) The classic triad for sigmoid volvulus
includes abdominal pain, abdominal distention, and constipation. Nausea and vomiting are
often present, and diagnosis can be made on plain radiograph in 80% of cases.
The correct answer is: . Mesenteric ischemia

13. A 29-year-old man with acquired immune deficiency syndrome (AIDS) comes to the
emergency department because of progressively increasing abdominal discomfort.
Examination shows voluntary guarding in the upper abdomen. His biochemistry is
normal except for an elevated amylase at 370 U/L (25–125 U/L). Which of the following
infections can trigger this disorder in AIDS patients?
Select one:
o . herpes virus
o . toxoplasmosis
o . Mycobacterium avium complex
o . Mycobacterium tuberculosis
o . Pneumocystis carinii

Pancreatitis in AIDS patients can be caused by cytomegalovirus and cryptosporidium as well as


M. avium complex. Drugs are another cause of AIDS-related pancreatitis. (Kasper, p. 1899)
The correct answer is: . Mycobacterium avium complex

14. A 60-year-old white male presents to the Emergency Room with sudden onset of
dyspnea. He is a truck driver and just returned from a long trip. His past medical history
is insignificant. He is not taking any medications. His blood pressure is 110/70 mmHg
and heart rate is 110/min. Physical examination reveals a moderately overweight man
with tachypnea. Lungs are clear on auscultation. ECG shows right axis deviation. You
order ventilation/perfusion scanning. Which of the following findings will help you the
most to confirm the diagnosis?
Select one:
o . An area of perfusion defect without ventilation defect
o . An area of ventilation defect without perfusion defect
o . Several small perfusion and ventilation defects
o . An area of ventilation and perfusion defect
o . Absence of ventilation and perfusion abnormalities

You should suspect pulmonary thromboembolism in this patient with acute onset of dyspnea
and a history of a long period of immobility (a driver who just returned from a trip). Dyspnea,
tachypnea, normal lung examination, and right axis deviation on ECG are suggestive of the
pulmonary embolism. The next step in the management of such a patient usually involves
ventilation/perfusion scanning. In patients with pulmonary thromboembolism, you expect to
reveal a relatively large area of perfusion defect without ventilation defect called mismatched
defect. Finding such a defect is diagnostic, and you should proceed with treatment; but, this
'classic' finding is present in less then 50% of patients with pulmonary thromboembolism I
Usually the findings are inconclusive, like several small perfusion defects (Choice D), or matched
ventilation/perfusion defect (Choice C). In such a case, further work-up is necessary. At the other
end, absence of ventilation and perfusion abnormalities (Choice E) helps to rule out significant
pulmonary thromboembolism as the cause of the problem. An area of ventilation defect without
perfusion defect (Choice A) is more suggestive of a respiratory problem.
Educational Objective:
Ventilation/perfusion scanning is a useful tool in diagnosing pulmonary thromboembolism.
Mismatched perfusion defect is characteristic, but it is found in less then 50% of the patients.
The correct answer is: . An area of perfusion defect without ventilation defect

15. A 14-year-old African American male is referred to your office after his older brother
experienced sudden cardiac arrest during hiking trip. He has no current complaints. He
denies any illicit drug use. His blood pressure is 110/60 mmHg and his heat rate is
75/min. Neck is supple, no jugular venous distention is appreciated. Carotid pulse seems
to have dual upstroke. Lungs are clear. There is strong apical impulse and a systolic
ejection type murmur along the left sternal border. Which of the following is most likely
to increase the murmur intensity in this patient?
Select one:
o . Squatting
o . Recumbency
o . Valsalva maneuver
o . Sustained handgrip
o . Leg raising

This patient most likely has hypertrophic cardiomyopathy. The condition is more common in
AfricanAmericans. It shows an autosomal dominant inheritance, so the sudden death of the
patient's brother is likely a clue to the diagnosis. The patient's physical exam is also suggestive.
A carotid pulse with dual upstroke occurs from midsystolic obstruction that develops as the
heart contracts. Hypertrophic cardiomyopathy typically causes a systolic ejection-type murmur
along the left sternal border, and often has a strong apical impulse. Contrary to most murmurs,
the murmur of hypertrophic cardiomyopathy increases as preload decreases since this lessens
the size of the ventricular cavity and causes increased outflow obstruction. Most murmurs will
decrease with decreased preload since there will be less flow. The Valsalva maneuver is the only
maneuver listed that will decrease venous return, and therefore preload.
(Answer A} Squatting decreases the vertical height of the blood column, and therefore will
increase venous return to the heart. This increase venous return will result in a larger size of the
ventricle and hence less outflow obstruction.
(Answer C) Sustained handgrip leads to an increase in systemic vascular resistance and increased
afterload. Increased afterload leads to a larger ventricular volume and thus a decrease in outflow
obstruction and a quieter murmur. Handgrip is typically used to differentiate between the
murmurs of aortic stenosis and mitral regurgitation where it causes a decrease and increase,
respectively, in the severity of the murmur.
(Choice D) Recumbency increases venous return. As described above, this will decrease outflow
obstruction and the murmur will therefore be less severe.
(Choice E) Leg raising will also increase venous return and will decrease the murmur of
hypertrophic cardiomyopathy.
Educational objective:
The murmur of hypertrophic cardiomyopathy is located at the left lower sternal border and is
worsened by maneuvers that decrease preload (e.g. Valsalva, standing}
The correct answer is: . Valsalva maneuver

16. A 72-year-old woman complains of fatigue, dyspepsia, and shortness of breath. Her
daughter tells you that her mother also has some slight memory loss and occasionally
complains of numbness in her legs. The laboratory tests you ordered show a hemoglobin
of 10.2 g/dL and an MCV of 110. The most likely cause is:
Select one:
o Autoantibodies to gastric parietal cells
o Autoantibodies to thyroglobulin
o Autoantibodies to ribosomal P protein
o Autoantibodies to dsDNA (double-stranded DNA)
o Autoantibodies to histones

(McPhee, 2/e, pp 290; Fauci, 14/e, pp 655, 656,1875, 2018.) The syndrome described is most
consistent with pernicious anemia, a macrocytic anemia, in which 90% of persons with this
disease have antibodies to gastric parietal cells and about 60% of persons possess anti-intrinsic
factor antibody. The antiparietal cell antibody destroys parietal cells and leads to malabsorption
of cobalamin and a macrocytic anemia. Antibodies to histones, dsDNA, and ribosomal P protein
occur in systemic lupus erythematosus (SLE). Autoantibodies to thyroglobulin are characteristic
of Hashimoto’s disease.
The correct answer is: Autoantibodies to gastric parietal cells

17. A 42-year-old woman is evaluated for depression, mood swings and poor. She also
complains of mild headaches and muscle weakness. She has had two ER visits for kidney
stones over the past year. She denies any illicit drug use. Her blood pressure is 160/105
mmHg and her heat rate is 85/min. Her laboratory findings are shown below: Sodium
140mEq/L, Potassium 3.6 mEq/L, Chloride 101 mEq/L, Bicarbonate 24 mEq/L, BUN 13
mEq/L, Creatinine 0.9 mEq/L, Glucose 98 mEq/L, Calcium 11.7 mg/dL. Which of the
following is the most likely cause of this patient's hypertension?
Select one:
o . Hypothyroidism
o . Parathyroid gland disease
o . Aortic dissection
o . Renal vascular stenosis
o . Renal parenchymal disease

This woman has hypertension, psychiatric symptoms (depression, mood swings, and poor sleep),
recurrent kidney stones, and hypercalcemia. This presentation is concerning for
hyperparathyroidism. The parathyroid gland produces parathyroid hormone (PTH) which acts to
increase serum calcium levels and decrease serum phosphate. The mnemonic "stones, bones,
groans, and psychiatric overtones" describes the common symptoms which result from
hypercalcemia. Specifically, hypercalcemia may lead to kidney stones, constipation/indigestion,
and depression/fatigue/psychosis. Patients with hyperparathyroidism also have an increased
incidence of hypertension, although it is an uncommon cause of secondary hypertension.
(Choice A) Renal parenchymal disease causes hypertension. However, parenchymal disease
severe enough to cause hypertension should also cause a decrease in glomerular filtration rate.
This patient's creatinine concentration is normal.
(Choice B) Renal artery stenosis is an important cause of secondary hypertension. Usually due to
atherosclerosis, renal artery stenosis generally presents with few systemic complaints and may
be entirely asymptomatic. It would not explain this woman's hypercalcemia, kidney stones, and
psychiatric symptoms. A bruit may often be heard upon auscultation of the affected renal artery.
(Choice C) Aortic dissections present with tearing chest pain that radiates to the back. They most
commonly occur in patients with hypertension but are not, themselves, causes of hypertension.
(Choice D) Hypothyroidism causes weight gain, fatigue, bradycardia, depression, and skin and
hair changes. This patient's hypercalcemia, kidney stones, and hypertension suggest
hyperparathyroidism, not hypothyroidism.
(Choice F) Corticosteroids, mineralocorticoids, and adrenal androgens are made in the adrenal
cortex. This patient's systemic symptoms do not suggest Cushing's syndrome, primary
hyperaldosteronism, or virili zation.
(Choice G) Increased activity within the medulla (pheochromocytoma) would cause
hyperadrenergic symptoms such as weight loss, tachycardia, hypertension, diaphoresis, and
anxiety.
Educational objective:
Hyperparathyroidism is an uncommon cause of secondary hypertension. Asymptomatic
hypercalcemia in a hypertensive patient or the presence of "stones, bones, groans and
psychiatric overtones" should raise suspicion for this condition.
The correct answer is: . Parathyroid gland disease

18. An 80-year-old Caucasian female is brought to the emergency room by her son with a
three-day history of fever and a foul-smelling, productive cough. Her past medical
history is significant for advanced dementia, diabetes, and hypertension. She takes
aspirin, metformin, insulin, and atenolol. She was admitted two times with pneumonia
during the past two months. Her temperature is 38.3°C (101°F), blood pressure is
100/70mmHg, pulse is 105/min, and respirations are 20/min. The patient is not oriented
in time and place. Physical examination reveals dry mucus membranes and decreased
skin turgor. Breath sounds are decreased to the right. A chest x-ray revealed right, lower
lobe infiltrate. Which of the following is the most important predisposing factor for this
condition in this patient?
Select one:
o . Depressed cell-mediated immunity
o . Gastro-esophageal reflux
o . Decreased thyroid function
o . Impaired epiglottic reflex
o . Decreased lung elasticity

This patient presents with community-acquired pneumonia, most likely from aspiration.
Impaired swallowing and epiglottic reflex are frequent abnormalities in patients with advanced
dementia. As a result, these patients are predisposed to aspiration of oropharyngeal secretions.
Anaerobic bacteria in combination with some aerobic oral flora (e.g., viridans streptococci) are
the usual etiologic factors of aspiration pneumonia. The infection may be necrotic and quickly
progress to an abscess.
Advanced age, per se, is a predisposing factor to pneumonia. Several factors may be involved,
including senile lung changes, pulmonary co-morbidities (Choice A), and decline in the immune
defense mechanisms (Choices E and F).
Pulmonary congestion due to left ventricular dysfunction impairs lymphatic and venous
drainage (Choices G and H) and can predispose to pneumonia, but no signs and symptoms of
heart failure are present in this patient.
Advanced gastro-esophageal reflux (Choice B) can cause aspiration of gastric content,
bronchoconstriction, and pneumonia. It is less likely than impaired epiglottic reflex as the cause
of pneumonia in this patient.
Educational Objective:
Patients with impaired consciousness, advanced dementia, and other neurologic disorders are
predisposed to aspiration pneumonia due to impaired epiglottic function.
The correct answer is: . Impaired epiglottic reflex

19. A 6-year-old Caucasian boy is hospitalized for acute sinusitis that was accompanied with
intensive nasal bleeding. Past medical history is significant for recurrent pulmonary
infections and several hospitalizations for parenteral antibiotic therapy. The sweat
chloride test is positive. The blood tests reveal a prothrombin time (PT) of 20 seconds.
Which of the following coagulation factors is most likely to be deficient in this patient?
Select one:
o . Fibrinogen
o . Factor V
o . Hageman factor
o . Factor VIII
o . Factor VII

o Severe recurrent respiratory infections and positive sweat chloride test are characteristic
for cystic fibrosis. Fat malabsorption is common in patients with cystic fibrosis and is due
to exocrine deficiency of the pancreas; therefore, deficiency of fat-soluble vitamins,
including vitamin K, is frequently present if no vitamin supplementation is provided.
Besides that, antibiotic therapy may predispose this patient to vitamin K deficiency.
Vitamin K is an important cofactor for the liver enzyme gamma-glutamyl carboxylase
which adds carboxyl groups to the glutamate residues of several coagulation factors:
factors II, VII, IX, and X as well as proteins C and S. These extra carboxyl groups increase
the affinity of these proteins for the phospholipids on the surface of the platelets.
o Fibrinogen (Choice A) and factor V (Choice E) does not require vitamin K-dependent
posttranslational modification. Hageman factor (Choice B) and factor VIII (Choice C) are
not vitamin K dependent; besides that, their deficiency does not prolong PT.
o Educational Objective:
o Vitamin K is an important cofactor in posttranslational modification of several
coagulation factors: factors II, VII, IX, and X as well as proteins C and S.
o The correct answer is: . Factor VII

20. A 24-year-old female is brought to the emergency room because of dizziness and near
syncope. She says that every time she stands up, she feels lightheaded. She has no
significant past medical problems. She does note having broken up with her boyfriend
three months ago, which has caused her to have decreased appetite and an associated
ten-pound weight loss. She has also missed her last two menstrual periods. She denies
using tobacco, alcohol or drugs. On physical examination, her temperature is 36.7°C
(98°F). When lying supine, her blood pressure is 100/70 mm Hg and her pulse is 88/min.
When she stands up, her blood pressure is 80/50 mm Hg and her pulse is 120/min. Other
than dry skin and mucous membranes, her physical examination is unremarkable. Initial
laboratory studies reveal: Serum sodium low, Serum potassium low, Urine sodium
increased, Urine potassium increased. This patient's dizziness is most likely due to which
of the following?
Select one:
o . Diuretic abuse
o . Mineralocorticoid deficiency
o . Laxative abuse
o . Low caloric intake
o . Self-induced vomiting

This young woman has orthostatic hypotension, which likely explains her dizziness. In addition,
she has abnormalities of her serum and urine electrolytes. Diuretic abuse, a technique
sometimes used by individuals desperate for weight loss, can cause hypovolemia, orthostatic
hypotension, and dizziness by increasing the elimination of sodium, potassium and water by the
kidneys, and would explain this patient's pattern of electrolyte abnormalities (Choice D). This
patient's recent break-up was likely the catalyst for this dangerous behavior. Other causes of
orthostatic hypotension include anything that decreases intravascular volume or vascular tone.
Decreased fluid intake, polyuria, and diarrhea decrease intravascular volume. Decreased vascular
tone can be caused by autonomic neuropathy or medications. Commonly implicated
medications include anti-hypertensive and psychiatric drugs.
(Choice A) Low caloric intake causes weight loss secondary to wasting of muscle mass and fat. It
could also lead to electrolyte abnormalities, but elevated urine electrolytes would not be
expected.
(Choice B) Self-induced vomiting may lead to low serum sodium and potassium, and the
associated loss of fluid can cause orthostatic hypotension. However, urinary sodium will be low
as the kidneys try to conserve water by maximally resorbing sodium and water.
(Choice C) Laxative abuse leads to loss of significant amounts of water from the colon, causing
dehydration, orthostatic hypotension, and weight loss. It may also cause electrolyte imbalances.
However, this patient is losing electrolytes via her genitourinary system.
(Choice E) Mineralocorticoid deficiency can lead to wasting of sodium and water, but serum
potassium levels are increased in such patients.
Educational objective:
Diuretic abuse leads to increased excretion of water and electrolytes by the kidneys.
Dehydration, weight loss, orthostatic hypotension as well as hypokalemia and hyponatremia
result. Urinary sodium and potassium will be elevated. Patients with eating disorders sometimes
abuse diuretics to induce weight loss.
The correct answer is: . Diuretic abuse

21. A 65-year-old man comes to the office and complains of pain and a rash with blisters
over the left side of his chest. He has experienced pain over the area for the past 2 days.
This morning, he noticed blisters while changing his shirt. He also complains of malaise
and headache. His pulse is 82/min, blood pressure is 140/90 mmHg, respirations are
14/min and temperature is 36.8°C (98.4°F). Physical examination reveals grouped, tense
vesicles arranged in a band along the left side of his chest. Which of the following is the
most likely etiology of his condition?
Select one:
o . Human papilloma virus
o . Varicella zoster virus
o . Poison ivy
o . Herpes simplex virus
o . Poxvirus

This patient has a classic presentation of herpes zoster (shingles), which is caused by the
varicella zoster virus (VZV). After an initial VZV infection (chicken pox), latent infection is
established in the sensory dorsal root ganglia. Shingles occurs when there is reactivation of this
latent infection. It most commonly occurs in older individuals, immunosuppressed patients or
during periods of stress. Pain usually precedes the vesicular eruption by 48 hours or more. The
eruption is most commonly unilateral and occurs in a band-like fashion along the affected
dermatome. Disseminated disease can occur in severely immunosuppressed patients.
(Choice A) Herpes simplex virus infection usually presents as small grouped vesicles on an
erythematous base over the orolabial or genital areas. Regional lymphadenopathy may be
present.
(Choice C) Poison ivy causes allergic contact dermatitis, which appears as a linear vesicles and
erythema that occurs after brushing with the plant. Allergic contact dermatitis is usually pruritic
and distributed in nondermatomal streaks on the extremities while shingles is painful.
(Choice D) The human papilloma virus causes warts and some forms of squamous cell
carcinoma.
(Choice E) Poxviruses cause smallpox, molluscum contagiosum and milkers' nodules. Each of
these infections has a different clinical presentation, but none resemble zoster.
Educational objective:
Shingles is characterized by a vesicular eruption that occurs in a dermatomal distribution and is
often preceded by pain. It is caused by the varicella zoster virus, which also causes chicken pox.
The correct answer is: . Varicella zoster virus

22. A 46-year-old construction worker is brought to the clinic by his wife because she has
noticed an unusual growth on his left ear for the past 8 months (see photo below). The
patient explains that, except for occasional itching, the lesion does not bother him. On
physical examination, you notice an 8-mm pearly papule with central ulceration and a
few small dilated blood vessels on the border. What is the natural course of this lesion if
left untreated?

Select one:
o . This is a benign lesion and will not change
o . Regression over time
o . Local invasion of surrounding tissue
o . Local invasion of surrounding tissue and metastasis via lymphatic spread
o . Disseminated infection resulting in septicemia

(Fauci, pp 310, 545-548.) This is a classic description of basal cell carcinoma. Basal cell carcinoma
is a malignant neoplasm of the epidermal basal cells that clinically presents as a pearly papule or
nodule with a central ulceration, raised borders, and telangiectasias. Basal cell carcinomas are
locally invasive and rarely metastasize; distant spread is reported in fewer than 0.1% of these
cancers. Invasion of surrounding tissue and metastasis are more frequently seen in squamous
cell carcinoma. Squamous cell carcinoma is malignant neoplasm of the keratinocytes; it is much
more aggressive than basal cell carcinoma, grows rapidly, and may metastasize via lymphatic
spread. Bacterial infections such as meningococcemia and necrotizing fasciitis could result in
septicemia without appropriate treatment but are acute, not chronic, conditions.
The correct answer is: . Local invasion of surrounding tissue

23. A 32-year-old female is brought to the emergency department with complaints of


weakness, tingling and numbness of her extremities. She is not on any medication. Her
pulse is 90/min, respirations are 14 /min and blood pressure is 110/70 mm Hg. The
physical examination is unremarkable. Her laboratory profile is shown below: Blood pH
7.56, HCO3- 37 mEq/L, Urine Na+ 16 mEq/L, Urine K+ 20 mEq/L, Urine Cl- 7 mEq/L,
Serum sodium 135 mEq/L, Serum potassium 2.9 mEq/L, Serum chloride 92 mEq/L, Blood
urea nitrogen 22 mg/dl, Serum creatinine 0.9 mg/dl. Which of the following is the most
likely cause of this patient's condition?
Select one:
o . Persistent diarrhea
o . Type I renal tubular acidosis
o . Bartter's syndrome
o . Hyperventilation syndrome.
o . Surreptitious vomiting

Metabolic alkalosis is characterized by an alkaline pH and a primary increase in the serum


bicarbonate level (> 24 mEq/L). It can be classified into two broad categories, namely chloride-
sensitive (hypochloremic, saline-responsive) and chloride-resistant ( normochloremic, saline-
unresponsive) metabolic alkalosis. Chloride-sensitive metabolic alkalosis is characterized by a
urinary chloride level< 20 mEq/day and signs of volume depletion. The common underlying
pathophysiology in all causes of chloride-sensitive metabolic alkalosis involves ECF volume
contraction. Volume contraction causes increased mineralocorticoid action, which in turn causes
bicarbonate retention, H+ loss and K+ loss. The urinary chloride remains low due to avid renal
retention of NaCI and water. Some causes of this condition are thiazide or loop diuretics and
loss of gastric secretions, such as with surreptitious vomiting (i.e. purging in bulimia nervosa).
This condition can be corrected with saline infusion to restore ECF volume.
(Choice B) Persistent diarrhea causes a non-anion gap metabolic acidosis.
(Choice C) Chloride-resistant metabolic alkalosis is characterized by a urinary chloride level >20
mEq/day and ECF volume expansion. Some disorders associated with chloride-resistant
metabolic alkalosis are primary hyperaldosteronism, Bartter syndrome, Gitelman's syndrome,
and excessive black licorice ingestion. Chloride-resistant metabolic alkalosis is not corrected by
saline infusion.
(Choice D) All forms of renal tubular acidosis cause non-anion gap metabolic acidosis.
(Choice E) Hyperventilation causes respiratory alkalosis.
Educational Objective:
Metabolic alkalosis can be classified into two broad categories, chloride-sensitive and chloride-
resistant, based on urinary chloride levels and ECF volume status. Chloride sensitive metabolic
alkalosis is associated with low urinary chloride excretion and volume contraction.
The correct answer is: . Surreptitious vomiting

24. A 28-year-old woman is brought to see a psychiatrist by her mother. The patient insists
that nothing is wrong with her, but the mother notes that the patient has been slowly
but progressively isolating herself from everyone. She now rarely leaves the house. The
mother says she can hear the patient talking to “people who aren’t there” while she’s in
her room. On examination, the patient is noted to have auditory hallucinations and the
delusional belief that her mother is going to kick her out of the house so that it can be
turned into a theme park. Which of the following is the lifetime prevalence for this
disorder?
Select one:
o . 10%
o . 3%
o . 5%
o . 1%
o . 15%

(Kaplan, p 468.) Schizophrenia affects 1% of the adult population. The incidence is comparable
in all societies. The 1-year incidence rate is 0.2 per 1000.
The correct answer is: . 1%

25. A 34-year-old homosexual male with a history of HIV presents to the clinic complaining
of a wheezing and multiple violaceous plaques and nodules on his trunk and extremities.
Physical examination of the oral mucosa reveals similar findings on his palate, gingiva,
and tongue. Chest x-ray is also significant for pulmonary infiltrates. What is the most
likely pathogenesis of this process?
Select one:
o . Disseminated HSV infection
o . Infection with human herpesvirus 6
o . Angioproliferative disease caused by infection with human herpesvirus 8
o . Proliferation of neoplastic T cells
o . Infection with Mycobacterium avium due to decreasing CD4 count

(Fauci, p 1186-1188; Wolff, pp 536-540.) This patient has Kaposi sarcoma (KS). In HIV-infected
individuals, KS is associated with human herpesvirus 8 (HHV-8). KS lesions are derived from the
proliferation of endothelial cells in blood/lymphatic microvasculature. They present as
violaceous patches, plaques, and/or nodules on the skin, mucosa, and/or viscera. The pulmonary
infiltrates observed on the chest x-ray of this patient are the result of visceral KS affecting the
lungs. Proliferation of neoplastic T cells is seen in cutaneous T cell lymphomas such as mycosis
fungoides. Human herpesvirus 6 (HHV-6) is the cause of exanthema subitum (roseola) in
children. It consists of 2- to 3-mm pink macules and papules on the trunk following a fever.
Mycobacterium avium causes pulmonary infection in HIV patients with a CD4 count < 50/μL.
Immunodeficient patients or patients with HIV who are infected with HSV can present with the
disseminated form of the disease. However, these lesions consist of a vesicular rash that is
different from the violaceous plaques observed in KS.
The correct answer is: . Angioproliferative disease caused by infection with human herpesvirus 8

26. A 60-year-old male presents to the emergency room with the chief complaint of
progressive exertional dyspnea and fatigue. He denies any chest pain, syncope, cough, or
edema. He suffered an acute anterior wall myocardial infarction one month ago. Chest
auscultation reveals bilateral crackles in his lower chest. Cardiac auscultation reveals a
pansystolic murmur at the apex with radiation to the axilla. ECG shows previously present
unchanged Q waves and a persistent ST segment elevation in the anterior leads. Based
on these findings, what is the most likely underlying cause of his symptoms?
Select one:
o . Recurrent ischemia
o . Ventricular free wall rupture
o . Interventricular wall rupture
o . Ventricular aneurysm
o . Right ventricular infarction

Ventricular aneurysm is a complication that may occur days to months after myocardial
infarction. Common consequences of ventricular aneurysms include symptoms of CHF,
ventricular arrhythmias, mitral regurgitation, and/or thrombus formation. This patient's physical
exam finding of a pansystolic murmur at the apex with radiation to the axilla is suggestive of
mitral regurgitation, the result of aneurysmal alterations of the ventricular geometry. Persistent
ST elevations are often seen on ECG in ventricular aneurysm. Echocardiography demonstrating
dyskinetic wall motion of a portion of the left ventricle may be used to confirm the diagnosis.
(Choice A} Interventricular wall rupture typically occurs around 5 days after infarction and causes
an acute left-to-right shunt with right sided heart failure and new-onset systolic murmur heard
best at the left lower sternal border.
(Choice B) Ventricular free wall rupture typically occurs around 5 days after myocardial infarction
and causes acute pericardia! tamponade and rapid decompensation with pulseless electrical
activity.
(Choice C) Pulmonary infarction secondary to pulmonary embolism can cause dyspnea, but it
should not cause new onset mitral regurgitation or ST elevations on ECG.
(Choice D) Recurrent ischemia is a consideration, given the ST elevations on ECG. However, it
appears that the ST elevations and Q waves are unchanged since the initial infarction. These
findings are more consistent with ventricular aneurysm.
(Choice E) Right ventricular infarction can occur with inferior wall Ml, but usually not with
anterior wall MI. Since LV function is relatively preserved in RV infarction, crackles would not be
expected on lung exam.
(Choice F) Papillary muscle rupture typically occurs 3-7 days after an infarct and can cause
severe acute mitral regurgitation and pulmonary edema. It does not typically cause persistent ST
elevations on ECG.
(Choice H) Acute pericarditis can occur in the first several days after infarction, while Dressier's
syndrome is an immune-mediated pericarditis that can occur weeks to months after infarction.
Diffuse ST elevations are typically present on ECG, and positional chest pain is usually the
primary complaint.
Educational objective:
Ventricular aneurysm is a late complication of myocardial infarction characterized by persistent
ST elevations on ECG. Patients may have CHF, ventricular arrhythmias, and/or thrombus
formation along with mitral regurgitation.
The correct answer is: . Ventricular aneurysm

27. A 38-year-old Mexican male presents to the emergency department with a history of
weight loss, fever, cough with sputum, nausea, abdominal pain, and postural dizziness
for the last three months. Adrenal insufficiency is suspected, and cosyntropin (synthetic
ACTH) stimulation test is performed. The rise of serum cortisol following an injection of
cosyntropin is grossly subnormal. CT scan of the abdomen shows calcification of both
adrenal glands. What is the most likely cause of this patient's adrenocortical
insufficiency?
Select one:
o . Human immunodeficiency virus infection
o . Tuberculosis
o . Adrenal tumor
o . Adrenal haemorrhage
o . Autoimmune adrenalitis

This patient has the typical clinical symptoms, laboratory findings (i.e., no rise in serum cortisol
levels following the injection of cosyntropin), and CT scan findings (i.e., calcification in both
adrenal glands) of primary adrenal insufficiency. The causes of primary adrenal insufficiency
include tuberculosis, fungal infections, and cytomegalovirus infections. Adrenal calcification is a
typical feature of adrenal tuberculosis, which is therefore the most likely cause of this patient's
adrenal insufficiency.
In the past, tuberculosis was the most common cause of primary adrenal insufficiency.
Tuberculosis still continues to be the prominent cause of primary adrenal insufficiency in
developing countries. In developed countries, however, the most common cause of primary
adrenal cortical insufficiency is autoimmune adrenalitis (currently responsible for more than 80%
of patients with primary adrenal cortical insufficiency). Treatment of tuberculosis usually does
not generally result in normalization of adrenal functions. Most patients require lifelong
replacement with glucocorticoids and mineralocorticoids.
(Choice A) Adrenal insufficiency occurs commonly in patients with HIV infection. Common
causes of adrenal insufficiency in patients with HIV infection include adrenal infections by
cytomegalovirus, atypical mycobacteria, fungi and tubercular bacilli. Sometimes, medications
used in the treatment of fungal infections (e.g., ketoconazole) can precipitate adrenal
insufficiency by inhibiting adrenal steroid synthesis.
(Choice C) Autoimmune adrenalitis is responsible for more than 80% of cases of primary adrenal
insufficiency in developed countries. The pathophysiology involves antibodies against one or
more adrenal enzymes responsible for steroid synthesis. CT scan does not show adrenal
calcification.
(Choice D) Primary adrenal insufficiency is very rare with adrenal metastasis. Calcification is not
seen in most cases.
(Choice E) Adrenal hemorrhage can lead to primary adrenal insufficiency; however, the onset of
clinical features is acute (unlike adrenal insufficiency secondary to infections, in which symptoms
progress over a few months), and adrenal imaging shows the presence of blood in the acute
stages.
(Choice F) Adrenal insufficiency in adrenoleukodystrophy is caused by the accumulation of very
long chain fatty acids (VLCFA). Imaging reveals enlarged adrenal glands without any
calcifications.
Educational Objective:
The most likely diagnosis in a patient who presents with clinical features of adrenal insufficiency
and calcifications in the adrenal glands is adrenal tuberculosis. Adrenal tuberculosis continues to
be the prominent cause of primary adrenal insufficiency in developing countries. In contrast,
autoimmune adrenalitis is currently the most common cause of primary adrenal insufficiency in
developed countries.
The correct answer is: . Tuberculosis

28. A 33-year-old woman develops mild epigastric abdominal pain with nausea and
vomiting of 2 days duration. Her abdomen is tender on palpation in the epigastric
region, and the remaining examination is normal. Her white count is 13,000/mL, and
amylase is 300 U/L (25–125 U/L). Which of the following is the most common
predisposing factor for this disorder?
Select one:
o . malignancy
o . alcohol
o . gallstones
o . hypertriglyceridemia
o . drugs

The common causes of acute pancreatitis are alcohol, gallstones, metabolic factors, and drugs.
Gallstones still remain the most common cause of pancreatitis. (Kasper, p. 1896)
The correct answer is: . gallstones

29. A 24-year-old male experiences syncope while shovelling snow. He regained


consciousness within one minute. He has been having some shortness of breath and
chest pains recently, mostly related to exercise. He denies any illicit drug use. His
temperature is 37.2°C (98.9°F), and blood pressure is 126/76 mmHg, pulse is 76/min and
respirations are 14/min. Physical examination shows a well-built male in no apparent
distress. Lungs are clear. A crescendo-decrescendo systolic murmur is heard along the
left sternal border without carotid radiation. Chest X-ray is normal. Which of the
following is the most likely cause of his syncopal episode?
Select one:
o . Aortic dissection
o . Coronary atherosclerosis
o . Left ventricular hypertrophy
o . Atrioventricular conduction delay
o . Mitral valve degeneration

This patient's crescendo-decrescendo systolic murmur along with left sternal border without
carotid radiation is the description of the murmur present in hypertrophic cardiomyopathy,
hence left ventricular hypertrophy is the correct answer. The location of the murmur and lack of
carotid radiation helps to differentiate from aortic stenosis. Common symptoms of hypertrophic
cardiomyopathy include syncope, dyspnea, and chest pain; all of which this patient has
experienced. Syncope in HCMP is multifactorial. In large part it is due to outflow obstruction
from the hypertrophied myocardium. However, syncope can also be secondary to arrhythmia,
ischemia, and a ventricular baroreceptor response that inappropriately causes vasodilation.
{Choice A} Atrioventricular conduction delay occurs when conduction through the AV node is
slowed, typically due to medications such as 13-Biockers or ischemic heart disease. This typically
results in bradycardia, but this patient's heart rate is within the normal range.
{Choice C) Syncope can occur with aortic dissection, particularly with those involving the
ascending aorta. However, the primary complaint in patients with aortic dissection is chest pain,
which this patient does not appear to have at the current time. Painless aortic dissection is rare.
Most patients with aortic dissection are also older and hypertensive.
{Choice D) Coronary atherosclerosis is unlikely in a patient this young without any underlying
coronary disease risk factors. It would also be unlikely to cause a murmur by itself.
{Choice E) Mitral degeneration leading to regurgitation rarely causes syncope. In addition, mitral
regurgitation typically causes a holosystolic murmur as opposed to the crescendo-decrescendo
type murmur heart in this patient.
Educational objective:
Syncope in a young patient with a crescendo-decrescendo murmur at the lower left sternal
border is most likely due to hypertrophic cardiomyopathy. Syncope in hypertrophic
cardiomyopathy is often multifactorial and can be due to outflow obstruction, arrhythmia,
ischemia, and a ventricular baroreceptor response that inappropriately causes vasodilation.
The correct answer is: . Left ventricular hypertrophy

30. A 57-year-old Caucasian female is diagnosed with deep venous thrombosis of the right
leg that was confirmed with Doppler ultrasonography. She was diagnosed with
pneumonia and empyema one week earlier, and treated with chest tube, antibiotics and
bed rest. On her 6th day of anticoagulation therapy, she develops right hemiparesis and
slight motor aphasia. The laboratory findings are: Red blood cells 4.3 million/mm3,
Hemoglobin 14.00 g/dL, White blood cells 7,000/cmm, Platelets 50,000/cmm, APTT 60
sec (N < 25-40 sec), Fibrin degradation products negative. The emergency head CT scan
does not reveal blood in the subarachnoid space or brain parenchyma. Which of the
following is the most probable cause of this patient's condition?
Select one:
o . Venous thromboembolism
o . Disseminated intravascular coagulation
o . Non-immune platelet degradation
o . Platelet sequestration and redistribution
o . Antibody-mediated platelet activation

Hospitalized patients who develop deep venous thrombosis (DVT) after a period of bed rest are
standardly treated with heparin. An adverse effect of heparin therapy is thrombocytopenia,
along with thrombosis, which may manifest as an acute ischemic stroke. The combination of
arterial/venous thrombosis and thrombocytopenia in patients receiving heparin therapy is highly
suggestive of heparin-induced thrombocytopenia. Antibodies against heparin-platelet factor-4
complex are responsible for this condition. The antibodies activate platelets, thereby leading to
premature removal of platelets from the circulation, and consequent thrombotic complications.
The other listed mechanisms of platelet degradation and thrombosis are less likely to occur in
this case.
(Choice A) Non-immune platelet degradation is characteristic for microangiopathic anemias.
(Choice C) Disseminated intravascular coagulation is characterized by thrombocytopenia,
thrombosis and/or bleeding. Fibrin degradation products are typically present.
(Choice E) Platelet sequestration and redistribution may be present in patients with significant
splenomegaly.
(Choice B) Venous thromboembolism typically leads to pulmonary artery occlusion. It can cause
stroke if the patient has persistent patent foramen ovale. In this case, the patient's HIT most
likely resulted in the formation of an arterial clot, which eventually caused a stroke.
Educational Objective:
The combination of arterial/venous thrombosis and thrombocytopenia in patients receiving
heparin therapy is highly suggestive of heparin-induced thrombocytopenia. Antibodies against
heparin-platelet factor-4 complex are responsible for the disease.
The correct answer is: . Antibody-mediated platelet activation

31. A 26-year-old man comes to the emergency department because he is "suffering from
the worst headache of his life." He feels nauseated and is photosensitive. His blood
pressure is 160/90 mm Hg, heart rate is 88/min, and temperature is 36.5°C (97.7°F). The
physical examination reveals no focal neurological symptoms, except for some
meningismus and vertigo, which is not localized to either side. CSF examination reveals
the presence of xanthochromia. What is the major cause of morbidity and mortality in a
patient with the above condition?
Select one:
o . Vasospasm with symptomatic ischemia and infarction
o . Secondary infection
o . Nimodipine use
o . Post-surgical complications
o . Post-angiographic complications

This patient has a subarachnoid hemorrhage, and is at risk for subsequent vasospasm of the
arteries at the base of the brain. Vasospasm following SAH occur in approximately 30% of
patients, and is the major cause of morbidity and mortality in such patients. In affected patients,
the signs of ischemia usually appear in about 7 days after the SAH.
(Choice E) Calcium channel blockers (e.g., nimodipine) are used to prevent vasospasm in
patients with SAH.
(Choices A, C, and D) Secondary infections, as well as complications after angiography and
surgery, are not as prevalent as vasospasm following SAH.
Educational Objective:
Vasospasm is the major cause of morbidity and mortality in patients with subarachnoid
hemorrhage (SAH). Calcium channel blockers (e.g., nimodipine) are used to prevent vasospasm
in patients with SAH.
The correct answer is: . Vasospasm with symptomatic ischemia and infarction

32. A 52-year-old male is referred to the neurology clinic for the evaluation of EEG
abnormalities. He presented with rapidly increasing memory impairment, and denied any
history of seizures or head trauma. The physical examination revealed no abnormalities,
except a myoclonus. An extensive work-up ruled out the presence of any medical illness;
however, the EEG report revealed sharp, triphasic and synchronous discharges. Which of
the following abnormalities is most likely in this patient?
Select one:
o . Neurodegeneration of frontal and temporal lobes
o . Histopathological findings of neurofibrillary tangles and amyloid plaques
o . Spongiform encephalopathy caused by a prion
o . Loss of nigrostriatal dopaminergic neurons
o . Defect in an autosomal dominant gene on chromosome 4

This patient's presentation is classic for Creutzfeldt-Jakob disease. The typical features, as stated
above, include rapidly progressive dementia, myoclonus and sharp, triphasic, synchronous
discharges on EEG. Creutzfeldt-Jakob disease is a spongiform encephalopathy caused by a slow
virus (prion).
(Choice A) Huntington's dementia is caused by a defect in an autosomal dominant gene on
chromosome 4. The pathology involves striatal neuro-degeneration, and the classic features are
early onset dementia (between 35 - 50 years of age), progressive choreiform movements of all
limbs, grimacing and ataxic gait.
(Choice C) Parkinson's disease is characterized by a progressive loss of nigrostriatal
dopaminergic neurons. The characteristic clinical features are bradykinesia, resting tremors,
cogwheel rigidity, classic shuffling gait and masked facies.
(Choice D) Alzheimer's dementia is a neurodegenerative disorder that is characterized by the
following histopathological findings: amyloid plaques, neurofibrillary tangles, and selective loss
of cholinergic neurons.
(Choice E) Pick's disease is a neurodegenerative disease of the frontal and temporal lobes. It
resembles Alzheimer's dementia clinically, but is more commonly seen in females. The onset of
symptoms is around 50 years. Personality and language changes are often more prominent than
cognitive symptoms.
Educational Objective:
Creutzfeldt-Jakob disease is characterized by rapidly progressive dementia, myoclonus and
sharp, triphasic, synchronous discharges on EEG. This spongiform encephalopathy is caused by a
slow virus (prion).
The correct answer is: . Spongiform encephalopathy caused by a prion

33. A 47-year-old woman loses consciousness for 2 minutes while shopping in a


supermarket. In the emergency room, she recounts feeling nausea and warmth
spreading over her body immediately before passing out. She has never had a similar
episode before. She has not seen a doctor for several years and does not take any
medications, nor does she use tobacco, alcohol or drug. Her family history is
unremarkable. Which of the following most likely caused this episode?
Select one:
o . Heat valve disease
o . Seizure
o . Neurocardiogenic syncope
o . Orthostatic hypotension
o . Cardiac arrythmia

Neurocardiogenic syncope, also known as vasovagal syncope, is a common cause of fainting. It


occurs due to excessive vagal tone and is characterized by nausea, diaphoresis, tachycardia, and
pallor immediately prior to the syncopal episode. It frequently occurs in response to stress, pain,
and certain bodily actions (e.g. urination). It is particularly common in young women. Vasovagal
syncope can be diagnosed with the tilt table test.
(Choice A) Conduction disorders (e.g. AV block), tachyarrhythmias (e.g. ventricular or
supraventricular tachycardia), and disorders of automaticity (e.g. sick sinus syndrome) can all
cause fainting. Generally there are no preceding signs or symptoms, except for palpitations in
some. Patients usually have underlying cardiac disease.
(Choice B) Seizures and syncopal episodes are sometimes difficult to distinguish. This patient
does not have clear signs of seizure (e.g. tongue biting), a history of seizures, or a post-ictal
state, making syncope more likely.
(Choice D) Syncope due to aortic stenosis most often occurs with activity. Dyspnea, chest pain,
and fatigue on exertion are symptoms of aortic stenosis that generally occur earlier in the
disease course.
(Choice E) Orthostatic hypotension is defined as a drop in systolic blood pressure greater than
20 mm Hg after the patient rises to a standing position. It is most common in the elderly,
diabetics, those with autonomic neuropathy (e.g. Parkinson's disease), those with hypovolemia,
and people taking diuretics, vasodilators, or adrenergic-blocking agents. Prolonged recumbence
increases the risk. Patients commonly experience pre-syncopallightheadedness.
(Choice F) Transient ischemic attacks (TIAs) can produce syncope, but are a rare cause because
the TIA must affect the posterior circulation and brain stem in order for syncope to occur. This
patient has no risk factors for stroke.
Educational Objective:
Neurocardiogenic, or vasovagal, syncope occurs due to excessive vagal tone. Episodes are
preceded by nausea, diaphoresis, tachycardia, and pallor. Pain, stress, and situations like medical
needles and urination can all precipitate vasovagal syncope.
The correct answer is: . Neurocardiogenic syncope

34. A 53-year-old man presents to your office complaining of weakness and exertional
dyspnea over the last week. He denies chest pain and palpitations. He has no other past
medical history. His father died of a myocardial infarction at age 55. On exam, his heart
rate is 100/min and blood pressure is 90/60 mmHg. Internal jugular venous pulsation is
observed 7 cm above the sternal angle. Lungs are clear to auscultation. Heart sounds are
muffled. Which of the following is the most likely cause of this patient's complaints?
Select one:
o . Left ventricular outflow obstruction
o . Pulmonary hypertension
o . Increased right ventricular compliance
o . Decreased cardiac contractility
o . Decreased left ventricular preload

This patient has all three of Beck's triad for cardiac tamponade: hypotension, jugular venous
distention (JVD), and muffled heart sounds. Cardiac tamponade occurs when the pericardia!
space fills with fluid. This can result from many causes, including infection, myocardial infarction,
malignancy, uremia, and trauma. Of note, the degree of clinical symptoms depends largely on
how quickly the pericardia! fluid accumulates. The faster it accumulates, the sooner symptoms
will develop.
The pathophysiology of cardiac tamponade is as follows. When fluid in the pericardia! sac
causes the pressure to rise above the diastolic pressure of the ventricles, the ventricles become
less able to expand to accept venous return to the heart. As a result of the decrease in preload,
stroke volume falls and thus cardiac output is reduced. The decrease in cardiac output is further
exacerbated during inspiration because the lower intrathoracic pressure allows more systemic
venous blood to return to the right ventricle, causing the intraventricular septum to bow into the
left ventricle and further reduce left ventricular filling. This phenomenon is responsible for the
finding of pulsus paradoxus on exam. A hepatojugular reflex might also be observed on exam.
(Choice A} Decreased cardiac contractility is observed in patients with systolic heart failure. They
will typically have signs of both left ventricular failure (e.g., crackles on pulmonary exam and an
S3} and right ventricular failure (e.g., JVD and peripheral edema}. The patient in this vignette
does not have signs of left ventricular failure.
(Choice B) Left ventricular outflow obstruction results from critical aortic stenosis or from
hypertrophic obstructive cardiomyopathy (HOCM}. Although this patient does have exertional
dyspnea, there is no other suggestion of aortic stenosis (e.g., syncope, systolic murmur} in this
vignette, and most patients with HOCM present at a much earlier age. Importantly, the systolic
murmur of HOCM will decrease with squatting and increase with straining.
(Choice D) Pulmonary hypertension can result from increased pulmonary arterial pressure, as
with primary pulmonary hypertension, or with increased pulmonary venous pressure, as occurs
in left ventricular failure. The cardiac exam will reveal a loud S2. Pulmonary arterial hypertension
is typically an indolent process and does not present as right heart failure until late in the
disease.
(Choice E) Right ventricular compliance will decrease, not increase, in cardiac tamponade.
Educational Objective:
The classic physical exam findings of cardiac tamponade are pulsus paradoxus and Beck's triad:
hypotension, jugular venous distention, and muffled heart sounds. When the pericardia! space
fills with fluid, the ventricles are less able to expand to accept venous return to the heart. As a
result, overall preload to the heart decreases.
The correct answer is: . Decreased left ventricular preload

35. A 27-year-old woman presents to the emergency room with a panic attack. She appears
healthy except for tachycardia and a respiratory rate of 30. Electrolytes include calcium
10.0 mg/dL, albumin 4.0 g/dL, phosphorus 0.8 mg/dL, and magnesium 1.5 mEq/L.
Arterial blood gases include pH of 7.56, PCO2 21 mm Hg, and PO2 99 mm Hg. Which of
the following is the most likely cause of the hypophosphatemia?
Select one:
o . Vitamin D deficiency
o . Hypomagnesemia
o . Poor dietary intake
o . Respiratory alkalosis with intracellular shift
o . Hyperparathyroidism

(Fauci, p 295.) Respiratory alkalosis is one of the commonest causes of hypophosphatemia; it


results from shift of phosphate from the extracellular to the intracellular space.
Hypomagnesemia alone would increase phosphorus by decreasing parathormone effect. Hyper-
parathyroidism can decrease phosphorus, but not to this degree; also, calcium is not elevated.
Severe hypophosphatemia is seen with malnutrition, especially during the refeeding stage when
carbohydrate intake causes phosphate to shift into the intracellular space. Such patients have
clear clinical evidence of malnutrition. In addition, malnutrition almost always causes
hypoalbuminemia. Vitamin D deficiency is uncommon in this age group and would be
associated with hypocalcemia.
The correct answer is: . Respiratory alkalosis with intracellular shift

36. A long-time patient calls your office to speak with you directly. She voices frustration
with your receptionist, who has repeatedly refused to allow her to schedule an
appointment to see you. The patient says that at her last visit, she had a heated
argument with the receptionist about a minor detail regarding her insurance coverage.
When she has called since then, the receptionist always responds by saying, "I'm sorry,
but there are too many patients waiting ahead of you." Which of the following defense
mechanisms is the receptionist demonstrating?
Select one:
o . Acting out
o . Distortion
o . Displacement
o . Introjection
o . Passive-aggressive behaviour

Psychological defense mechanisms are our unconscious means of responding and adapting to
different situations while still preserving our self-image. These mechanisms usually serve to
decrease anxiety, and can be classified as immature, neurotic, or mature. Immature defense
mechanisms are more commonly used by children or adolescents and result in socially
unacceptable behavior that prevents the individual from coping with reality. Neurotic defense
mechanisms are fairly common in adults and provide short-term relief, but often result in long-
term complications. Mature defense mechanisms are most often used by adults and allow for a
"healthy" and more constructive approach to reality.
Passive-aggressive behavior is an immature defense mechanism in which an individual expresses
his aggression toward another person with repeated, passive failures to meet the other person's
needs.
(Choice A) An immature defense mechanism, distortion is the altered perception of disturbing
aspects of external reality in an effort to make it more acceptable. An example would be an
intravenous drug abuser who contracts hepatitis C and blames his condition on inadequate
control of hepatitis C within the community.
(Choice B) A neurotic defense mechanism, displacement is the shifting of emotions associated
with an upsetting object or person to a safer alternate object or person that represents the
original in some regard. An example would be a woman who is angry with her husband and
throws away his baseball card collection as retaliation.
(Choice C) Acting out is an immature defense mechanism that involves directly expressing an
unconscious wish or impulse to avoid addressing the accompanying emotion. An example
would be a child who throws a temper tantrum because he is disappointed that his mother
won't take him to the park.
(Choice D) An immature defense mechanism, introjection is the assimilating of another person's
attitude into one's own perspective. An example would be a battered woman who believes her
husband is right when he says she is worthless.
(Choice F) Suppression is a mature defense mechanism that involves intentionally postponing
the exploration of anxiety-provoking thoughts by substituting other thoughts. This allows for
the individual to cope with the present reality instead. An example would be a woman who
focuses on her children's needs instead of thinking about her father's diagnosis of cancer.
(Choice G) An immature defense mechanism, denial is the failure to accept a disturbing aspect
of external reality. An example would be a patient diagnosed with a terminal illness who refuses
to accept that he will likely die soon.
(Choice H) A neurotic defense mechanism, dissociation involves completely blocking off
disturbing thoughts or feelings from consciousness in an attempt to avoid emotional upset. An
example would be a patient who was rescued from a burning building and now denies any
memory of the event.
Educational Objective:
Passive-aggressive behavior is a psychological defense mechanism in which an individual
expresses his aggression toward another person with repeated, passive failures to meet the
other person's needs.
The correct answer is: . Passive-aggressive behaviour

37. A 19-year-old woman was traveling in a rural area of South America. She returned 3
weeks ago and, over the past few days, has gradually developed lower abdominal pain
and diarrhea. Now the symptoms are much worse with 10 stools a day consisting mostly
of mucus and blood. She is afebrile, the abdomen is tender in left lower quadrant, and
the remaining examination is normal. Her stool is mostly comprised of blood and mucus.
Which of the following is the most likely causative organism?
Select one:
o E. histolytica infection
o Shigella infection
o Salmonella infection
o Escherichia coli infection
o Vibrio parahaemolyticus infection

The time course, clinical features, and stool examination are characteristic of intestinal
amebiasis. It is very common in most developing countries in the tropics and infects about 10%
of the world’s population. It is the third most common cause of death from parasitic disease.
(Kasper, p. 1215)
The correct answer is: E. histolytica infection
38. A 65-year-old man comes to the emergency room complaining of 2 days of severe pain
and swelling of his right knee. He denies any recent respiratory illness, diarrhea or urinary
symptoms. He has a history of severe degenerative joint disease and underwent total
knee replacement four years ago. He drinks one can of beer every night and does not
use tobacco or illicit drugs. His temperature is 40°C (104°F), blood pressure is 120/60
mmHg, and pulse is 110/min. Examination shows warmth, swelling, and tenderness over
the knee. His range of motion is restricted. Synovial fluid white blood cell count is
98,000/microl. Which of the following is the most likely cause of his current condition?
Select one:
o . Staphylococcus aureus
o . Streptococcus species
o . Borrelia burgdorferi
o . Chlamydia trachomatis
o . Neisseria gonorrhoea

This patient is suffering from septic arthritis. The presence of a prosthetic joint significantly
increases the risk of joint infection. Septic arthritis classically presents with the acute onset of a
painful, swollen joint, limited range of motion, and fever. However, crystal-induced arthritis may
present similarly, and therefore this patient's WBC count of 98,000/microL on synovial fluid
analysis is critical to the diagnosis. A WBC count of 10,000- 50,000/microL may be found in
crystal-induced arthritis, whereas a count of 50,000- 150,000/microL is strongly suggestive of
septic arthritis. Furthermore, while fever is sometimes found in crystal-induced arthritis, a
temperature of 1 04 •F is more consistent with a purulent infection. The knee is the most
common joint affected by septic arthritis, and hematogenous spread is the most common
mechanism. Staphylococcus aureus is the most frequent cause of prosthetic joint septic arthritis.
(Choice A} After Staphylococcus aureus, streptococcal species are the second most common
cause of nongonococcal arthritis. The most reliable way to identify the causative organism in
septic arthritis is a gram stain and culture. In the absence of gram stain, this patient's
demographics, prosthetic joint, and aggressive infection suggest Staph. aureus is more likely
than a streptococcal infection.
(Choice C) Neisseria gonorrhea is the most common cause of septic arthritis in young, sexually
active individuals. It may present as a purulent arthritis affecting one or more joints, or as a
polyarthritis associated with tenosynovitis and skin rash.
(Choice D) Reactive arthritis, often linked to genitourinary infection with Chlamydia trachomatis,
presents with the triad of arthritis, conjunctivitis, and urethritis.
(Choice E) Borrelia burgdorferi causes Lyme disease. Arthritis, which often affects the knee, is a
symptom of chronic Lyme disease. A history of tick bite and erythema migrans is classic for
Lyme disease. Other chronic sequelae include facial palsy and cardiac conduction abnormalities.
(Choices F & G) Blastomycosis and Mycobacterium tuberculosis are causes of fungal and
tuberculous arthritis, respectively. Both are far less common than bacterial arthritis, are less
virulent and acute, and present with a lower synovial fluid WBC count.
(Choice H) Salmonella species and other gram-negative enteric pathogens may cause septic
arthritis in immunocompromised and elderly patients.
Educational objective:
Staphylococcus aureus is the most frequent cause of prosthetic joint septic arthritis.
The correct answer is: . Staphylococcus aureus

39. A 73-year-old Caucasian man is brought to the office by his daughter, who is concerned
that he might be depressed. He is a retired surgeon, and has lived alone ever since his
wife died a year ago. His daughter visits him every 6 months; she feels bad about not
being able to visit him more frequently because her job and family keep her very busy.
He denies having any feelings of sadness, guilt, weight loss, loss of appetite, suicidal
ideation, deafness, vertigo, and decreased or blurred vision. His medical problems
include hypertension, diabetes mellitus-type 2 and a myocardial infarction 10 years ago.
His current medications are glyburide, aspirin and enalapril. He denies the use of
tobacco, alcohol, or drugs. His vital signs are within normal limits. He appears withdrawn,
less energetic than usual, and walks stiffly. He sits with a stooped posture. He has a fixed
facial expression, and his voice sounds monotonous. His deep tendon reflexes are 2+.
Sensations and motor strength are normal. There is increased resistance to passive
flexion. Which of the following types of gait is most likely to be present in this patient?
Select one:
o . Hypokinetic gait
o . Gait disequilibrium
o . Waddling gait
o . Cerebellar ataxia
o . Spastic gait

The most likely diagnosis of this patient is Parkinsonism. Hallmarks of this disease include a
mask-like , immobile facial expression, bradykinesia, resting tremor, rigidity and festinating gait.
There is a general slowing of all motor activity (hypokinetic gait). Patients assume a stooped
posture, and walk in series of short, accelerating steps, such that the feet shuffle and scrape the
floor. The arms remain immobile and do not swing during walking. The underlying pathology
involves the degeneration of neurons in the substantia nigra, thereby leading to decreased
dopaminergic activity and increased cholinergic activity. The diagnosis is made clinically. The
treatment includes dopaminergic and/or anticholinergic drugs.
(Choice A} Cerebellar ataxia is usually ipsilateral (i.e., the patient tends to fall towards the side of
the lesion}. Other features of cerebellar disorders include nystagmus, hypotonia, dysarthria, loss
of coordination, and the inability to perform rapid, alternating movements.
(Choice C) Muscular dystrophy results in a waddling gait owing to weakness of the gluteal
muscles.
(Choice D) Spastic gait is seen with lesions of the upper motor neuron (e.g., spinal cord injury or
cerebral palsy}. The movements of the affected extremities are slow, stiff, and effortful.
(Choice E) Gait disequilibrium results from disorders of the frontal lobe and multiple sensory
systems.
(Choice F) Sensory ataxia is seen with lesions involving the peripheral nerves, dorsal roots or
posterior columns. Loss of proprioception results in a wide-based, high-stepping gait.
Romberg's sign may be positive (i.e., the patient sways on standing with his feet together and
eyes closed}.
(Choice G) Vestibular ataxia results in an en-bloc gait, with minimal movements of the head
during walking. It is typically a staggering gait, and is accompanied by vertigo and nystagmus.
(Choice H) The hypokinetic gait in patients with Parkinsonism is typically narrow-based;
therefore, a broadbased gait, in the presence of other clinical features suggestive of
Parkinsonism, is ample reason to consider other syndromes such as multiple system atrophy,
spinocerebellar atrophies, and multi-infarct gait disorders. Wide-based gait is also seen in
sensory ataxia, some cerebellar disorders and muscular dystrophies.
(Choice I} Dystonic gait is characterized by involuntary, sustained, twisting movements of the
limbs and trunk.
Educational Objective:
Parkinsonism is caused by overactivity of cholinergic neurons and underactivity of dopaminergic
neurons in the substantia nigra. A shuffling gait (i.e., the patient appears as if he was chasing his
center of gravity} is characteristic of the disease.
The correct answer is: . Hypokinetic gait

40. A 37 year-old man with chronic schizophrenia is brought to see a new psychiatrist for
treatment. While taking the history, the psychiatrist finds that the patient functions with a
flat affect and circumstantial speech all the time. He has few friends. He is able to hold a
menial job at the halfway house where he lives, and his behavior is not influenced by
delusions or hallucinations currently. What should the psychiatrist rate the patient on
Axis V (global assessment of functioning)?
Select one:
o . 70
o . 30
o . 55
o . >95
o . 15

(Kaplan, p 310.) A global assessment of functioning between 51 and 60 denotes the presence of
a moderate amount of symptoms. One can see mild symptomatology (eg, circumstantiality or
occasional panic attacks) or this rating denotes moderate problems with social functioning
(problems working, few friends, etc).
The correct answer is: . 55
41. A 50-year-old man presents to your office complaining of pain and swelling of the right
knee. He bumped his right knee into a pole 2 days ago while working. He had one
episode of similar pain two years ago that resolved with over-the-counter analgesics. He
denies any illicit drug use. He has no other medical problems and does not take any
medications. His temperature is 37.2°C (98.9°F), and blood pressure is 126/76 mmHg.
Examination reveals swelling, warmth and decreased range of motion of the right knee.
All other joints are within normal limits. Synovial fluid analysis shows positive birefringent
crystals and negative Gram stain. These crystals are most likely composed of?
Select one:
o . Calcium pyrophosphate
o . Hydroxyapatite
o . Calcium oxalate
o . Monosodium urate
o . Ammonium phosphate

This patient's history and synovial fluid analysis are suggestive of pseudogout, the acute form of
calcium pyrophosphate dihydrate (CPPD) crystal disease. Attacks of pseudogout often occur in
the setting of trauma, surgery, or medical illness. This patient's knee trauma was the likely
precipitant of his condition. Patients with pseudogout present with acute pain, swelling, redness,
and limited motion of the involved joint( s), with the knee being most commonly affected (>50%
of cases). Fever, leukocytosis with a left shift, and chondrocalcinosis (calcified articular cartilage)
may also occur in pseudogout. While this patient's history is consistent with pseudogout, gout
and septic arthritis may present similarly. Therefore, the diagnosis cannot be made based on
history and examination alone. Synovial fluid analysis is critical for distinguishing between these
conditions. The identification of rhomboid, positively birefringent crystals on synovial fluid
analysis is diagnostic of pseudogout.
(Choice A) Hydroxyapatite is the complexed form of calcium found in teeth and bones. However,
it can also exist in the cartilage of patients with degenerative osteoarthritis and may be isolated
from synovial fluid of such patients.
(Choice B) Monosodium urate crystal deposition is seen in gout. These crystals are needle-
shaped and negatively birefringent on synovial fluid analysis.
(Choice D) Calcium oxalate is the most common crystal found in renal calculi. Calcium oxalate
crystals are not isolated from synovial fluid.
(Choice E) Struvite (magnesium ammonium phosphate) may be found in the renal calculi of
patients with urinary tract infection caused by urease-producing organisms (Klebsiella, Proteus).
Educational objective:
Pseudogout cannot be reliably distinguished from gout and septic arthritis based on history and
physical alone. It is diagnosed by the presence of rhomboid, positively birefringent crystals on
synovial fluid analysis, and radiographic evidence of chondrocalcinosis.
The correct answer is: . Calcium pyrophosphate
42. A 72-year-old male comes to the office with intermittent symptoms of dyspnea on
exertion, palpitations, and cough occasionally productive of blood. On cardiac
auscultation, a low-pitched diastolic rumbling murmur is faintly heard at the apex. What
is the most likely cause of the murmur?
Select one:
o . A congenital anomaly
o . Anemia from chronic blood loss
o . A silent MI within the past year
o . Long-standing hypertension
o . Rheumatic fever as a youth

(Fauci, pp1465-1467.) The history and physical examination findings suggest mitral stenosis.
Dyspnea may be present secondary to pulmonary edema; palpitations are often related to atrial
arrhythmias (PACs, SVT, atrial flutter or fibrillation); hemoptysis may occur as a consequence of
pulmonary hypertension with rupture of bronchial veins. A diastolic rumbling apical murmur is
characteristic. If the patient is in sinus rhythm, a late diastolic accentuation of the murmur occurs
because of increased flow across the mitral valve with atrial contraction. A loud first heart sound
and early diastolic opening snap may also be present. The etiology of mitral stenosis is usually
rheumatic, rarely congenital. Hypertension may cause an S4 gallop but not a diastolic murmur.
Myocardial infarction may cause mitral regurgitation because of papillary muscle dysfunction
and anemia may cause a pulmonic flow murmur; both of these are systolic murmurs.
The correct answer is: . Rheumatic fever as a youth

43. A 54-year-old woman presents to your office complaining of difficulty walking. She
describes severe weakness and occasional pain in her thigh muscles. She has stumbled
and fallen several times over the last week. Her past medical history is significant for
hypertension treated with hydrochlorothiazide and metoprolol. She consumes two to
three cans of beer on weekends. Her younger brother died of a neurological disease
when he was 20 years old. Her mother suffers from hypertension and diabetes mellitus.
Her heart rate is 90/min and blood pressure is 170/100 mmHg. Chest examination is
within normal limits. A bruit is heard over the left carotid artery. Neurologic examination
reveals hyporeflexia and decreased strength in all muscle groups. Her ESR is 12 mm/hr.
ECG shows flat and broad T waves with occasional premature ventricular contractions.
Which of the following is the most likely cause of this patient's current complaints?
Select one:
o . Subdural hematoma
o . Lumbar spinal stenosis
o . Ischemic stroke
o . Electrolyte disturbance
o . Epidural hematoma

fibrillation, torsades de pointes, and ventricular fibrillation can occur. This woman is taking a
potassium-wasting diuretic, which is the most likely cause of her hypokalemia. Other common
causes include diarrhea, vomiting, anorexia, and hyperaldosteronism. This patient's symptoms
should resolve with potassium supplementation.
(Choice A} Stroke does not cause muscle pain and usually has focal findings. The presentation
for an ischemic stoke will depend on the location of the lesion.
(Choices 8 and C) The hallmarks of an epidural hematoma are headache, confusion,
somnolence , and focal neural deficits that develop within hours of a trauma. Subdural
hematomas present similarly to epidural hematomas but the onset is typically more delayed.
(Choice D) Lumbar spinal stenosis presents with back pain radiating to the buttocks and thighs
that interferes with walking and lumbar extension. Lumbar flexion will often improve the
symptoms.
(Choice E) Multiple sclerosis (MS} is a common demyelinating disease. Symptoms are separated
by time and space (i.e., episodic} and most commonly include numbness, weakness, spastic
paraparesis, paresthesias, gait abnormalities, and vision difficulties.
(Choice F) Amyotrophic lateral sclerosis (ALS} is a degenerative motor neuron disease that
affects both upper and lower motor neurons. Sensation is preserved and cognition is rarely
affected. Symptoms include weakness and difficulty chewing, swallowing, coughing, and
breathing. Patients develop hyperreflexia, spasticity, and fasciculations. It is usually fatal within 5
years.
(Choice G) The patient in this vignette does not have other symptoms of hyperthyroidism.
(Choice H) Inflammatory myopathies often present with muscle weakness and may have
associated muscle pain or a skin rash. ESR and creatine kinase will be elevated.
(Choice I} Neuromuscular junction disorders include myasthenia gravis and the Lambert-Eaton
myasthenic syndrome. These both have insidious onset and present with proximal muscle
weakness. Ptosis and diplopia may be present as well.
Educational Objective:
Hypokalemia is a common electrolyte abnormality that causes weakness, fatigue, and muscle
cramps. When severe, it can lead to paralysis and arrhythmia. The ECG may show U waves, flat
and broad T waves, and premature ventricular beats.
The correct answer is: . Electrolyte disturbance

44. A 27-year-old man presents to the emergency department with unremitting nose
bleeding. He reports having a similar bleeding episode one year ago that was stopped in
the ER. He works as a computer programmer and has a sedentary lifestyle. He drinks
alcohol on social occasions but does not smoke or use illicit substances. On physical
examination, there are several ruby-colored papules on his lips that blanch partially with
pressure. Digital clubbing is also present. His abdomen is soft and non-tender. The liver
span is 8 cm and the spleen is not palpable. Laboratory findings are: Hematocrit 60%,
WBC count 8,000/mm3, Platelets 180,000/mm3. Which of the following is most likely
responsible for this patient's increased hematocrit?
Select one:
o . Pulmonary hypertension
o . Polycythemia vera
o . Arteriovenous shunting
o . Plasma volume loss
o . Carboxyhemoglobinemia

This patient most likely has hereditary telangiectasia (Osler-Weber-Rendu syndrome), an


autosomal dominant disorder characterized by diffuse telangiectasias, recurrent epistaxis, and
widespread AV malformations (AVMs). In hereditary telangiectasia, AVMs tend to occur in the
mucous membranes, skin, and gastrointestinal tract, but may also be present in the liver, brain,
and lung. AVMs in the lungs can shunt blood from the right to the left side of the heart, causing
chronic hypoxemia and a reactive polycythemia. Pulmonary AVMs can also present as massive,
sometimes fatal, hemoptysis.
(Choice A} Polycythemia vera is a myeloproliferative disorder that causes increased production
of all threeblood cell lines.
(Choice B) Plasma volume loss can cause spurious polycythemia by concentrating red blood cells
within the vasculature. The most common causes of decreased plasma volume are diarrhea,
vomiting, and poor oral intake.
(Choice C) Pulmonary hypertension and polycythemia commonly co-exist. First, polycythemia
can cause pulmonary hypertension by increasing blood viscosity within the pulmonary
vasculature. Additionally, in patients with chronic hypoxemia, both pulmonary hypertension and
polycythemia can result. However, AVMs and pulmonary hypertension are generally not related.
(Choice E) Carbon monoxide (CO) poisoning decreases the amount of oxygen that can be
carried in the blood, and the body compensates by developing a secondary polycythemia. Based
on this particular patient' s history and presentation, hereditary telangiectasia is a more likely
diagnosis.
(Choice F) Pulmonary conditions that cause hypercarbia often also cause polycythemia, but it is
not the hypercarbia itself that causes the increased red blood cell production.
Educational objective:
Patients with hereditary telangiectasia (Osler-Weber-Rendu syndrome) can develop pulmonary
AVMs associated with hemoptysis and right-to-left shunt physiology. This diagnosis must be
entertained in this patient with recurrent nose bleeds and oral lesions.
The correct answer is: . Arteriovenous shunting

45. A 70-year-old Caucasian man comes to the emergency department because of the
sudden onset of nausea, vomiting, diaphoresis, and chest pain. His other medical
problems include hypertension, diabetes mellitus-type 2, and aortic stenosis. He has
smoked one-and-a-half packs of cigarettes daily for 30 years and drinks 4 ounces of
alcohol daily. His temperature is 37.2°C (99°F), blood pressure is 100/60 mmHg, pulse is
60/min, and respirations are 18/min. The patient's pulse oximetry showed 98% at room
air. Examination shows normal first and second he sounds. Lungs are clear to
auscultation. His EKG is shown below. Which of the following is the most likely
mechanism of this patient's condition?

Select one:
o . Vasospasm of the left circumflex artery
o . Occlusion of the right coronary artery
o . Occlusion of the left anterior descending artery
o . Occlusion of the left circumflex artery
o . Inflammation of the pericardium

This patient has clear evidence of ST elevation in the inferior leads (II, Ill, and aVF) suggesting
acute inferior wall myocardial infarction. Inferior wall myocardial infarction results from the
occlusion of either the right coronary artery or the left circumflex artery. In most studies with
acute inferior myocardial infarction, the culprit lesion was in the right coronary artery than in the
left circumflex artery with a ratio of 4-5:1. Also, remember that in patients with inferior
myocardial infarction who have right ventricular infarction, the culprit artery virtually always is
the right coronary artery. This patient has bradycardia and hypotension suggesting involvement
of SA node and right ventricle; so, in this case it is most likley a RCA occlusion.
(Choice C) Left anterior descending artery occlusion causes anterior wall myocardial infarction.
(Choice D) Pericarditis will have diffuse ST elevation in all leads.
Educational Objective:
The most likely culprit lesion for acute inferior wall myocardial infarction is right coronary artery
occlusion, especially if it is complicated by right ventricular infarction (hypotension) and
bradycardia.
The correct answer is: . Occlusion of the right coronary artery
46. A 32-year-old homosexual male with HIV infection presented to his physician with skin
lesions. He first noted these lesions last month, and has since observed a change in color
from pink to violet. There is no associated pain, itching, or burning. He was diagnosed
with HIV infection three years ago and has been noncompliant with his medications. His
pulse is 80/min, blood pressure is 115/70 mm Hg, respirations are 14/min, and
temperature is 37.1° C (98.8°F). The appearance of his lesions is illustrated below. His
CD4 count is 30/microl, and viral load is 300,000copies/ml. Which of the following is the
most likely cause of his current condition?

Select one:
o . Human herpesvirus 8 (HHV-8)
o . Human papillomavirus
o . Pneumocystis jiroveci
o . Poxvirus
o . Herpes simplex type 2 (HSV-2)

The cutaneous lesions of Kaposi sarcoma are asymptomatic, elliptical, and arranged linearly.
Commonly involved regions include the legs, face, oral cavity, and genitalia. The lesions begin as
papules, and later develop into plaques or nodules. The color typically changes from light brown
to violet. There is no associated necrosis of the skin or underlying structures. In the US, this
disease is most commonly seen in homosexual HIV patients. Kaposi sarcoma in HIV patients is
caused by human herpesvirus 8.
(Choice C) Pneumocystis jiroveci is the preferred name for the bacteria previously known as
Pneumocystis carinii. This organism is a common cause of pneumonia in AIDS patients with CD4
counts lower than 200/microl. Although extrapulmonary disease may present as papules and
nodules in the ear canal, such disease is not very common.
(Choice D) Molluscum contagiosum is caused by poxvirus, and presents as asymptomatic,
centrallyumbilicated papules on the face, neck, and anogenital areas.
(Choices B, E, and F) Herpes simplex lesions are vesicular and painful. Cryptococcus is an
important cause of meningitis in HIV patients. Human papillomavirus causes common and
genital warts, as well as cervical cancer.
Educational Objective:
Recognize the cutaneous lesions of Kaposi sarcoma. Kaposi sarcoma in HIV patients is caused by
human herpesvirus 8.
The correct answer is: . Human herpesvirus 8 (HHV-8)

47. A 46-year-old male presents with swelling of his face that is especially prominent in the
periorbital area. He also complains of bilateral ankle swelling. He denies shortness of
breath, fever and discoloration of urine. He is a non-smoker and non-alcoholic. His past
medical history is not significant. He is currently not taking any medication. His pulse is
78/min, blood pressure is 130/70mmHg, respirations are 14/min and temperature is
37.1°C (99.0°F). Examination shows bilateral pitting ankle edema. Auscultation reveals
clear lungs, normal heart sounds, and no murmurs. Dipstick urinalysis is positive for
protein. 24-hour urine collection shows proteinuria of 4.6 g/day. Lab studies show: Total
serum calcium 7.5 mg/dL, Albumin 2.2 g/dL, Phosphorus 3.5 mg/dL, Magnesium 2.2
mg/dL, Creatinine 0.8 mg/dL. Which of the following is the most likely cause of his low
serum calcium level?
Select one:
o . Increase 25-hydroxylation of vitamin D
o . Decreased serum albumin
o . Decreased levels of parathyroid hormone
o . Decreased 25-hydroxylation of vitamin D
o . Decreased 1-alpha-hydroxlation of 25-OH vitamin D

Albumin plays an important role in the homeostasis of the three forms of plasma calcium, which
are: ionized calcium (45%), albumin-bound calcium (40%), and calcium bound to inorganic and
organic anions (15%). Patients with hypoalbuminemia can have a low level of total plasma
calcium; however, they may not manifest with clinical hypocalcemia because their levels of
ionized calcium (the physiologically active form) have remained normal. It is therefore very
important to calculate the corrected serum calcium level, which helps determine the clinical
significance of the low total plasma calcium level.
The formula for corrected calcium level is:
Corrected calcium in mg/dl = measured total calcium+ 0.8 (4.0 g/dl - measured serum albumin
in g/dl ).
Another method is to roughly estimate the corrected calcium level:
With every 1 g/dl change in serum albumin level from 4 g/dl , there is a resultant change in total
plasma calcium level by 0.8 mg/dl . For instance, if the serum albumin level decreases from 4.0
g/dl to 3.0 gm/dl , the serum total calcium will decrease by 0.8 mg/dl .
In this patient, the corrected total plasma calcium level is 8.94 mg/L; therefore, his low total
serum calcium level is most likely due to hypoalbuminemia, and is not clinically significant.
(Choices A and B) Vitamin D is either formed in the skin or absorbed from the intestine. A series
of hydroxylation steps leads to the formation of 1 , 25 ( OH) 2 D, the physiologically active form
of vitamin D. The first step (25-hydroxylation) occurs in the liver; this produces 25 ( OH) D, which
is physiologically inert. The second step ( 1-alpha-hydroxylation) occurs in the kidney; this
produces 1 , 25 ( OH)2 D, which actively plays a role in the absorption of calcium (and
phosphorus) from the small intestine.
In patients with chronic renal failure, the second step is impaired, thereby resulting in
hypocalcemia secondary to decreased intestinal absorption. This patient has normal creatinine
levels, making it unlikely that his low calcium level was caused by impaired 1-alpha-
hydroxylation. Hypoalbuminemia thus remains as the most likely cause of this patient's calcium
level.
(Choice C) The most important causes of hypoparathyroidism are surgical and autoimmune.
Hypoparathyroidism causes hypocalcemia, but it also causes elevated levels of serum
phosphorus. This patient's serum phosphorus level is normal, making hypoparathyroidism an
unlikely diagnosis.
Educational objective:
The serum albumin level should always be measured simultaneously with the serum calcium
level in order to calculate the correct total serum calcium value. With every 1 g/dl change in
serum albumin, serum calcium changes by 0.8 mg/dl .
The correct answer is: . Decreased serum albumin

48. A 32-year-old Caucasian male presents to the ER with a 12-hour history of anorexia and
vomiting. He says that he feels “a little dizzy”. He denies abdominal pain or diarrhea. His
past medical history is insignificant His blood pressure is 110/70 mmHg while supine and
100/60 mmHg while sitting. His heart rate is 90/min. His laboratory values are: Serum
sodium 139 mEq/L, Serum potassium 3.1 mEq/L, Serum calcium 8.9 mEq/L, Serum
chloride 88 mEq/L, Serum bicarbonate 33 mEq/L, Blood glucose 95 mg/dl, BUN 20
mg/dl, Serum creatinine 1.1 mg/dl. Which of the following is the most likely cause of the
decreased chloride level in this patient?
Select one:
o . Metabolic alkalosis
o . Volume depletion
o . Intracellular shift
o . Gastrointestinal loss
o . Bicarbonate reabsorption in the kidney

The chemistry values obtained for this patient demonstrate hypochloremia, hypokalemia, and an
elevated bicarbonate level. These findings are all consistent with gastrointestinal losses due to
vomiting (Choice A}. Gastrointestinal fluid is rich in both hydrogen chloride and potassium
chloride. Therefore, any process by which excessive gastric contents are lost (vomiting,
nasogastric suctioning}, may result in loss of hydrogen, chloride, and potassium ions. The results
include hypochloremia and hypokalemia, as seen in this patient. Furthermore, the loss of
hydrogen ions increases the concentration of bicarbonate. An arterial blood gas would be
expected to confirm metabolic alkalosis in this patient by demonstrating a pH> 7.4 in addition
to the elevated bicarbonate.
(Choice B & E) For each hydrogen chloride ion lost due to vomiting, a bicarbonate ion is
produced. In addition, the volume contraction caused by vomiting leads to activation of the
renin-angiotensin system which stimulates bicarbonate reabsorption. It is important to realize
that while volume contraction and bicarbonate reabsorption are increased in this patient, they
are the result of hydrogen chloride loss; not the cause of it. These answer choices reverse cause
and effect.
(Choice C) This answer choice again reverses cause and effect. In this patient, vomiting has
caused significant loss of hydrogen chloride, resulting in metabolic alkalosis. Metabolic alkalosis
is the consequence- not the causeof vomiting.
(Choice D & F) Hypokalemia accompanies, but does not cause this patient's hypochloremia. It is
the result of gastrointestinal loss of potassium, the intracellular shift of potassium caused by
alkalosis, and increased renal excretion of potassium caused by aldosterone. The aldosterone is
ramped up by the renin-angiotensin system as a means to retain fluid.
Educational objective:
Gastric contents are rich in hydrogen, chloride, and potassium. Therefore vomiting causes
hypochloremic
metabolic alkalosis and hypokalemia. Bicarbonate levels rise as a result of hydrogen loss and
activation of
the renin-angiotensin system. The administration of isotonic sodium chloride and potassium is
used to
reverse these electrolyte abnormalities.
The correct answer is: . Gastrointestinal loss

49. A 34-year-old obese female is brought to the emergency department complaining of


severe right upper quadrant abdominal pain that began suddenly earlier this morning.
She is accompanied by her husband. An ultrasound evaluation is performed and the
woman is diagnosed with acute cholecystitis. Because her symptoms worsen with
conservative treatment, the decision is made to operate. The patient is discharged home
five days after her successful cholecystectomy. Upon discharge, the patient and her
husband request all medical records associated with her stay. What is the most
appropriate response on the behalf of the physician?
Select one:
o . Politely inform the couple that it is unlawful to provide patients with their medical
records
o . Inquire as to why the records are needed
o . Refuse to provide the records
o . Give them the medical chart in its entirety
o . Provide a copy of the original records
o Patients have the legal right to personally access their medical records. Although the
actual medical chart is the property of the physician or hospital, copies of the chart
should be provided to the patient on request.
o (Choice B) Third-party requests for a patient's medical records should be refused unless
the patient has provided written legal consent beforehand. However, there is no reason
to deny the patient access to her records.
o (Choice C) It is legal to provide patients with their medical records. To say otherwise is
incorrect.
o (Choice D) The actual medical chart is always the property of the physician or hospital
that created it. In addition, it is important to maintain complete records in the event of
later inquiries. Therefore, the patient should be given a copy of her chart and not the
original.
o (Choice E) Patients do not need to justify requests for medical records. Were the patient
to insist upon obtaining the original chart, however, the physician would be justified in
asking why such an unusual transaction is necessary.
o Educational Objective:
o Patients have the legal right to obtain copies of their medical records.
o The correct answer is: . Provide a copy of the original records

50. A 12-year-old African American male is found to have a murmur during a routine sports
physical. He has a family history of sudden death at a young age. It is a harsh crescendo-
decrescendo murmur that begins after S1 and is best heard at the left lower sternal
border. Valsalva maneuver intensifies the murmur. Which of the following is the most
likely mitral valve abnormality in this patient?
Select one:
o . Abnormal mitral leaflet motion
o . Rupture of chordae tendinae
o . Prolapse of the mitral valve
o . Mitral annulus calcifications
o . Dilated mitral valve annulus

A crescendo-decrescendo murmur at the left lower sternal border in a young individual is most
likely due to hypertrophic cardiomyopathy. The disease is more common in African-Americans.
The murmur classically increases during Valsalva maneuver due to decreased preload and
decreased filling of the left ventricle. This is in contrast to most other cardiac murmurs which
decrease in intensity with decreased preload. The outflow obstruction in hypertrophic
cardiomyopathy is due to both a hypertrophied interventricular septum and an abnormality in
the motion of the mitral valve leaflets referred to as systolic anterior motion (SAM). SAM results
in increased outflow obstruction, and in some patients may be of greater clinical consequence
than septal hypertrophy.
(Choice A) Dilation of the mitral valve annulus can occur with dilated cardiomyopathy or
ischemic cardiomyopathy and leads to mitral regurgitation. This murmur is typically heard as a
holosystolic murmur at the apex with radiation to the axilla.
(Choice B) Rupture of the chordae tendinea can occur secondary to mitral valve prolapse,
endocarditis, trauma, or myocardial infarction. The result is mitral regurgitation.
(Choice D) Mitral annulus calcification is most common in patients over age 40. It may result in
mitral regurgitation.
(Choice E) Prolapse of the mitral valve can occur due to myxomatous degeneration of the valve
and chordae tendinae. It is typically heard as a midsystolic click at the apex.
Educational objective:
Outflow obstruction in hypertrophic cardiomyopathy results from both septal hypertrophy and
systolic anterior motion (SAM) of the mitral valve.
The correct answer is: . Abnormal mitral leaflet motion

51. A 45-year-old female presents complaining of constipation and abdominal pain for the
past two weeks. She also complains of urinary frequency and constant thirst. Her past
medical history is significant for obesity. She tells you that she has been trying very hard
to lose weight, and over the past six months has even attempted various fad diets. She
assures you that she supplements her intake with numerous over-the-counter vitamins
and minerals. She has managed to lose 20 lbs during this time. Her medical history is
also significant for atrial fibrillation for the past 4 years, for which she takes diltiazem. On
physical examination, her temperature is 36.8°C (98.2°F), blood pressure is 120/70
mmHg, pulse is 90/min, and respirations are 13/min. Her mucous membranes are dry,
and her abdomen is soft and non-tender without rebound or rigidity. Bowel sounds are
present. Urinalysis is within normal limits. Which of the following is most likely
responsible for her current symptoms?
Select one:
o . Diabetic ketoacidosis
o . Adrenal insufficiency
o . Diltiazem
o . Vitamin A overdose
o . Vitamin D overdose

This patient's symptoms are most likely due to ingestion of toxic levels of vitamin D, a fat-
soluble vitamin obtained through diet and exposure to sunlight. Patients trying to lose weight
often use excessive amounts of vitamin and mineral supplements. Because vitamin D plays a role
in calcium absorption, overdose of this vitamin can lead to hypercalcemia. Thus manifestations
of vitamin D toxicity - including constipation, abdominal pain, polyuria and polydipsia - are
similar to symptoms of hypercalcemia.
Obese patients should be counseled regarding healthy approaches to weight loss, such as
behavior modification, exercise, and dietary therapy. Because of the potential for vitamin and
mineral toxicities, the National Institutes of Health discourage dietary supplementation except in
cases of known or suspected deficiency.
(Choice A} Diltiazem, a calcium channel blocker, can also cause constipation, but it does not
cause other symptoms of hypercalcemia like polyuria and polydypsia. Also, diltiazem is less likely
to be the culprit here as this patient has been taking diltiazem for 4 years, but has only had
symptoms over the past several weeks.
(Choice B) Diabetic ketoacidosis (DKA} is not an uncommon initial presentation for type 1
diabetes. Other findings in undiagnosed type 1 diabetes include polyuria, polydipsia, and weight
loss. However, urinalysis usually shows glucose and ketones, and constipation is not typical.
Furthermore, type 1 diabetes is usually diagnosed earlier in life. Type 2 diabetes, more common
in overweight, middle-aged individuals, is unlikely to cause DKA.
(Choice D) Adrenal insufficiency presents with weight loss, fatigue, muscle weakness, orthostatic
hypotension, and headache. Diarrhea, not constipation, is a common presenting symptom.
(Choice E) Vitamin A is a fat-soluble vitamin. Toxicity may cause dry skin, headache, abdominal
pain, and blurry vision. Vitamin A toxicity can also cause pseudotumor cerebri.
(Choice F) Symptoms of hypothyroidism include constipation, cold intolerance, fatigue, and
weight gain.
(Choice G) Malabsorption presents with diarrhea, weight loss, edema, and vitamin deficiencies.
Educational objective:
Fat-soluble vitamins are stored in the body for long periods oftime, and hence have the
potential to accumulate to toxic levels when supplements are taken. Vitamin D toxicity leads to
hypercalcemia, constipation, abdominal pain, weight loss, polyuria, and polydipsia. In cases of
toxicity, both vitamin D and calcium should be withheld until symptoms improve and levels
return to normal.
The correct answer is: . Vitamin D overdose

52. A 63-year-old woman with a long history of hypertension faints after experiencing the
sudden onset of severe chest pain that radiates to her back. She is rushed to the
emergency room. Upon arrival she is agitated and demands quick pain relief. Her heart
rate is 110/min and blood pressure is 90/50 mmHg. Jugular veins are distended. An
intra-arterial catheter shows significant variation of systolic blood pressure related to the
respiratory cycle. Chest x-ray reveals widening of the mediastinum. Which of the
following is the most likely cause of this patient's syncope?
Select one:
o . Papillary muscle dysfunction
o . Vagal hyperactivity
o . Intravascular volume loss
o . Pericardial fluid accumulation
o . Cardiac tachyarrhythmia
This patient suffered an aortic dissection complicated by cardiac tamponade, which led to
syncope. Aortic dissection typically occurs in patients with hypertension and presents as severe
chest pain radiating to the back. Chest x-ray may reveal a widened mediastinum.
One complication of aortic dissection is rupture into the pericardia! sac, leading to cardiac
tamponade. Cardiac tamponade occurs when enough blood accumulates in the pericardia!
space that the pressure exerted on the heart chambers begins to limit diastolic filling. The result
is decreased preload and decreased cardiac output. Syncope may result from decreased cranial
blood flood. Tamponade presents as hypotension, evidence of right heart failure (distended
jugular veins), and tachycardia. Pulsus paradoxus, or a disappearance of distal pulses on
inspiration, may be present.
(Choice A) Papillary muscle dysfunction is a well-described complication of acute myocardial
infarction in which a papillary muscle is no longer able to hold the corresponding valve leaflet in
place. It presents as acute mitral regurgitation with a holosystolic murmur and left-sided heart
failure. Papillary muscle dysfunction is not a consequence of aortic dissection.
(Choice B) Intravascular volume loss typically occurs from bleeding, poor oral intake, vomiting,
or diarrhea. Patients may present in shock with or without an obvious source of bleeding. More
commonly, however, patients present with orthostatic hypotension and syncope.
(Choice D) Vagal hyperactivity is the mechanism behind vasovagal syncope. This disorder is
characterized by nausea, diaphoresis, tachycardia, and pallor prior to syncope.
(Choice E) Cardiac tachyarrhythmias, both atrial (e.g., atrial fibrillation or flutter) and ventricular
(e.g., ventricular tachycardia), may cause syncope. However, they typically present without
warning, not preceded by chest pain. They also do not cause pulsus paradoxus or a widened
mediastinum on chest x-ray.
(Choice F) Aortic stenosis can cause syncope but is not likely to be the cause in this case for
several reasons. In aortic stenosis, syncope usually occurs with activity. Patients frequently have
a preceding history of exertional symptoms (dyspnea, chest pain, and/or fatigue). The chest pain
seen with aortic stenosis is not as severe as with aortic dissection and the pain does not typically
radiate to the back. Also, aortic stenosis does not cause pulsus paradoxus or a widened
mediastinum on chest x-ray.
Educational Objective:
Cardiac tamponade is one of the most deadly consequences of aortic dissection. It is
characterized by hypotension, tachycardia, and heart failure.
The correct answer is: . Pericardial fluid accumulation

53. A 56-year-old male presents with progressively worsening dyspnea over a 4 month
period. He denies fever, chest pain, cough or ankle swelling and does not use tobacco,
alcohol or drugs. He works for a home insulation and plumbing company. He has never
been abroad and does not own any pets. His only medications are hydrochlorothiazide
and metoprolol for blood pressure control. On physical examination, his temperature is
36.8°C (98.2°F), pulse is 76/min, blood pressure is 130/78 mmHg, and respirations are
15/min. Examination shows digital clubbing and fine bibasilar end-inspiratory crackles.
Jugular venous pressure is 7 cm and there is no peripheral edema. Which of the
following additional findings is most likely in this patient?
Select one:
o . Increased residual lung volume
o . Reduced FEV1/FVC ratio
o . Increased pulmonary capillary wedge pressure
o . Decreased diffusion lung capacity (DLCO)
o . Decreased pulmonary arterial pressure

Individuals with an occupational history involving mining, shipbuilding, insulation, or pipe work
are at risk for asbestosis, a type of pneumoconiosis. Typically, there is a latency period of >20
years between the initial exposure to asbestos and disease presentation. Progressive dyspnea
without cough over months, plus chest pain and fever, is typical for asbestosis. Digital clubbing
and bibasilar end-inspiratory crackles are common findings on physical examination, each seen
in approximately 50% of affected patients. Typical chest x-ray findings include interstitial
abnormalities of the lower lung fields and pleural plaques. As in other forms of interstitial lung
disease, pulmonary function tests reveal a restrictive pattern (decreased lung volumes with
normal or elevated FEV1/FVC) accompanied by reductions in diffusion lung capacity (DLCO) and
pulmonary compliance (Choice B).
(Choice A) Pulmonary capillary wedge pressure (PCVVP) is an indicator of the left atrial pressure.
An elevated PCVVP in the setting of dyspnea confirms a cardiac origin for the patient's
symptoms. However, this particular patient's normal jugular venous pressure, lack of edema and
absence of chest pain make a cardiac cause unlikely.
(Choice C) Interstitial lung disease is one of many potential causes of pulmonary hypertension.
In asbestosis, scarred lung parenchyma may impede blood flow within the lungs, thereby
causing elevated pulmonary arterial pressure.
(Choice D) Residual lung volume is the amount of air remaining in the lung after maximal
exhalation. It is increased in obstructive lung diseases like asthma, COPD, and bronchiectasis. In
restrictive lung diseases like asbestosis, the residual lung volume is normal or decreased.
(Choice E) The FEV1/FVC ratio is reduced to <80% of normal in obstructive lung diseases. In
restrictive lung diseases, the values of FEV1 and FVC may also be reduced, but the ratio is >80%.
Educational objective:
Asbestosis is a form of pneumoconiosis resulting from inhalation of particles involved in many
industrial processes. Progressive dyspnea, clubbing and end-inspiratory crackles are typical signs
and symptoms. Pulmonary function tests reveal a restrictive lung disease pattern, with decreased
lung volumes, decreased DLCO, and a normal FEV1/FVC.
The correct answer is: . Decreased diffusion lung capacity (DLCO)

54. A 17-year-old man comes to the emergency department and complains of intensive left
flank pain that radiates to the groin. He refers to his symptom as "stone passage," which
he has experienced "for so many times since childhood." His uncle has the same
problem. Urinalysis shows hexagonal crystals. The urinary cyanide nitroprusside test is
positive. Which of the following is the most likely cause of this patient's condition?
Select one:
o . Excessive intestinal reabsorption of oxalate
o . Infection
o . Parathyroid adenoma
o . Amino acid transport abnormality
o . Abnormality of uric acid metabolism

The most likely diagnosis is cystinuria, which is actually a group of disorders characterized by
impaired amino acid transport and several modes of inheritance. The problem lies with the
defective transport of dibasic amino acids (cystine, lysine, arginine, and ornithine) by the brush
borders of renal tubular and intestinal epithelial cells. Cystine is poorly soluble in water; this
leads to the formation of hard, radioopaque renal stones.
In this case, the clues to the correct diagnosis are the patient's personal history of recurrent
stones since childhood, positive family history, typical hexagonal crystals on urinalysis, and
positive urinary cyanide nitroprusside test. The urinary cyanide nitroprusside test can detect
elevated cystine levels, which can help confirm the diagnosis; this test is widely used as a
qualitative screening procedure, and is particularly helpful for the detection of homozygotes.
(Choice B) Primary hyperparathyroidism due to parathyroid adenoma can lead to recurrent
stones, but this patient's clinical scenario is more typical for cystinuria.
(Choices C and D) Excessive uric acid excretion in the urine due to hyperuricemia and excessive
intestinal reabsorption of oxalate due to intestinal diseases are other causes of nephrolithiasis,
but are unlikely in this case.
(Choice E) Infection with urease-producing microorganisms can result in struvite stone
formation, but this is also less likely for this patient.
Educational Objective:
Cystinuria is an inherited disease causing recurrent renal stone formation. Look for a personal
history of recurrent kidney stones from childhood and a positive family history. The
characteristic stones are hard and radioopaque. Urinalysis shows typical hexagonal crystals. The
urinary cyanide nitroprusside test is widely used as a qualitative screening procedure.
The correct answer is: . Amino acid transport abnormality

55. A 91-year-old woman presents to the emergency department with a chief complaint of
shortness of breath over the past 2 days. She has a history of hypertension and coronary
artery bypass surgery 25 years earlier. Her blood pressure is 178/92 mmHg and she has
jugular venous distension, hepatomegaly, and 3+ lower extremity edema. ECG is
remarkable for left ventricular hypertrophy, no ST-segment elevations or depressions, no
Q waves, and no T wave abnormalities. Echocardiogram reveals an ejection fraction of
60% and left atrial dilatation. There is universal left ventricular thickening. No valvular
regurgitation or stenosis was noted. Which of the following underlying conditions is the
most likely cause of this patient’s symptoms?
Select one:
o Mitral valve prolapse
o Myocarditis
o Hypertrophic obstructive cardiomyopathy
o Hypertensive heart disease
o Ischemic heart disease

You should be able to recognize diastolic dysfunction as a cause of heart failure. In diastolic
heart failure, LVEF is normal (>50%). Heart failure results from an inability of the LV to fill during
diastole rather than an inability of the LV to eject blood into systemic fl ow. Hypertensive heart
disease is one of the most common causes of diastolic heart failure. This patient’s medical
history of hypertension, in-office measurement indicating high blood pressure, and the ECG
showing LVH is consistent with this diagnosis.
Answer B is incorrect. Hypertrophic obstructive cardiomyopathy (HOCM) may manifest with
hypertension and an ECG showing LVH. However, echocardiography of patients with HOCM
typically shows asymmetric septal wall thickening (septum:ventricular wall thickness ratio >1.3).
Subaortic stenosis oftentimes results from septal wall thickening, causing obstruction across the
aortic outflow tract. HOCM is often familial and is manifest earlier in life (childhood through
young adulthood). Patients rarely present in the ninth decade of life.
Answer C is incorrect. Ischemic heart disease is one of the most common causes of diastolic
dysfunction and must always be included on the differential. However, an ECG showing lack of
ST elevation (indicating acute infarct) or depression (indicating ischemia), Q waves, or T-wave
inversions (indicating previous infarct) makes this diagnosis less likely.
Answer D is incorrect. This patient’s normal LVEF is more consistent with diastolic heart failure.
MVP results in systolic heart failure instead of diastolic heart failure. Increased regurgitation over
the mitral valve results in an increased preload and inability of the ventricle to eject this
increased volume (decreased LVEF). Furthermore, no regurgitation was noted on Doppler fl ow
on echocardiography, making this answer choice unlikely.
Answer E is incorrect. Myocarditis can be associated with diastolic heart failure, but is less likely
in this case given no recent viral illness. This patient’s age, past medical history, and ECG are all
consistent with hypertensive heart disease.
The correct answer is: Hypertensive heart disease

56. A 29-year-old white female presents to the emergency department with nausea,
vomiting, severe generalized abdominal pain, and hypotension. She is subsequently
admitted to the intensive care unit. Her past medical history is significant for
hypothyroidism secondary to Hashimoto's thyroiditis, for which she has been taking
levothyroxine. She denies smoking cigarettes, drinking alcohol, and using any
intravenous drugs. Her mother also has hypothyroidism. Her blood pressure is 70/50
mmHg, heart rate is 110/min, temperature is 98.4°F (37.0°C) and respiratory rate is
24/min. Physical examination reveals dry and pigmented mucous membranes. The
skin creases also show increased pigmentation. Lab studies show: Serum chemistry:
Serum Na 130 mEq/L, Serum K 6.1 mEq/L, Chloride 96 mEq/L, Bicarbonate 18 mEq/L,
BUN 33 mg/dL, Serum creatinine 1.3 mg/dL, Blood glucose 56 mg/dL. CBC:
Hemoglobin 10.8 g/L, Platelets 300,000/mm3, Leukocyte count 6,500/mm3,
Neutrophils 70%, Eosinophils 10%, Lymphocytes 20%. The random serum cortisol
level is 3.2 mcg/dL (normal=5 to 25 mcg/dL), and ACTH level is 142 pg/mL (normal=
9 to 52 pg/mL). What is the most likely involved pathophysiologic mechanism of this
patient's disorder?
Select one:
 . Congenital
 . Autoimmune
 . Infective
 . Hemorrhagic
 . Infiltrative

The patient in this vignette has features consistent with primary adrenal insufficiency (e.g.,
hypotension, pigmentation, hyponatremia, hyperkalemia, eosinophilia, markedly elevated
ACTH levels, and low serum cortisol levels despite hypotension and stress). The most
common cause of primary adrenal insufficiency in developed countries is autoimmune
adrenalitis (responsible for more than 80% of patients with primary adrenal cortical
insufficiency). The presence of autoantibodies against one or more adrenal steroidogenic
enzymes has been postulated to be a cause of autoimmune adrenal destruction. In addition,
approximately 50% of patients with autoimmune primary adrenal insufficiency (Addison's
disease) have autoimmune diseases involving other endocrine glands, such as the thyroid,
parathyroid, and ovaries. Other nonendocrine autoimmune conditions, such as pernicious
anemia and vitiligo, are also common.
(Choice B) A number of infections can cause primary adrenal insufficiency. These infections
include cytomegalovirus infection, fungal infection, and tuberculosis.
(Choice C) Malignant infiltration of the adrenal glands can occur, and is actually fairly
common in certain cancers (e.g., small cell lung cancer); however, patients typically do not
have primary adrenal insufficiency, unless more than 90% of the adrenal glands has been
destroyed.
(Choice D) Patients on Coumadin (warfarin) can have adrenal hemorrhage with acute stress
such as sepsis. This can occur even when the INR is in the therapeutic range. Stress increases
the ACTH levels, which in turn increases the adrenal blood flow, thus predisposing such
patients to adrenal hemorrhage. Hemorrhagic necrosis of the adrenal glands may also be
seen in patients with some systemic infections (e.g., children with meningococcemia or
Pseudomonas sepsis).
(Choice E) Adrenoleukodystrophy is the common cause of congenital problems which lead
to adrenal insufficiency. It is typically seen in young males, and its pathophysiology involves
the accumulation of very long chain fatty acids within the adrenal glands. Females can serve
as carriers; they do not show any clinical manifestations of adrenoleukodystrophy. Adrenal
insufficiency soon after birth is also seen in congenital adrenal hyperplasia due to classic 21
-hydroxylase deficiency. In this case, the patient is unlikely to have a congenital cause of
adrenal insufficiency because she became symptomatic in her late 20s. Autoimmune
adrenalitis remains as the most likely cause of her adrenal insufficiency.
Educational Objective:
In developed countries, more than 80% of patients have primary adrenal insufficiency due to
autoimmune adrenalitis. Patients with autoimmune adrenalitis have autoimmune
involvement of other endocrine glands, such as the thyroid, parathyroid, and ovaries.
The correct answer is: . Autoimmune

57. A 23-year-old Caucasian male with muscular weakness, vomiting and abdominal pain
is brought to the emergency department. He had a minor respiratory illness 2 days
ago. His past medical history is significant for diabetes mellitus, type 1. He admits
skipping his insulin shots yesterday and today because he had no appetite. His
temperature is 37.8°C (100°F), blood pressure is 110/70 mmHg, pulse is 110/min, and
respirations are 27/min. His oral mucosa is dry. The laboratory values are: Serum
sodium 132 mEq/L, Serum potassium 5.4 mEq/L, Serum calcium 8.9 mEq/L, Serum
chloride 96 mEq/L, Serum bicarbonate 12 mEq/L, Blood glucose 470 mg/dl,BUN 19
mg/dL, Serum creatinine 1.1 mg/dL. Which of the following is the most likely cause
of the increased potassium level in this patient?
Select one:
 . Extracellular shift
 . Intracellular potassium excess
 . Decreased gastrointestinal loss
 . Tissue destruction
 . Increased renal reabsorption of potassium

Metabolic acidosis during diabetic ketoacidosis (DKA) is typically accompanied by


hyperkalemia. This hyperkalemia is sometimes called paradoxical, because the body
potassium reserves are actually depleted due to increased gastrointestinal losses and
osmotic diuresis. The main causes of this paradoxical hyperkalemia are: 1) extracellular shift
of potassium in exchange to hydrogen ion, with resultant intracellular potassium deficit, and
2) impaired insulin-dependent cell entry of the potassium ion. Early potassium
supplementation is very important in the treatment of patients with DKA. Treatment with
insulin and intravenous fluids lead to a rapid decrease in serum potassium levels even if they
are initially elevated. All hyperkalemic patients should start receiving potassium once the
serum potassium level goes below 4.5 mEq/L. In patients with normal or low potassium
levels, potassium replacement should be started with initiation of intravenous fluid therapy.
(Choices A and B) Increased renal and gastrointestinal loss (not absorption) is seen in
patients with DKA.
(Choice C) Tissue destruction accounts for hyperkalemia observed in patients with excessive
tissue damage (e.g., crush syndrome).
(Choice E) Intracellular potassium is depleted in DKA due to the transcellular shift in
exchange for hydrogen ion and hyperosmolality.
(Choice F) Osmotic diuresis is one of the causes of depletion of total body potassium in
patients with DKA.
Educational Objective:
Metabolic acidosis observed during diabetic ketoacidosis is typically accompanied by
hyperkalemia; this is sometimes called paradoxical hyperkalemia because the body
potassium reserves are actually depleted.
The correct answer is: . Extracellular shift

58. A 23-year-old woman presents to the ED complaining of pain with urination. She has
no other complaints. Her symptoms started 3 week ago. During this time, she has
been to the clinic twice, with negative urine cultures each time. Her condition has not
improved with antibiotic therapy with sulfonamides or quinolones. Physical
examination is normal. Wet mount showed epithelial cells. Which of the following
organisms is most likely responsible for the patient’s symptoms?
Select one:
 . Staphylococcus aureus
 . Escherichia coli
 . Chlamydia trachomatis
 . Herpes simplex virus
 . Trichomonas vaginalis

(Tintinalli, pp 630-637.) Chlamydia trachomatis urethritis accounts for 5% to 20% of cases of


dysuria, and its presence may be suspected when urine cultures are sterile. If clinical
symptoms and urinalysis point to a UTI, urine cultures are sterile, and standard antibiotic
regimens effective against most urinary pathogens fail, consider Chlamydia urethritis.
(a) Although group A streptococcus is a possible pathogen, it would be very rare and would
be susceptible in most cases to the antibiotics that the patient has taken. (b) Initial episodes
of herpes simplex virus can result in symptoms that mimic true dysuria secondary to
sensitivity of lesions on the external genital surface or near the urethra. Generally, these
episodes are accompanied by fever, chills, and systemic symptoms, in addition to extremely
painful and tender lesions. Also, urinary frequency would not be common. (c) Although T
vaginalis would not respond to the previous antibiotic therapy, trichomoniasis presents with
a copious frothy vaginal discharge and would have been seen on wet mount preparation of
the vaginal secretions. (d) Escherichia coli accounts for 70% to 90% of community- acquired
UTIs in women. In a woman without recurrent UTIs, the antibiotics taken by this patient
would be appropriate. In a patient with recurrent UTIs, it is more likely that the E coli would
be resistant to standard antimicrobial regimens. This pathogen should have also been
grown on the standard culture medium.
The correct answer is: . Chlamydia trachomatis

59. A 25-year-old African American woman presents with a photo distributed skin rash
and arthralgias. She is found to have low-range proteinuria and abnormal urinary
sediment. Renal biopsy findings are consistent with focal proliferative
glomerulonephritis. Her complete blood count shows: Erythrocyte count 3.2
mln/mm3, Platelets 60,000/mm3, Leukocyte count 2,500/mm3. Which of the
following is the most likely cause of these hematologic findings?
Select one:
 . Peripheral destruction of blood cells
 . Ineffective hemopoiesis
 . Abnormal pooling of blood cells
 . Bone marrow hypoplasia
 . Dilutional pancytopenia

The patient described in this clinical vignette displays a number of symptoms characteristic
for systemic lupus erythematosus (SLE). This chronic autoimmune disorder affects mostly
women of childbearing age. Black women have a higher risk of developing SLE than
Caucasian and Asian women. Hematologic abnormalities are common in patients with SLE.
They occur due to formation of antibodies against blood cells and represent a form of type
II hypersensitivity reaction. Anemia in SLE is caused by autoimmune hemolysis, and
develops due to formation of warm lgG antibodies to RBCs. It is characterized by
spherocytosis, a positive direct Coombs test, and extravascular hemolysis. The pathogenesis
of SLEassociated thrombocytopenia is identical to that of ITP; antibodies against platelets
are formed causing destruction of platelets. Neutropenia due to antibody-mediated
destruction of WBCs also occurs, but this is less common.
(Choice A) Bone marrow hypoplasia or aplasia occurs due to exposure to environmental
toxins, certain medications ( antimetabolites, chloramphenicol), infectious agents
(parvovirus) or as an inherited condition (F anconi anemia).
(Choice B) Ineffective hematopoiesis refers to blood cell breakdown in bone marrow before
release into the circulation. Thalassemias and myelodysplastic syndrome are the examples of
such conditions.
(Choice C) Hypersplenism can result in abnormal pooling of blood cells in the spleen with
subsequent destruction of these cells. It manifests with pancytopenia and splenomegaly,
and may occur in cirrhosis, malaria, sickle cell disease and many other disorders.
(Choice E) Dilutional pancytopenia can occur after a massive packed red blood cell
transfusion or massive infusions of crystalloid solutions due to increased plasma volume out
of proportion to the number of blood cells.
Educational Objective:
Pancytopenia (decreased RBC, WBC and platelets) is common in patients with SLE. It occurs
due to the formation of autoantibodies against blood cells, a form of type II hypersensitivity
reaction.
The correct answer is: . Peripheral destruction of blood cells

60. A 80-year-old man with Type II diabetes and hypertension presents with increasing
dyspnea. He appears short of breath, blood pressure is 170/95 mmHg, pulse 100/min
and regular. The JVP is at 7 cm; there is a loud second heart sound and a systolic
ejection murmur at the right sternal border, which does not radiate. The lungs have
bibasilar crakles up to the scapula. The CXR has bilateral infiltrates and vascular
redistribution. His echocardiogram reports aortic sclerosis, concentric left ventricular
hypertrophy (LVH), and normal ejection fraction. Which of the following is the most
likely mechanism for this condition?

Select one:
 valvular heart disease
 diastolic dysfunction
 hypertrophic obstructive cardiomyopathy (HOCM)
 systolic dysfunction
 hibernating myocardium

Diastolic dysfunction is an important cause of heart failure in the elderly. It is commonly


associated with a history of hypertension and diabetes. Normal ejection fraction and aortic
sclerosis rule out either systolic or valvular heart disease as causes. In HOCM there is
nonconcentric hypertrophy. (Fuster, pp. 710, 2276–2277)
The correct answer is: diastolic dysfunction
61. A 68-year-old Caucasian man is admitted with a diagnosis of left lower lobe
pneumonia, and is started on gatifloxacin. He has a long history of diabetes,
hypothyroidism, hypercholesterolemia, and hypertension. He also has diabetic
retinopathy, peripheral neuropathy, and nephropathy. He has an arterio-venous
fistula placed for a possible dialysis. His medications are insulin, furosemide,
atorvastatin, metoprolol and levothyroxine. After having his blood drawn for some
laboratory studies today, he bleeds persistently. Laboratory studies show: Hb 11.5
g/dl, Platelets 160,000/cmm, Blood glucose 178 mg/dl, BUN 56 mg/dl, Serum
creatinine 3.5 mg/dl. His baseline creatinine level is between 3.2-3.5 mg/dl. Which of
the following is the most likely cause of his bleeding?
Select one:
 . Disseminated intravascular coagulation
 . Consumptive coagulopathy
 . Platelet dysfunction
 . Thrombocytopenia
 . Factor VIII deficiency

Abnormal hemostasis is a common manifestation seen in patients with chronic renal failure.
Abnormal bleeding and bruising are characteristic of uremic coagulopathy. Nowadays,
ecchymoses and epistaxis are the only major bleeding manifestations seen due to the
advent of dialysis; however, Gl bleeding, hemopericardium, subdural hematoma, and
bleeding from surgical or invasive sites can still occur due to uremic coagulopathy.
The pathogenesis is multifactorial, but the major defect involves platelet-vessel wall and
platelet-platelet interaction. Several uremic toxins have been implicated in the pathogenesis
of platelet dysfunction seen in chronic renal failure ( CRF), the chief among which is
guanidinosuccinic acid. Activated partial thromboplastin ( aPTT), prothrombin (PT), and
thrombin times (TT) are generally normal. Bleeding time (BT) is reflective of platelet function,
and is usually prolonged. The platelet count is normal, but there is platelet dysfunction that
causes bleeding.
A number of agents such as desmopressin (DDAVP), cryoprecipitate, and conjugated
estrogens have been used to correct the coagulopathy in uremic patients. DDAVP increases
the release of factor Vlll:von Willebrand factor multimers from endothelial storage sites.
(Choices A, C, B, and E) Disseminated intravascular coagulation, factor VIII deficiency,
consumptive coagulopathy, and thrombocytopenia are not common causes of bleeding in
uremic patients. Furthermore, this patient's normal platelet count makes the diagnosis of
DIC unlikely.
Educational Objective:
Platelet dysfunction is the most common cause of abnormal hemostasis in patients with
CRF. PT, PTT, and platelet count are normal. BT is prolonged. DDAVP is usually the treatment
of choice, if needed. DDAVP increases the release of factor Vlll:von Willebrand factor
multimers from endothelial storage sites. Platelet transfusion is not indicated because the
transfused platelets quickly become inactive.
The correct answer is: . Platelet dysfunction

62. A 10-year-old boy presents with fever, headache, photophobia, and neck discomfort
in the middle of summer. He is alert and oriented, but has neck pain with flexion and
extension of the head. His fundi are normal, and there are no focal neurologic
findings or skin changes. A lumbar puncture reveals normal protein and glucose with
a cell count of 240/mL (90% lymphocytes). Which of the following is the most likely
causative organism?
Select one:
 Neisseria meningitides
 Streptococcus pneumoniae
 HSV-1
 Listeria monocytogenes
 enterovirus (coxsackievirus or echovirus)

The cerebrospinal fluid (CSF) picture in this individual is consistent with viral meningitis
rather than a bacterial process. Enteroviruses are a prominent cause of viral meningitis in
the summer and fall months. They received their name because they multiply in the GI tract.
Fever, sometimes associated with respiratory symptoms, is the most common sequela of
enterovirus infection. There are about 70 enteroviruses that affect humans. These include
polioviruses, coxsackieviruses, echoviruses, and others. The spectrum of disease includes
paralytic disease, encephalitis, aseptic meningitis, pleurodynia, exanthems, pericarditis,
myocarditis, and nonspecific febrile illnesses. They can on occasion cause fulminant disease
in a newborn. The most important enteroviruses are the three poliovirus serotypes.
The correct answer is: enterovirus (coxsackievirus or echovirus)

63. A 38-year-old male with steroid-dependent sarcoidosis presents to the physician's


office because of progressive right hip pain. He localizes the pain to right groin and
states that the pain is present on weight bearing and at rest. His temperature is
37.2°C (98.9°F) and blood pressure is 156/86 mm Hg. Examination shows decreased
range of motion due to pain. He also has a round face and fullness in supraclavicular
area. Purple striae are present on skin. Muscle power is slightly decreased in the
proximal thigh muscles in both legs. Reflexes are 2+, and there are no sensory
deficits. Plain films of the right hip show no significant abnormalities ESR is 10
mm/hr. Which of the following is the most likely cause of his hip pain?
Select one:
 . Disruption of bone vasculature
 . Inflammatory arthritis
 . Cartilage degeneration
 . Aortoiliac occlusion
 . Inflammation of the trochanteric bursa

The patient described is most likely experiencing corticosteroid-related osteonecrocrosis of


his right femoral head. Osteonecrosis is also referred to as aseptic necrosis, avascular
necrosis, ischemic necrosis and osteochondritis dessicans. There are numerous factors that
are believed to play an etiologic role in this condition. This process, however, proceeds
through a common pathway regardless of the suspected etiology. In all cases, the
vasculature to the affected bone is disrupted causing bone and bone marrow infarction.
Subsequently, the bone is unable to remodel and trabecular thinning occurs ultimately
leading to collapse of the affected bone. This process may take months to years, on
occasion, to occur. Clinically, the patient experiences progressive pain during this time. In
the case of osteonecrosis of the femoral head, patients experience anterior hip pain
worsened by activity and relieved by rest with progressive limitation of range of motion.
Progression of disease will ultimately cause joint instability and pain at rest. In the first few
months of symptoms, x-rays will often fail to demonstrate an abnormality. MRI is the most
sensitive test for this condition. Precipitating factors other than chronic corticosteroid use
include chronic alcohol use, trauma and the antiphospholipid syndrome.
(Choice A} There are several forms of inflammatory arthritis including rheumatoid arthritis,
gout, pseudogout, psoriatic arthritis, ankylosing spondylitis, lupus and ochronosis among
others. Each has a unique presentation, but the case described is most consistent with
osteonecrosis related to chronic corticosteroid use.
(Choice B) Cartilage degeneration occurs in osteoarthritis. This condition typically affects
patients over 40 years of age, though such patients may also present with anterior hip pain
worsened by activity. This patient's clinical scenario is more suggestive of osteonecrosis due
to the history of chronic corticosteroid use.
(Choice D) Inflammation of the trochanteric bursa (trochanteric bursitis} is caused by friction
between the tendons of the gluteus medius and tensor fascia lata over the greater
trochanter of the femur. Pain is localized over the lateral hip and is worsened by palpation.
Pain caused by pressure on the lateral hip may interfere with sleeping in patients with this
condition.
(Choice E) Male patients with aortoiliac occlusion (Leriche syndrome} classically present with
low back, hip, buttock and thigh claudication accompanied by impotence and atrophy of
the lower extremities. Femoral pulses are typically weak and a bruit may be heard over the
iliac and femoral arteries.
Educational objective:
Chronic corticosteroid use and chronic excessive ingestion of alcohol account for over 90%
of cases of avascular necrosis of bone (osteonecrosis}. In the hip, patients present with
slowly progressive anterior hip pain with limitation of range of motion.
The correct answer is: . Disruption of bone vasculature
64. A 27-year-old female is brought into the emergency room by the local paramedics.
She was found unconscious at the scene of a house fire. On examination it does not
appear that she has suffered any burns. Black soot is noted near her nares and
mouth. Her respirations are slow but spontaneous. Her capillary refill time is 4
seconds. Supplemental oxygen by a non-rebreather mask is begun. Her arterial
blood gas and preliminary laboratory values are shown below: Blood pH 7.22, PaO2
100 mmHg, PaCO2 39 mmHg, HC03- 11 mEq/L, WBC count 9,000/cmm, Hb 14
mg/dl, Na+ 138 mEq/L, K+ 4 mEq/l, CI- 98 mEq/L, Troponin 0.4ng/ml. Which of the
following is the most likely primary cause of the patient's acid-base disturbance?
Select one:
 . Impaired excretion of lactic acid
 . Reduced oxygen utilization by tissues
 . Increased gut absorption
 . Increased metabolic rate
 . Decreased oxygen delivery to tissue

The patient described is the victim of a house fire and has most likely suffered a smoke
inhalation injury. Carbon monoxide (CO) poisoning is the most common systemic toxicity
seen in smoke inhalation. CO binds to hemoglobin with an affinity 260 times that of oxygen.
Upon inhalation, CO displaces oxygen from hemoglobin thereby decreasing the
hemoglobin binding sites available for oxygen and the oxygen content of the blood.
Because CO binds so avidly to hemoglobin, it also is able to change the shape of the
oxyhemoglobin dissociation curve. Specifically, CO causes a left shift of the curve and a
shape change from sigmoid to asymptotic. This change disallows unloading of oxygen from
hemoglobin in the tissues. By this twofold mechanism, CO poisoning causes decreased
oxygen delivery to the tissues. This results in increased anaerobic metabolism by the tissues
leading to increased lactic acid production and development of an anion gap metabolic
acidosis.
(Choice A) An increased metabolic rate may cause lactic acidosis if there is insufficient
oxygen present in the tissues and anaerobic metabolism ensues. This is commonly seen with
post seizure lactic acidosis.
(Choice C) Reduced oxygen utilization by tissues does occur in CO poisoning because of the
ability of CO to bind and inhibit cytochrome aa3, but CO binds to this enzyme with less
affinity than oxygen. This is not the primary mechanism for acidosis in this case.
(Choice D) Impaired excretion of lactic acid as well as other serum acids occurs in renal
failure resulting in an anion-gap metabolic acidosis.
(Choice E) Increased gut absorption of lactic acid may occur in cases of intestinal blind loop
syndrome where excessive production and absorption of 0-lactate causes a hypochloremic
metabolic acidosis.
Educational objective:
Lactic acidosis resulting from poor oxygen delivery to the tissues is known as type A lactic
acidosis. Causes include CO poisoning and circulatory failure (shock).
The correct answer is: . Decreased oxygen delivery to tissue

65. While palpating the pulse of a patient, you note that the pulse wave has two peaks.
You auscultate the heart and are certain that there is only one heartbeat for each two
pulse waves. Which of the following best describes this finding?
Select one:
 Pulsus bisferiens
 Pulsus alternans
 Pulsus bigeminus
 Pulsus parvus et tardus
 Dicrotic pulse

(Fauci, 14/e, p 1232.) Pulsus bisferiens (bisferious pulse) is seen in AI and in hypertrophic
cardiomyopathy (HCM). In the latter, the first wave or percussion wave is due to the rapid
flow rate of initial contraction, and the second wave or tidal wave is due to the slower rate
of continued contraction. The dicrotic pulse has two palpable pulses, but one is in systole
and the other is in diastole. Pulsus bigeminus is an alteration in pulse amplitude that follows
a ventricular premature beat. Pulsus alternans is a regular alternating pulse amplitude due
to alternating left ventricular contractile force; it is usually seen with severe left ventricular
decompensation and cardiac tamponade. Pulsus parvus et tardus (“small and slow rising”)
represents a delayed systolic peak due to obstruction to left ventricular ejection. It is seen in
aortic stenosis (AS).
The correct answer is: Pulsus bisferiens

66. A 54 year-old male comes to the physician because of abdominal distention. He


thinks his big belly is making it hard for him to breathe. He received radiation
therapy for Hodgkin lymphoma several years ago and was told that he is cured. He
drinks alcohol on a regular basis. His temperature is 36.7°C (98°F), blood pressure is
120/76 mm Hg, pulse is 80/min and respirations are 20/min. Examination shows a
jugular venous pressure 7 cm above the sternal angle. Dullness to percussion and
decreased breath sounds are noted in the right lung base. The abdomen is quite
distended with an obvious fluid wave. Mild hepatomegaly is present. Extremities
have 2+ lower extremity edema. Initial laboratory studies are shown below: Serum
creatinine 08 mg/dL, Alburnin 4.0 mg/dL, Total bihrubin 1.0 mg/dl, Prothrombin time
11 sec. Urinalysis is within normal limits. Which of the following is the most likely
cause of his condition?
Select one:
 . Thoracic duct obstruction
 . Urinary protein loss
 . Portal vein compression
 . Inelastic pericardium
 . Portal vein thrombosis

Constrictive pericarditis is a condition where the pericardium is thickened or scarred


resulting in restriction of diastolic filling. Because the heart is unable to fill properly during
diastole, cardiac output is compromised and venous pressures become chronically elevated
resulting in increased jugular venous pressures (JVP), severe ascites and hepatic congestion,
dyspnea and weakness. On examination, Kussmaul's sign, failure of JVP to decrease on
inspiration, is often present though not specific. The most common causes of constrictive
pericarditis are cardiac surgery, viral pericarditis and radiation therapy. Many cases are
idiopathic. The diagnosis is established by identifying a calcified pericardium on chest x-ray,
identifying a thickened pericardium on CT or MRI scanning or by measuring pressures
during cardiac catheteri zation. Treatment is with diuretics or pericardiectomy.
(Choice A) Urinary protein loss causes generalized edema, including pulmonary edema in
some cases, due to decreased oncotic pressure in capillary beds. However, his urinalysis
shows no protein.
(Choices B and C) Portal vein thrombosis or compression would cause ascites and peripheral
edema but would not cause increased JVP or hepatomegaly.
(Choice E) Thoracic duct obstruction would cause lymphedema, a firm, nonpitting edema
that does not cause the skin changes associated with venous stasis such as ulceration and
dermatitis.
(Choice F) Venous valve incompetence in the lower extremities causes dependent edema,
but ascites and increased JVP are not normally observed in the absence of concomitant
disease.
Educational objective:
Constrictive pericarditis causes hepatomegaly, ascites and increased JVP due to decreased
diastolic filling and impairment of cardiac output. Common causes include radiation
therapy, viral pericarditis and cardiac surgery.
The correct answer is: . Inelastic pericardium

67. A 62-year-old man has progressive symptoms of dyspnea, and more recently noticed
difficulty lying supine. Examination shows an elevated JVP at 8 cm, with a third heart
sound, pedal edema, and bibasilar crackles on auscultation. Which one of the
following may be implicated in fluid retention for this condition?
Select one:
 increased estrogen
 increased aldosterone
 decreased renin
 increased growth hormone
 decreased vasopressin

Retention of fluid is complex and not due to any one factor, however, hormones may
contribute. Growth hormone does not have fluidretaining properties. The exact mechanisms
that initiate renal conservation of salt and water are not precisely understood, but may
include arterial volume receptors sensing a decrease in the effective arterial blood volume.
Aldosterone, renin, and vasopressin are generally increased in heart failure. (Fuster, p. 713)
The correct answer is: increased aldosterone

68. A 50-year-old woman presents to your office complaining of lower extremity edema
that stated several weeks ago, and slowly progressed thereafter. Her past medical
history is significant for hypertension, treated with metoprolol for 2 years.
Amlodipine was added recently because of inadequate control of BP with metoprolol
alone. She does not smoke or consume alcohol. She has no known drug allergies.
Her blood pressure is 130/80mmHg and her heart rate is 64/min. The physical
examination reveals bilateral symmetric 3+ pitting edema of both lower extremities,
without any skin changes or varicosities. Her neck vein pulsation is normal. Other
physical findings are within normal limits. Her laboratory studies reveal the following:
Serum albumin 4.5 g/dL, Total serum bilirubin 0.8 mg/dL, Serum sodium 140 mEq/L,
Serum potassium 4.0 mEq/L, Serum creatinine 0.8 mg/dL. Urinalysis is within normal
limite. What is the most likely cause of the edema in this patient?
Select one:
 . Side effect of her medications
 . Heart failure
 . Renal disease
 . Venous insufficiency
 . Liver disease

It is important to remember that peripheral edema is a common side effect of the treatment
with dihydropyridine Ca-channel antagonists like amlodipine. These medications can cause
significant edema, as in this patient, due to the property to dilate peripheral blood vessels; it
is NOT an allergic reaction! If the edema is significant, the drug should be discontinued. The
clues to the correct diagnosis in this case are the therapy with a dihydropyridine Ca-channel
antagonist, amlodipine initiated recently, and normal physical examination with normal
laboratory findings. Of course, it is necessary to exclude other more serious causes of
edema.
Congestive heart failure (Choice A) is an important cause of lower extremity edema, but the
absence of other symptoms of heart failure (e.g., dyspnea, orthopnea) and physical findings,
especially elevated neck vein pulsation and liver enlargement, makes this diagnosis unlikely.
Liver diseases (Choice B) can cause lower extremity edema, but usually ascites dominates
over peripheral edema, and abnormal laboratory findings characteristic of liver dysfunction
are present (e.g., hypoalbuminemia, hyperbilirubinemia).
Renal diseases (Choice C) can cause peripheral edema due to massive proteinuria (as with
nephrotic syndrome) or fluid retention (such as during acute nephritic syndrome). The
absence of proteinuria, hypoalbuminemia, and normal creatinine level are inconsistent with
renal causes of the edema.
Venous insufficiency (Choice D) is a possibility in this case. Symmetric edema without skin
changes and varicosities, however, argues against venous insufficiency.
Educational objective:
Dihydropyridine Ca-channel antagonists can cause peripheral edema and should always be
considered in the differential diagnosis of this condition, along with other causes, such as
heart failure, renal disease and venous insufficiency.
The correct answer is: . Side effect of her medications

69. A 31-year-old woman voices her anger at the increasing number of immigrants who
compete for job opportunities in her chosen field of computer science. She observes
that many of her friends in the local community are also experiencing difficulty in
securing appealing employment because of the influx of immigrants. This woman
volunteers in her spare time to provide vocational guidance to new immigrant
families in the community. Which of the following psychological defense
mechanisms is she demonstrating?
Select one:
 . Sublimation
 . Reaction formation
 . Splitting
 . Suppression
 . Altruism

Psychological defense mechanisms are our unconscious means of responding and adapting
to different situations while still preserving our self-image. These mechanisms usually serve
to decrease anxiety, and can be classified as immature, neurotic, or mature. Immature
defense mechanisms are more commonly used by children or adolescents and result in
socially unacceptable behavior that prevents the individual from coping with reality.
Neurotic defense mechanisms are fairly common in adults and provide short-term relief, but
often result in long-term complications. Mature defense mechanisms are most often used
by adults and allow for a "healthy" and more constructive approach to reality.
A neurotic defense mechanism, reaction formation is the transformation of an unwanted
thought or feeling into its opposite. In this scenario, the woman is angry with immigrants
who compete with her for employment. She then transforms her anger into a willingness to
offer job search advice to this very same group of people.
(Choice A) Suppression is a mature defense mechanism that involves intentionally
postponing the exploration of anxiety-provoking thoughts by substituting other thoughts.
This allows for the individual to cope with the present reality instead. An example would be
a woman who focuses on her children's homework questions instead of thinking about her
father's diagnosis of cancer.
(Choice B) Splitting is an immature defense mechanism in which individuals classify
themselves and others as "all good" or "all bad." An example would be a woman suffering
from borderline personality disorder who considers her current therapist to be ideal and her
former therapist to be loathsome.
(Choice D) Altruism is an example of a mature defense mechanism in which individuals
provide a helpful, gratifying service to others as a means of quelling their own anxieties. An
example is a rape victim who establishes a rape crisis center in her community.
(Choice E) Sublimation is a mature defense mechanism that allows for unacceptable
impulses to be channeled into more acceptable activities. An example would be a man with
a fiery temper who channels his anger into athletic pursuits.
Educational Objective:
Reaction formation is the transformation of an unwanted thought or feeling into its
opposite.
The correct answer is: . Reaction formation

70. A 69-year-old woman complains of easy fatigue and one episode of presyncope. On
examination of the jugular venous pressure (JVP), there are irregular large a waves.
The ECG has fixed PP and RR intervals but varying PR intervals. Which of the
following conditions is this most likely caused by?

Select one:
 a reentry phenomenon
 surgical removal of an atrium
 a heart rate under 60 beats/min
 a drug effect
 independent beating of atria and ventricles
AV dissociation is the independent beating of atria and ventricles and is recognized on the
ECG by fixed PP and RR intervals but variable PR intervals.
AV block is one cause of AV dissociation. (Fuster, pp. 904–905)
The correct answer is: independent beating of atria and ventricles

71. A 45-year-old Asian man presents to your office complaining of easy fatigability. He
denies abdominal pain, distention, nausea, vomiting, or significant weight loss. His
past medical history includes a gastrectomy for a non-healing gastric ulcer. He is not
currently taking any medications. He quit smoking several years ago and does not
use alcohol or illicit drugs. His vital signs are within normal limits. Physical
examination reveals a shiny tongue and pale palmar creases. No lymphadenopathy,
hepatomegaly, or splenomegaly is present. His blood hemoglobin level is 7.5 mg/dL
and W8C count is 3,800/mm3. Stool tests for occult blood are repeatedly negative.
This patient's condition involves which of the following pathophysiologic
mechanisms?
Select one:
 . Impaired DNA synthesis
 . RBC membrane instability
 . Impaired hemoglobin synthesis
 . Mechanical RBC injury
 . Impaired glutathione synthesis

Vitamin B 12 deficiency is common after a total or partial gastrectomy. Under normal


circumstances, gastricderived intrinsic factor binds to vitamin B 12 in the stomach and
facilitates its absorption in the ileum. If a significant portion of the stomach has been
removed, sufficient intrinsic factor will not be produced and B 12 absorption will be severely
limited. Other common causes of B 12 deficiency are pernicious anemia, gastritis, intestinal
disorders preventing absorption, and elderly age. Clinically, vitamin B 12 deficiency presents
as a megaloblastic, macrocytic anemia with glossitis and neurologic changes.
Vitamin B 12 is a required cofactor for the demethylation of methyl-tetrahydrofolate ( CH3-
THF) to THF. THF itself is a cofactor used in a number of biochemical processes, including
the formation of purine molecules for DNA synthesis. Thus, B 12 deficiency interferes with
DNA synthesis.
(Choice A) RBC membrane instability is the mechanism behind the anemia seen in
hereditary spherocytosis.
(Choice B) Impaired hemoglobin synthesis is the mechanism behind iron deficiency anemia.
Hemoglobin synthesis is also impaired in the hemoglobinopathies, such as sickle cell
anemia and a- and 13-thalassemia.
(Choice D) Impaired glutathione synthesis is the mechanism behind the hemolytic anemia of
glucose-6- phosphate dehydrogenase deficiency.
(Choice E) Mechanical injury to red blood cells can occur when they encounter foreign
material in the
vascular system. The classic example of this is hemolysis due to red blood cell shearing on
artificial heart valves. Schistocytes are seen on peripheral blood smear.
(Choice F) Immune-mediated hemolysis occurs when lgG autoantibodies attach to the red
blood cell membrane and promote their removal from circulation in the spleen.
Educational Objective:
Vitamin B 12 deficiency is common after a total or partial gastrectomy. Vitamin B 12 is a
necessary cofactor in the pathway leading to purine synthesis. B 12 deficiency decreases
DNA synthesis and erythropoiesis, leading to a megaloblastic anemia.
The correct answer is: . Impaired DNA synthesis

72. A 60-year-old woman with heart failure and normal renal function is started on
furosemide (Lasix) 80 mg/day. She notices a good diuretic response every time she
takes the medication. A few weeks later, she is feeling unwell because of fatigue and
muscle weakness, but her heart failure symptoms are better. Which of the following
is the most likely explanation for her muscle weakness?
Select one:
 hyperkalemia
 hypokalemia
 hypernatremia
 hyponatremia
 anemia

Hypokalemia can result in paralytic ileus, rhabdomyolysis, weakness, and cardiac


repolarization abnormalities. It is a common complication along with hyponatremia of
starting patients on diuretics. (Kasper, p. 260)
The correct answer is: hypokalemia

73. A 53-year-old woman presents to your office with right-sided abdominal pain that
started two days ago. She describes the pain as constant and burning in nature.
There is no associated nausea, vomiting or diarrhea. The patient reports taking over-
the-counter antacids and ibuprofen, which brought no relief. Her medical history is
significant for breast cancer diagnosed one year ago, for which she underwent a
modified radical mastectomy and is receiving chemotherapy, the last course of which
was completed two months ago. On physical examination, her temperature is 36.7°C
(98°F), blood pressure is 120/70 mm Hg, pulse is 80/min, and respirations are 16/min.
Her lung fields are clear to auscultation and her abdomen is soft and non-distended.
The liver span is 10 cm and the spleen is not palpable. Lightly touching the skin to
the right of the umbilicus elicits intense pain. In one week the patient is most likely to
develop:
Select one:
 . Intestinal obstruction
 . Skin lesions
 . Ascites
 . Fever and jaundice
 . Black stool

This patient is immunocompromised secondary to her chemotherapy, and presents with an


area of constant burning pain without any other physical examination abnormalities. Herpes
zoster (shingles) is the most likely diagnosis. Shingles develops when there is reactivation of
the varicella zoster virus (V?Y, the virus that causes chickenpox) in a dorsal root ganglion
where it has remained dormant since a previous chickenpox infection. Reactivation along
the ganglion's distribution causes pain and a vesicular rash in a dermatomal pattern. The
elderly and the immunosuppressed (patients with HIV or diabetes, patients receiving
chemotherapy, etc.) are at highest risk.
Pain is the most common symptom of shingles. The pain can develop several days before
the classic shingles rash appears, or the pain and rash may appear concurrently. The pain is
typically intense, deep, and burning in quality. A small subset of people with shingles go on
to develop a chronic pain syndrome called post-herpetic neuralgia. Treatments for acute
shingles include analgesics and antiviral medications like acyclovir.
Educational Objective:
Herpes zoster (shingles) develops when there is reactivation of V?Y. Patients experience pain
and a vesicular rash in a dermatomal distribution. The pain may precede the rash by several
days or they may appear concurrently.
The correct answer is: . Skin lesions

74. A 25-year-old complains of fever and myalgias for 5 days and now has developed a
macular rash over his palms and soles with some petechial lesions. The patient
recently returned from a summer camping trip in Tennessee. Which of the following
is the most likely cause of the rash?
Select one:
 . Sexual exposure
 . Undercooked pork
 . Tick exposure
 . Contact dermatitis
 . Contaminated water

(Fauci, pp 122-128, 1061.) The rash described is most consistent with Rocky Mountain
spotted fever, for which a tick is the intermediate vector. Secondary syphilis could present
with a macular rash in the same distribution, but the associated symptoms would be
atypical. Always think of these two diagnoses when a rash begins on the palms and soles.
Contact dermatitis would not cause petechial lesions. The skin lesions in disseminated
gonococcal infection can be distal, but are usually few in number and are pustular.
Giardiasis does not cause a rash. Trichinosis, typically associated with periorbital edema and
severe myalgias, can cause splinter hemorrhages and a maculopapular rash, but would
rarely show the distal involvement seen in this patient.
The correct answer is: . Tick exposure

75. A 42-year-old woman is evaluated for chronic abdominal pain and fatigue. Her pain
is epigastric, crampy, and sometimes awakens her from sleep. She denies any recent
weight loss, nausea, or vomiting. Her diet consists mainly of fruits and vegetables.
She also complains of a "strange appetite" for paper and ice that she has never had
before. Upper gastrointestinal endoscopy reveals an ulcer located on the anterior
wall of the duodenal bulb. Her unusual appetite is most directly related to:
Select one:
 . Vitamin deficiency
 . Chronic bleeding
 . Oral leukoplakia
 . H. pylori infection
 . Lactose intolerance

Pica is the phenomenon of having an appetite for items other than food, such as paper
products, clay, or dirt. It is a behavioral symptom sometimes present in iron deficiency.
Pagophagia is pica for ice and is quite specific for iron deficiency anemia. It may be present
before anemia develops and rapidly responds to iron supplementation. Pica may also be
present as a manifestation of psychiatric disease and people may eat very odd items, such
as light bulbs or hair. The patient in this vignette has a low-iron diet and a chronic bleeding
ulcer that has led to iron deficiency and pica.
(Choice A) It is likely that this patient has H. pylori infection; however, it is the chronic
bleeding, not the infection itself, that causes pica.
(Choice B) Vitamin B 12 (cobalamin) deficiency can occur in the setting of chronic gastritis
due to decreased production of intrinsic factor. B 12 deficiency is not associated with ulcers
or pica.
(Choice D) Oral leukoplakia is a precancerous white patch or plaque on the oral mucosa.
Approximately 1% -20% will progress to cancer within 10 years. Oral leukoplakia is not
associated with iron deficiency anemia or pica.
(Choice E) Lactose intolerance is not responsible for iron deficiency or pica.
(Choice F) Reflux esophagitis can cause iron deficiency if it is present for a prolonged period.
In this vignette, however, esophagitis was not seen on endoscopy. Rather, a duodenal ulcer
was observed.
Educational Objective:
Pica refers to an appetite for nonnutritive substances, such as ice, clay, dirt, and paper
products. Pica, especially for ice, can be indicative of iron deficiency.
The correct answer is: . Chronic bleeding

76. A 46-year-old man complains of exertional dyspnea and dry cough. He also
describes occasional episodes of a suffocating night-time cough that is relieved only
when he stands up. His medical history is significant for a myocardial infarction six
months ago. His current medications are metoprolol, aspirin and simvastatin. He
does not use tobacco but drinks alcohol on social occasions. His father died of a
stroke and his mother suffers from diabetes mellitus. His blood pressure is 150/100
mmHg and his heart rate is 60/min. Chest examination reveals bibasilar rales. His
cardiac apex is palpated in the sixth intercostal space. The liver span is 12 cm.
Bilateral pitting leg edema is also present. Which of the following most likely
contributes to his edema?
Select one:
 . High sodium delivery to the distal tubule
 . Increased renal potassium loss
 . Increased renal blood flow
 . High portal venous resistance
 . Constriction of the renal arterioles

This patient has ischemic heart disease, cardiomegaly, pulmonary edema, paroxysmal
nocturnal dyspnea, elevated venous pressure, and peripheral edema, all findings consistent
with a diagnosis of congestive heart failure ( CHF). In heart failure, there is a decrease in the
effective circulating blood volume, leading to renal hypoperfusion. Due to activation of the
renin-angiotensin-aldosterone system, the kidney's arterioles constrict (both afferent and
efferent) and sodium reabsorption is enhanced. These changes increase water retention and
elevate the total body volume, further exacerbating the CHF.
(Choice B) In CHF, there is a decrease in sodium delivery to the distal tubule due to both
decreased renal perfusion and increased sodium retention more proximally in the kidney.
(Choice C) In CHF, renal perfusion is decreased, not increased.
(Choice D) Sodium is the principle electrolyte responsible for edema. Potassium does not
play a significant role in the development of edema.
(Choice E) Portal hypertension is the hallmark of cirrhosis. This patient does not have
significant risk factors for cirrhosis (e.g. alcoholism, hepatitis C) and does not have other
stigmata of chronic liver disease like spider angiomata, palmar erythema, or caput medusa.
(Choice F) Hypoalbuminemia is the leading cause of decreased plasma colloid pressure. Low
serum albumin concentrations typically result from nephrotic range proteinuria or poor
albumin synthesis. This patient's presentation is more consistent with congestive heart
failure than hypoalbuminemia.
Educational objective:
Edema in congestive heart failure is due to both mechanical impedance to fluid removal and
increased renal sodium retention.
The correct answer is: . Constriction of the renal arterioles

77. A 23-year-old male comes to ER with five day history of diarrhea and abdominal
pain. Initially, the diarrhea was watery occurring five-six times per day but yesterday
he noticed blood in the stool which prompted his visit to ER. He describes his
abdominal pain as colicky and severe. He also complains of nausea and decreased
appetite but he has had no vomiting. His past medical history is insignificant and
never had similar symptoms. He is not sexually active and he denies any illicit drug
use. He has no history of recent travel. His father had colon cancer and his uncle died
of liver cirrhosis. His temperature is 36.6°C (98.0°F), blood pressure is 123/82 mmHg
and heart rate is 102/min. On examination, he has prominent periumbilical and right
lower quadrant tenderness but no guarding or rebound. Rectal examination reveals
brownish stool mixed with blood. Which of the following is the most likely diagnosis?
Select one:
 . E coli infection
 . Clostridium difficile colitis
 .Inflammatory bowel disease
 . Protozoal infection
 . Vibrio infection

Diarrhea has a wide differential diagnosis, not all causes of which are infectious. The
presence of bloody diarrhea along with an absence of a travel history limits the differential
somewhat. This patient's abdominal tenderness with an absence of fever is most suggestive
of infection with Enterohemorrhagic E. coli (EHEC). Shigella, Salmonella, and Campylobacter
can also cause bloody diarrhea but often result in fever and/or lack of abdominal pain. EHEC
is different from other strains of E. coli because it produces a Shiga toxin that causes its
propensity to cause bloody diarrhea. The most common serotype of EHEC in the US is 0
157:H7. Most cases are caused by ingestion of undercooked ground beef, although it is not
uncommon for patients to not remember a particular exposure. Potential complications
include development of HemolyticUremic Syndrome (HUS) or Thrombotic
Thrombocytopenic Purpura (TIP). A stool culture could be considered to confirm the
diagnosis and determine antibiotic susceptibilities.
(Choice A) Clostridium difficile diarrhea should be suspected if the patient reported a history
of recent hospitalization or antibiotic usage.
(Choice C) Inflammatory bowel disease, particularly ulcerative colitis which may present with
bloody diarrhea, would be in the differential diagnosis in this case. However, the short
course of this patient's illness makes an infectious etiology more likely.
(Choice D) Protozoal infections with Giardia, Cryptosporidium, and Entamoeba histolytica
are more rare causes of diarrhea and often occur in patients with a particular risk factor such
as travel or immunocompromise.
(Choice E) Vibrio can be a cause of diarrheal illness in individuals living along the Gulf Coast
or with a recent travel history, but is overall a much more rare cause of diarrhea and
typically does not present with bloody diarrhea.
(Choice F) Viral gastroenteritis usually results in vomiting in addition to diarrhea.
(Choice G) Yersinia is a rare cause of diarrhea and is usually associated with fever.
Educational objective:
The finding of bloody diarrhea has a somewhat more limited differential diagnosis than
diarrhea in general. The presence of abdominal pain and lack of fever in a patient without a
travel history makes Enterohemorrhagic E. coli (EHEC) the most likely diagnosis. Most causes
are caused by ingestion of undercooked beef.
The correct answer is: . E coli infection

78. A 65-year-old man was brought to the emergency department after his wife and son
were unable to wake him up this morning. His past medical history is significant for
hypertension for the past 7 years. For the past year, he has had several episodes of
chest pain, which was triggered by physical activity and relieved by rest. His current
medications are nitrates and "baby aspirin." His blood pressure is 140/80 mm Hg,
heart rate is 85/min, and respirations are 15/min. The physical examination reveals an
obese man with impaired consciousness. He has a Glasgow score of 6, asymmetric
pupils, and brisk deep tendon reflexes in the left extremities. Plantar stimulation
provokes extension of the left great toe. The EKG shows no abnormalities. His CK-MB
serum levels are within the normal range, and his LDL cholesterol level is 150 mg/dl.
What is the most likely etiopathology of this patient's symptoms?
Select one:
 . Acute left ventricular failure with decreased cerebral perfusion
 . Chronic subdural hematoma
 . Thrombus migration from the left side of the heart
 . Atherosclerotic emboli obstructing a major cerebral artery
 . Hemorrhagic stroke resulting from hypertensive crisis

Cerebrovascular disease is responsible for 200,000 deaths each year in the United States,
and occurs most commonly in the middle and late years of life. The etiopathology can either
be ischemic (85%) or hemorrhagic ( 15%). Ischemic cerebrovascular accidents result from
migration of emboli from sites that are more prone to atherosclerotic disease (e.g., aortic
arch, carotid bifurcation, and intracranial vessels). The emboli then migrate to and obstruct
the small cerebral vessels, thereby interrupting the cerebral blood flow and causing
subsequent death of the affected tissues. The neuronal damage results in the appearance of
focal neurologic signs, which appear in the next few seconds after the obstruction. Clinically,
atherothrombotic strokes occur at rest and have a gradual onset. Patients often experience
successive strokes with increasing frequency.
This patient has many risk factors of atherosclerotic disease (age, obesity, long history of
hypertension, and
hypercholesterolemia), thereby making this etiopathology more likely. The neurologic exam
reveals hemiplegia of the pyramidal type (brisk deep tendon reflexes and Babinski sign),
which is suggestive of an upper motor neuron lesion due to obstruction of a major cerebral
artery. The asymmetrical pupils are due to the increased intracranial pressure, which resulted
from ischemia and subsequent cerebral edema.
(Choice B) This diagnosis could have been considered if there were EKG signs of myocardial
infarction and/or a high level of cardiac enzymes.
(Choice C) Ischemic cerebrovascular accidents can also be caused by migration of red
thrombi from the left side of the heart ( cardioembolic strokes). This is often associated with
a sudden onset and preexisting cardiac disease, such as atrial fibrillation, myocardial
infarction, mural thrombus and dilated cardiomyopathy. EKG findings of arrhythmias or
recent myocardial infarction (which both may be responsible for the formation of a red
thrombus in the heart) are characteristic. Cardioembolic strokes have a more sudden onset
and often involve a large cerebral artery. The most common site of obstruction is the lateral
striate arteries (arteries of stroke), which are the penetrating branches of the middle cerebral
artery. These supply the internal capsule, the caudate nucleus, putamen and globus pallidus.
There is also a higher risk of transforming into a hemorrhagic stroke. In this case, the
patient's normal cardiac enzyme levels and EKG findings make this diagnosis unlikely.
(Choice D) A hemorrhagic stroke usually occurs while the person is awake or under stress. It
is less common than ischemic cerebrovascular accidents, and is most commonly caused by a
hypertensive crisis. Other causes include transformation of a cardioembolic stroke,
arteriovenous malformation, amyloid angiopathy and coagulation disorders. The most
commonly affected structures are the basal ganglia. The associated neurologic symptoms
result from the mass effect of the extravasated blood.
(Choice E) Chronic subdural hematoma is characterized by clinical symptoms (e.g.,
headache, progressive change in personality, slowed thinking, seizure) which are more
prominent than focal neurologic signs.
Educational Objective:
Atherothrombotic strokes occur at rest and have a gradual onset. Often, the patient
experiences successive strokes with increasing frequency. Atherosclerotic risk factors are
frequently present in such patients.
The correct answer is: . Atherosclerotic emboli obstructing a major cerebral artery

79. A 66-year old female has been your patient for the last 8 years. She was diagnosed
with colorectal carcinoma 2 years ago, and eventually underwent an endoscopic
resection. Since then, she has been healthy, and has been coming to the office
regularly for follow-up visits. She is very grateful, and has stated many times that she
owes her life to you. You are currently a co-investigator of a retrospective
observational study of patients with colon cancer, and you believe that including her
medical information will be extremely beneficial. What course of action must you
take so that you can include this patient's data in your study?
Select one:
 . Include the data and inform her whenever she comes next time
 . Have the data de-identified by a colleague, then include it in the study
 . Include the data only after taking informed consent
 . Call her and obtain verbal consent to include her data
 . Include the data, as she has been your patient for so many years

A patient's data may be used for research purposes only after obtaining informed consent.
This includes explaining the following to the patient:
1 . The purpose of the research
2. The foreseeable risks or discomforts that the patient may experience
3. That refusal or withdrawal will not affect current or future care that the patient is receiving
4. That the patient can withdraw from the study anytime he/she wants to
(Choice C) Informed consent has to be written; verbal consent is not recommended.
Educational Objective:
A patient's data may be used for research purposes only after obtaining informed consent.
The correct answer is: . Include the data only after taking informed consent

80. A 34-year-old male who recently emigrated from Asia comes to the clinic and
complains of a two-month history of exertional shortness of breath and easy
fatigability. He has been taking isoniazid and rifampin for his tuberculosis, which was
diagnosed four months ago. Due to his religious beliefs, he completely turned into a
vegetarian for the last year. Physical examination reveals severe pallor. Peripheral
smear shows macrocytosis with hypersegmented polymorphonuclear neutrophils.
His WBC and platelet counts are within normal limits. This patient's most likely
problem is due to which of the following?
Select one:
 . B12 deficiency because of the pernicious anemia
 . B12 deficiency because of the vegetarian diet
 . Drug-induced B12 deficiency
 . Chronic myeloid leukemia
 . Myelodysplastic syndrome

The most likely diagnosis is vitamin B 12 deficiency due to pernicious anemia, which is an
autoimmune disease resulting from the destruction of parietal cells. The next step in the
management of this patient is measurement of the intrinsic factor antibody titers and B 12
levels.
(Choice A} A strict vegetarian diet can cause vitamin B 12 deficiency; however, the total body
vitamin B 12 stores are depleted slowly, and can usually last for three to four years after
starting such a diet. This patient has been on a vegetarian diet for only one year; therefore, a
diet-related B 12 deficiency is very unlikely.
(Choice C) Anti-tuberculous drugs such as INH can cause B6 or pyridoxine deficiency (not
B12 deficiency}.
(Choices D and E) Normal \NBC and platelet counts are unlikely with myelodysplastic
syndromes and chronic myeloid leukemia. Furthermore, CML is usually characterized by
elevated WBC count, splenomegaly and systemic symptoms; whereas, myelodysplastic
syndromes are usually characterized by pancytopenia. Myelodysplastic syndrome is usually
seen in elderly patients.
Educational Objective:
The causes of vitamin B 12 deficiency include a strict vegetarian diet (after three or four
years} and pernicious anemia.
The correct answer is: . B12 deficiency because of the pernicious anemia

81. An 8-year-old boy is brought to the office by his mother for a routine check-up. He has
fair skin, blond hair and blue eyes. His past medical history is insignificant. His mother
wants to know what the best possible photo-protection is for her son, because "his skin
has always been sensitive to the sun, and he is almost unable to tan." He had two
episodes of sunburn recently. Physical examination reveals several junctional nevi. Which
of the following is the best response to this patient's mother?
Select one:
 . Minimize sun exposure in the middle of the day
 . Recommend applying sunscreens before sun exposure
 . Rest under trees or umbrellas during the day
 . Emphasize that clothing is typically useless for sun protection
 . Reassure and provide routine care

Sun exposure is associated with skin photo-aging and several types of skin cancer, including
melanoma. These potential subsequent conditions emphasize the importance of appropriate
patient education regarding photo-protection, especially in the high-risk group. Encouraging
photo-protection is the leading preventive measure in skin care, and sun avoidance is the best
method of photo-protection. Patients are generally instructed to avoid outdoor activities in the
middle of the day (between 1 0 am and 4 pm), which is the period of peak sun radiation
intensity.
(Choice B) Sunscreens are useful adjuncts to photo-protection, but offer insufficient protection
from ultraviolet radiation (UVR) when used alone. Overdependence on sunscreens may
sometimes even increase or encourage outdoor exposure. Sunscreens should be applied 15~0
min prior to sun exposure to allow enough time for protective film development.
(Choice D) Resting under umbrellas decreases UVR exposure by 70%, but does not affect
reflected radiation. Shade from trees provides insufficient UVR protection.
(Choice E) Clothing is considered to be an effective tool for sun protection; however, it does not
supercede sun avoidance.
Educational Objective:
Appropriate patient education about photo-protection, especially in the high-risk group, is
important. Sun avoidance remains as the best method of photo-protection.
Know this: Sunscreens should be applied 15~60 min prior to sun exposure to allow enough time
for protective film development.
The correct answer is: . Minimize sun exposure in the middle of the day

82. A 73-year-old man from a nursing home develops headache, fever, cough, sore throat,
malaise, and severe myalgia during a community outbreak affecting numerous other
residents at the home. The symptoms gradually resolve after 3 days, and he starts feeling
better but then there is a reappearance of his fever, with cough and yellow sputum
production. On examination, his temperature is 38.5°C, pulse 100/min, respiration
24/min, oxygen saturation 88% and crackles in the right lower lung base, bronchial
breath sounds and dullness on percussion. CXR reveals a new infiltrate in the right lower
lobe. Which of the following is the most likely causative organism?
Select one:
 primary viral pneumonia
 an autoimmune reaction
 Mycoplasma pneumoniae
 Neisseria catarrhalis
 Streptococcus pneumoniae

This man has a complication of viral influenza; a secondary bacterial pneumonia has
developed. This usually occurs 2–3 days after the initial viral symptoms resolve. Primary viral
pneumonia with influenza is not common. S. pneumoniae, Staphylococcus, and H.
influenzae are the most common bacterial invaders in pulmonary complications of influenza.
Pneumonia is the leading cause of death and may also be due to S. pneumoniae and H.
influenzae. Mixed viral and bacterial pneumonia is common; pure viral pneumonia in
influenza is uncommon (but can be very severe). (Kasper, p. 1068)
The correct answer is: Streptococcus pneumoniae

83. A 46-year-old farmer comes to the physician because of pain and swelling of his
right heel. He accidentally stepped on a rusty nail while working 2 weeks ago.
Although he applied over the-counter antibiotic cream locally and took
acetaminophen orally, he continued to have pain at the site of the injury. He has no
other medical problems and takes no medication. He has no known drug allergies.
He smokes 2-3 cigarettes a day and drinks alcohol occasionally. His temperature is
38.3°C (101°F), blood pressure is 140/90 mm Hg, pulse is 84/min and respirations are
14/min. On examination, the right heel is swollen, red, and warm and tender to
touch. A small puncture wound is visible. Laboratory reports show mild leukocytosis.
X-ray of the right foot shows features suggestive of osteomyelitis. The most likely
cause of the patient's symptoms is infection with which of the following?
Select one:
 . Escherichia coli
 . Staphylococcus epidermidis
 . Pseudomonas aeruginosa
 . Beta-hemolytic streptococci
 . Clostridium tetani

Although Staphylococcus aureus is the most common cause of osteomyelitis in children and
adults, Pseudomonas aeruginosa is a frequent cause of osteomyelitis in adults with a history
of a nail puncture wound (especially when the puncture occurs through rubber-soled
footwear). The patient presents with local pain and swelling, fever, and an increased white
cell count. Blood cultures may reveal the infecting microorganism; otherwise, a bone biopsy
is required. Plain radiographs take about 2 weeks or more to show evidence of the disease.
Treatment is with oral or parenteral quinolones and aggressive surgical debridement.
(Choices A and 8} Streptococci and coagulase-negative staphylococci rarely cause
osteomyelitis in the absence of predisposing factors such as diabetes mellitus whereas S.
aureus (coagulase-positive) is a common cause of osteomyelitis.
(Choice C) Although the patient is at a risk of developing tetanus, Clostridium tetani does
not cause osteomyelitis.
(Choice D) Candida osteomyelitis is sometimes seen in injection drug users, but the
presentation is subacute, unlike bacterial osteomyelitis.
(Choice E) Escherichia coli is a rare cause of osteomyelitis. Pseudomonas is a much more
frequent cause of puncture wound osteomyelitis.
(Choice F) Mycobacterium tuberculosis is a rare cause of osteomyelitis in the United States,
although it is common in developing countries. The most commonly affected bone is the
spine (Pott's disease).
Educational objective:
A nail puncture wound in an adult resulting in osteomyelitis is most likely due to
Pseudomonas aeruginosa.
The correct answer is: . Pseudomonas aeruginosa

84. A 47-year-old male presents to your office with a two-month history of lethargy and
decreased libido. His medical records reveal that he has been treated for joint pain
and swelling over the last six months and was diagnosed with diabetes mellitus one
year ago. Physical examination reveals hepatomegaly and testicular atrophy. Which
of the following cardiac abnormalities is most likely to also be present in this patient?
Select one:
 . Aortic stenosis
 . Hypertrophic cardiomyopathy
 . Atrial septal defect
 . Cardiac conduction block
 . Endocardial fibroelastosis

This patient presents with signs and symptoms of a multi-organ system disease process,
including hypogonadism (testicular atrophy, decreased sexual drive), arthropathy, pancreatic
endocrine dysfunction (diabetes) and hepatomegaly. Hemochromatosis is a condition that
can cause the above findings, in addition to cardiac abnormalities. Cardiac involvement may
include dilated or restricitve cardiomyopathy leading to heart failure or various conduction
abnormalities. Hemosiderin deposition in the pituitary can lead to hypogonadotropic
hypogonadism and deposition in the joints can cause arthropathy.
(Choice A) Atrial septal defects are usually isolated, sporadic congenital anomalies. Only
rarely do they occur as part of an inherited disorder involving other organ systems (e.g.
Holt-Oram syndrome, which also causes upper limb deformities).
(Choice B) Aortic stenosis generally results from localized senile calcific changes to the valve,
which may be accelerated by the presence of a congenitally bicuspid aortic valve.
(Choice C) Hypertrophic cardiomyopathy results from mutations in genes that encode
cardiac sarcomere proteins. It is not known to be associated with hepatomegaly, testicular
atrophy, pancreatic endocrine insufficiency or arthropathy.
(Choice E) Endocardial fibroelastosis is a rare, idiopathic, diffuse fibroelastic thickening of
the left ventricular endocardium. It usually occurs in the first two years of life and is
generally not associated with hepatomegaly, testicular atrophy, pancreatic endocrine
insufficiency or arthropathy.
Educational objective:
Hereditary hemochromatosis can cause cirrhosis, pancreatic fibrosis (diabetes) and
increased skin pigmentation due to melanin and hemosiderin deposition (bronze diabetes),
as well as abnormalities of cardiac conduction.
The correct answer is: . Cardiac conduction block

85. A 45-year-old black female presents to you because of a dry cough and has not been
feeling well for the past month. She did have some pain in her shins a few weeks
ago. She denies any allergies, smoking, or other medical problems. Her ECG shows
normal sinus rhythm. The chest x-ray is shown below. What is the most likely
pathophysiology in this patient?

Select one:
 . Pulmonary venous congestion
 . Interstitial lung disease
 . Lupus pneumonitis
 . Inflammatory granulomas
 . Malignancy of the lung

Sarcoid is a granulomatous disease seen typically in young black females. It is a systemic


disorder, which primarily affects the lungs. The condition may be asymptomatic or the
patients may present with dyspnea, fever, night sweats, or weight loss. The classic finding on
a chest x-ray is bilateral mediastinal adenopathy, as seen above. Some times patients may
present just with erythema nodosum (painful skin lesions over the shins). Some individuals
may have skin, eyes, liver, kidney, and central nervous system involvement. Elevated calcium
and angiotensin converting enzyme levels are also seen. Once mediastinal adenopathy is
seen on a chest x-ray, the next step is to perform a mediastinal bronchoscopy and obtain
tissue for biopsy. Symptomatic patients require steroids (Choice C).
(Choice A) The above clinical presentation and findings are not representative of congestive
heart failure. In a young female, congestive heart failure would be a rare finding without any
history. On chest x-ray, the lung fields will be engorged and Kerley B lines will be seen.
(Choice B) Lupus can present with numerous systemic organ findings, but adenopathy is not
typical. Patients may have a rash, arthralgia, pneumonitis, brain syndromes, nephritis, and a
vasculitis. Diagnosis is by findings of antibodies to dsDNA
(Choice D) Lung cancer can present with adenopathy. Small cell cancer can present in such a
way; however, the majority of patients with such significant adenopathy from lung cancer
will be symptomatic. Weight loss, night sweats, and hemoptysis may be the other
symptoms. Small cell cancer is rare in patients under the age of 40-50, especially if they are
nonsmokers.
(Choice E) Most interstitial lung disease present with lung fibrosis and not adenopathy.
Patients may have progressive dyspnea, weight loss, and fever. Lung parenchymal changes
are seen in the majority, but a normal x-ray does not rule out interstitial lung disease. ACT
scan is required, followed by a lung biopsy.
Educational Objective:
Sarcoid can present in a black female with bilateral hilar adenopathy. Recognize the
association with erythema nodosum. Biopsy typically shows non-caseating granulomas.
Serum Ca and ACE levels are elevated. Steroids are the treatment of choice in symptomatic
patients.
The correct answer is: . Inflammatory granulomas

86. A 43-year-old man feels vaguely unwell. Physical examination is unremarkable except
for evidence of scleral icterus. The skin appears normal. Which of the following is the
most likely explanation for why early jaundice is visible in the eyes but not the skin?
Select one:
 . the lighter color of the sclera
 . the high elastin content of scleral tissue
 . the high blood flow to the head with consequent increased bilirubin delivery
 . secretion via the lacrimal glands
 . the high type II collagen content of scleral tissue

The sclera are high in elastin content, which has an affinity for bilirubin. Therefore, jaundice
is usually detected here first. Fluorescent lighting makes recognition more difficult. In some
individuals, dark skin color makes jaundice more difficult to detect. (Kasper, p. 238)
The correct answer is: . the high elastin content of scleral tissue
87. A 57-year-old male is hospitalized for hyponatremia. Physical examination reveals no
signs of fluid overload. Serum analysis reveals a sodium concentration of 125mEq/1,
glucose level of 12mg/dl, BUN of 8mg/dl, and creatinine of 0.7mg/dl. The urine
osmolarity is 330mOsm/l and urine sodium concentration is 45mEq/1. After a 2L
normal saline infusion, the serum sodium is 126mEq/1 and the urine sodium
excretion is increased to 90 mEq/1. Which of the following is the most likely cause of
this patient's hyponatremia?
Select one:
 . Advanced liver disease
 . Inappropriate ADH secretion
 . Surreptitious diuretic use
 . Interstitial renal disease
 . Psychogenic polydipsia

Hyponatremia is defined as a serum sodium value less than 130 mEq/L. It typically results
from defects in the total body water balance. The underlying cause can be determined
based on assessment of the serum osmolarity extracellular fluid volume status and urine
sodium concentration as follows·

The serum osmolarity can be calculated as follows:


s osm = (2 X Na) + (BUN / 2.8) + (Glucose / 18)
This patient has a low serum osmolarity (260 mOsm/kg) in the setting of a clinically normal
intravascular volume status. The BUN and creatinine are normal with a BUN/Cr ratio less
than 20, indicating adequate renal perfusion and function. The initial spot urine sodium is 45
mEq/L, which is inappropriately elevated considering the patient's hyponatremia (the
kidneys should be retaining sodium maximally, giving a urine sodium less than 1 0 mEq/L).
We can therefore conclude that this patient has euvolemic hyponatremia due to renal
sodium losses. Given that the urine osmolarity is increased and greater than the serum
osmolarity, the syndrome of inappropriate antidiuretic hormone secretion (SIADH) is the
most appropriate diagnosis.
ADH acts on the distal tubule and collecting ducts to increase free water reabsorption from
the urine. Inappropriate secretion of ADH thus causes hyponatremia. Diagnostic criteria for
SIADH include:
1. s osm < 270
2· u osm > s osm
3. UNa> 20 mEq/L
4. Absence of hypovolemia
5. Normal renal, adrenal and thyroid function
6. No obvious surgical, traumatic or painful stimulus known to activate the neuroendocrine
stress response, including ADH release
7. Absence of other known causes of hyponatremia
This patient meets the first four criteria, and based on the information given, may also meet
the last three. In addition, when a normal saline fluid bolus is given, the urine sodium
excretion appears to increase while the serum sodium is relatively unaffected, indicating that
the kidneys are continuing to inappropriately excrete sodium and retain water under the
influence of ADH.
(Choice B) Psychogenic polydipsia would result in euvolemic hyponatremia because obligate
sodium losses in the urine would decrease the plasma sodium concentration as the body
attempted to eliminate the excess free water. The urine osmolality would be decreased.
(Choices C & E) Diuretic overuse, interstitial nephritis and other renal salt-wasting conditions
would cause hypovolemia, elevations in the BUN and BUN/Cr ratio, and possible
isosthenuria (U0 8m = S0 8ml due to impaired renal concentrating ability.
(Choice D) Advanced liver disease, particularly cirrhosis, would cause signs of total body
water overload (ascites, interstitial edema) due to hypoalbuminemia (a result of decreased
liver synthetic function). Widespread edema causes a relative depletion of intravascular
volume, which causes the kidneys to maximally retain water. In this setting, renal sodium
retention would be maximal resulting in a low UNa.
Educational objective:
SIADH is characterized by euvolemic hyponatremia, decreased serum osmolarity, elevated
urine osmolarity (often greater than serum osmolarity), increased urine sodium
concentration, and failure to correct with normal saline infusion.
The correct answer is: . Inappropriate ADH secretion
88. A 61-year-old man is being evaluated for progressive exertional dyspnea and
decreased exercise tolerance. He has also noticed some ankle swelling. The patient
has a 40 pack-year smoking history but denies illicit drug use or occupational
exposures. His chest x-ray is shown below. The abnormal diaphragmatic contour
seen here most likely results in which of the following?

Select one:
 . Higher inspiratory flow
 . Decreased lung compliance
 . Increased thoracic wall recoil
 . Increased work of breathing
 . Reduced expiratory effort

This patient's history, presentation, and chest x-ray are consistent with chronic obstructive
pulmonary disease (COPD). The hallmark of COPD is progressive airflow obstruction with a
forced expiratory volume in 1 second (FEV1) to forced vital capacity (FVC) ratio of less than
0.7. Patients with COPD also have increased total lung volume, residual volume, and lung
compliance. The increased lung volumes cause the sort of diaphragmatic flattening seen
above. Large lung volumes and diaphragmatic flattening make it more difficult to decrease
intrathoracic pressure during inhalation and thus increase the work of breathing.
(Choice A) Patients with COPD have reduced inspiratory flow rates.
(Choice B) Due to the obstructive nature of the disease, patients with COPD require
increased expiratory effort. However, the diaphragm plays little role in expiration.
(Choice D) Diaphragmatic flattening does not increase thoracic wall recoil.
(Choice E) COPD results in coalescence of alveoli to form a spongier, more compliant lung.
Patients with restrictive lung disease have decreased lung compliance.
Educational objective:
COPD causes airflow limitation and increased lung volumes. The latter results in
diaphragmatic flattening, evident on chest x-ray, which further exacerbates patients'
dyspnea and work of breathing.
The correct answer is: . Increased work of breathing

89. A 60-year-old woman comes to the physician because of a 3-month history of


worsening fatigue and back pain. She has had diabetes mellitus for the past three
years, and hypertension for the past ten years. Laboratory studies show: Hb 9.0
mg/dL, Serum calcium 11.2 mg/dL, Serum phosphorus 3.5 mg/dL, BUN 38 mg/dL,
Serum creatinine 2.0 mg/dL. Which of the following is the most likely cause of this
patient's renal failure?
Select one:
 . Diabetes mellitus
 . Primary hyperparathyroidism
 . Para protein
 . Renal artery stenosis
 . Hypertension

The clinical presentation of this patient is suggestive of multiple myeloma. Multiple


myeloma (plasma cell myeloma, myelomatosis, Kahler's disease) is characterized by clonal
proliferation of plasma cells, resulting in excessive production of a single immunoglobulin
type (monoclonal paraproteins). It is most commonly seen in the elderly (median age of 65),
and is characterized by normochromic normocytic anemia, fatigue, and bone pain
(especially in the back and chest). Labs show anemia (Hb< 12), hypercalcemia, Bence Jones
proteins in the urine, and a monoclonal protein peak on electrophoresis. X-rays may reveal
punched out lesions, osteoporosis, or pathologic fractures.
Approximately 50% of multiple myeloma patients develop some degree of renal
insufficiency along the course of their disease. This is most likely due to obstruction of the
distal and collecting tubules by large laminated casts containing paraproteins (mainly Bence
Jones proteins). Hypercalcemia, hyperuricemia, amyloid deposition and pyelonephritis may
occur with multiple myeloma and contribute to renal insufficiency. Acute renal failure may
also be precipitated after an infection, dehydration, and the use of IV dye.
(Choice A) Hypercalcemia usually does not occur in patients with hypertension-related renal
failure.
(Choice B) Diabetic nephropathy usually occurs after 10- 15 years of having diabetes
mellitus.
(Choice D) Primary hyperparathyroidism can also cause hypercalcemia, which can lead to
dehydration (from polyuria) and subsequent acute renal failure; however, decreased serum
phosphorus levels are also seen with this condition. Secondary hyperparathyroidism can
occur in patients with renal failure due to hypocalcemia and hyperphosphatemia. Multiple
myeloma thus remains as the most likely cause of this patient's hypercalcemia.
Educational Objective:
Any elderly patient with bone pain, renal failure, and hypercalcemia has multiple myeloma
until proven otherwise. Approximately 50% of multiple myeloma patients develop some
degree of renal insufficiency; this is most likely due to obstruction of the distal and
collecting tubules by large laminated casts containing paraproteins (mainly Bence Jones
protein).
The correct answer is: . Para protein

90. A 21-year-old woman presents to the ED complaining of diarrhea, abdominal


cramps, fever, anorexia, and weight loss for 3 days. Her BP is 127/75 mm Hg, HR is 91
beats per minute, and temperature is 100.8°F. Her abdomen is soft and nontender
without rebound or guarding. WBC is 9200/μL, β-hCG is negative, urinalysis is
unremarkable, and stool is guaiac positive. She tells you that she has had this similar
presentation four times over the past 2 months. Which of the following
extraintestinal manifestations is associated with Crohn disease but not ulcerative
colitis?
Select one:
 . Ankylosing spondylitis
 . Erythema nodosum
 . Uveitis
 . Nephrolithiasis
 . Thromboembolic disease

(Tintinalli, pp 536-539.) Crohn disease is characterized by chronic inflammation extending


through all layers of the bowel wall. Onset is generally between the ages of 15 and 40 years.
Crohn disease should be suspected in any patient whose symptoms show a picture
consistent with chronic inflammatory colitis. Extraintestinal manifestations are seen in 25%
to 30% of patients. The incidence is similar for Crohn disease and ulcerative colitis. They
include aphthous ulcers, erythema nodosum, iritis or episcleritis, arthritis, and gallstones.
Nephrolithiasis is seen as a result of hyperoxaluria because of increased oxalate absorption
in patients with ileal disease. Because ulcerative colitis affects only the large bowel, this
extraintestinal manifestation is seen only in patients with Crohn disease.
The correct answer is: . Nephrolithiasis

91. A 60-year-old man presents with fever and malaise 6 weeks after mitral valve
replacement. On examination, his temperature is 38°C, blood pressure 130/80 mm
Hg, pulse 80/min, and a loud pansystolic murmur at the apex, which radiates to the
axilla. He has no skin or neurologic findings. Which of the following is the most likely
causative organism?
Select one:
 a fungus
 Staphylococcus saprophyticus
 pneumococcus
 Staphylococcus epidermidis
 Staphylococcus aureus

About half of all early-onset (<60 days after surgery) prosthetic endocarditis is caused by
staphylococcal infection, with S. epidermidis predominating. Early-onset prosthetic
endocarditis is generally the result of intraoperative contamination of the prosthesis or a
bacteremic postoperative complication. (Kasper, p. 732)
The correct answer is: Staphylococcus epidermidis

92. A 60-year-old man is brought to the ER by his wife because he lost consciousness in
the bathroom at night. He says that he woke up, went to the bathroom to urinate,
and fainted there. He rapidly recovered his consciousness without any indication of
disorientation. He has never had such an episode before. He admits' problems with
urination,' including difficulty with initiating urination and frequent awakening to
void at night. He does not take any medication. His past medical history is
insignificant. He smokes 2 packs of cigarettes per day and does not consume alcohol.
His blood pressure is 130/80 mmHg while supine, and 132/80 mmHg while standing.
His heart rate is 70/min. His physical examination is within normal limits. The ECG is
normal. What is the most probable cause of the syncopal episode in this patient?
Select one:
 . Transient ischemic attack (TIA)
 . Situational syncope
 . Postural hypotension
 . Seizure
 . Arrhythmia

The most probable diagnosis in this patient is situational syncope related to micturition. The
scenario described (middle age or older male with prostatic hypertrophy, who lost his
consciousness after awakening and voiding at night) is typical for this type of syncope. The
pathophysiologic mechanism underlying the situational syncope includes autonomic
dysregulation, which can be partially explained by straining and rapid bladder emptying.
Cardioinhibitory and/or vasodepressor mechanisms may be involved.
Cardiovascular causes like arrhythmia (Choice A) are less likely because there are no signs of
a structural heart disease on the physical examination and the ECG.
Postural hypotension (Choice B) is unlikely because there are no signs of orthostatic BP
changes, as well as no obvious cause for the problem, such as medications or hypovolemia.
A patient with no previous history and a rapid recovery of consciousness argue against
seizure (Choice D) as a cause of the episode.
Transient ischemic attack (Choice E) is a rare cause of syncope, and is usually accompanied
by symptoms of focal neurologic dysfunction (ataxia, paresis, diplopia).
Educational objective:
Situational syncope should be considered in the differential diagnosis of syncopal episodes.
The typical scenario would include a middle age or older male, who loses his consciousness
immediately after urination, or a man who loses his consciousness during coughing fits.
The correct answer is: . Situational syncope

93. A 43-year-old man presents to your office with low energy and increased fatigability.
He also complains of daytime sleepiness and occasional headaches. He drinks two to
three glasses of wine daily but does not smoke. He sleeps in a separate room from
his wife because she finds his constant snoring annoying. On physical examination,
his blood pressure is 160/100 mmHg and his heart rate is 80/min. His BMI is 31.5
kg/m2. His abdomen is soft and non-tender. The liver span is 10 cm and the spleen is
not palpable. Laboratory findings are: Hematocrit 60%, WBC count 9,000/mm3,
Platelets 190,000/mm3. Which of the following is most likely responsible for this
patient's increased hematocrit?
Select one:
 . Ineffective erythropoiesis
 . Increased erythropoietin production
 . Plasma volume contraction
 . Carboxyhemoglobinemia
 . Clonal proliferation of myeloid cells

This patient has obstructive sleep apnea (OSA), a disease characterized by recurrent
transient obstruction of the upper airway due to pharyngeal collapse during sleep. Affected
patients tend to be overweight or obese and have excessive daytime sleepiness, snoring,
morning headaches, impotence, and arterial hypertension. The transient obstructive
episodes in OSA cause short-term hypoxemia, which is sensed by the kidneys and stimulates
increased erythropoietin production. Erythropoietin drives the creation of more red blood
cells and results in polycythemia.
(Choice A) Plasma volume contraction can cause spurious polycythemia by concentrating
red blood cells within the vasculature. The most common causes of volume contraction are
diarrhea, vomiting, and poor oral intake.
(Choice B) Polycythemia vera is a myeloproliferative disease that causes a clonal
proliferation of myeloid cells. In polycythemia vera, the concentrations of all blood cells lines
are increased.
(Choice C) Patients with carbon monoxide (CO) poisoning typically present with dizziness,
headaches, nausea, and confusion, and risk factors for CO inhalation can usually be
ascertained from the patient history. CO poisoning decreases the blood's oxygen-carrying
capacity, and the body responds by developing a secondary polycythemia. OSA is a more
likely diagnosis than CO poisoning in this patient based on his history.
(Choice E) Ineffective erythropoiesis is a hallmark of both vitamin 8 12 and folic acid
deficiencies. In these deficiency diseases, the erythroid precursors in the bone marrow die.
The result is a megaloblastic anemia, not polycythemia.
Educational objective:
Hypoxemia-induced increases in erythropoietin production are responsible for the
polycythemia observed in obstructive sleep apnea. Treatment of the OSA causes the
polycythemia to improve.
The correct answer is: . Increased erythropoietin production

94. A 65-year-old Caucasian male had undergone cardiac catheterization followed by


aortic valve replacement for severe aortic stenosis and coronary artery bypass
grafting for three-vessel disease. His postoperative course was complicated by atrial
fibrillation and a urinary tract infection. His other medical problems include
hypertension, diabetes, and hypercholesterolemia. He is also receiving heparin,
ciprofloxacin, and amiodarone. On postoperative day five, he developed prolonged
bleeding from the venipuncture site. His labs show: Hb 11.5 g/dL, MCV88 fl, Platelet
count 50,000/cmm, Leukocyte count 7,500/cmm, Segmented neutrophils 68%, Bands
1%, Eosinophils 1%, Lymphocytes 24%, Monocytes 6%, Prothrombin time 12 sec
(INR=1.0), Partial thromboplastin time 65 sec. His preoperative labs were
unremarkable. What is the most likely cause of these findings in this patient?
Select one:
 . Vitamin deficiency
 . Hemolytic uremic syndrome
 . Thrombotic thrombocytopenic purpura
 . Medication effect
 . Idiopathic thrombocytopenia purpura

The coagulation profile in this patient is suggestive of elevated PTI and a decrease in
platelet count. While the elevated PTI is a therapeutic effect of heparin, the
thrombocytopenia is an adverse effect of the same. Patients on heparin should be regularly
followed up with platelet counts.
(Choice A) Thrombotic thrombocytopenia purpura (TIP) is a rare form of consumptive
thrombocytopenia caused by widespread platelet thrombi that form in the microcirculation.
Thrombocytopenia, microangiopathic hemolytic anemia, fever, and neurological signs
characterize it.
(Choice B) Hemolytic uremic syndrome (HUS) often follows diarrhea. HUS and TIP belong to
a similar spectrum. HUS is also manifested by thrombocytopenia, microangiopathic
hemolytic anemia, and fever; but, instead of more neurologic signs, patients will have renal
failure.
(Choice C) Idiopathic thrombocytopenia purpura (ITP) is a diagnosis of exclusion and is
characterized by thrombocytopenia. All other cell lines, PT, and PTI will be normal.
(Choice D) Vitamin-K deficiency leads to a fall in plasma levels of all the prothrombin
complex proteins (factor 2, 7, 9, 10, and protein C and S). This would lead to a prolonged
PTfollowed by a prolonged PTI and PT » PTI. Thrombocytopenia is not a feature.
(Choice F) Disseminated intravascular coagulation is a potentially life threatening
coagulation disorder characterized by an initial thrombotic phase followed by a phase of
procoagulant consumption and secondary fibrinolysis. Labs would show thrombocytopenia,
prolonged PT, PTI, and thrombin time. A reduced fibrinogen level from depletion of
coagulation proteins and elevated fibrin degradation products (FOP) from intense
secondary fibrinolysis is also seen.
(Choices G and H) Factor VIII and IX deficiency are an X-linked inherited plasma coagulation
disorders and would cause prolongation of PTI. Also, it would manifest early in life. It is
impossible to clinically distinguish between factor VIII and IX deficiency, and a detailed
laboratory work up is required.
(Choice I) Von Willebrand's disease is the most frequently diagnosed inherited bleeding
disorder in adults. Labs would show an increase in bleeding time with an increase in PTI. PT
will be normal. Platelet count is normal.
Educational Objective:
HIT is seen in about 5- 15% of patients taking therapeutic heparin with onset between 3- 15
days after initiation of heparin and resolution within 4-5 days of discontinuation of heparin.
While the elevated PTI is a therapeutic effect of heparin, the thrombocytopenia is an
adverse effect of the same.
The correct answer is: . Medication effect

95. An 18-year-old man is brought to the emergency room by police after he is found
wandering in the street, screaming loudly at passersby. In the emergency room he is
placed in an examination room, and paces the floor and pounds his fist against the
door repeatedly. Which of the following actions should be taken by the psychiatrist
first?
Select one:
 . Rule out psychosis
 . Rule out an organic mental disorder
 . Make sure the physical environment is safe for the interviewer
 . Give the patient 5 mg of haloperidol IM
 . Put the patient into soft restraints

(Kaplan, p 909.) The first step in assessing a potentially violent (or already demonstrably
violent patient in this vignette) is to assure the physical environment is safe for the
interviewer, since if this condition is not met, none of the other assessment steps can be
taken. Ruling out organic mental disorders and psychosis can only be done by directly
assessing the patient, and the interviewer must be safe to do so. The patient may well
require haloperidol and restraints (although in a patient like this soft restraints would not be
chosen, rather full leather restraints would be required), but again, this must come second
to the interviewer’s safety so these things may be determined.
The correct answer is: . Make sure the physical environment is safe for the interviewer

96. A 67-year-old Caucasian male complains of progressive visual loss in his right eye
over the past several months. He has a history of hypertension and type 2 diabetes
mellitus. Current medications include a daily baby aspirin, hydrochlorothiazide,
lisinopril, and metformin. There is no family history of visual problems. He has a 35
pack year smoking history and admits to occasional alcohol use. He is afebrile with a
blood pressure of 137/82 mmHg and pulse of 73/min. Cardiac and pulmonary
examinations are unremarkable. A neurologic examination demonstrates no focal
motor or sensory abnormalities. The patient is asked to cover his left eye and to look
at a small spot on a grid made of parallel vertical and horizontal lines. He describes
the vertical lines as being bent and wavy. Which of the following is the most likely
cause of this patient's complaints?
Select one:
 . Peripheral retinal degeneration
 . Enlarged blind spot
 . Macular degeneration
 . Lens opacity
 . Increased intraocular pressure

The most likely diagnosis in this case is macular degeneration, which is the leading cause of
blindness in industrialized countries. One of the earliest findings in macular degeneration is
distortion of straight lines such that they appear wavy. The grid test described above is
frequently used to screen for patients with macular degeneration. The primary risk factor for
macular degeneration is increasing age, although smoking can increase the risk as well.
Patients may be asymptomatic, but others complain of visual problems in either one or both
eyes. Driving and reading are often some of the first activities that are affected since they
require fine visual acuity, which is provided primarily by the macula. In addition to the grid
test described above, ophthalmologic examination can identify drusen deposits in the
macula, which are common lesions seen with this disorder.
(Choice A} Lens opacification is the cause of visual loss from cataracts. Cataracts, however,
should not cause straight lines to appear wavy.
(Choice B) An enlarged blind spot may be seen with papilledema, but this patient has no
evidence of increased intracranial pressure to result in papilledema.
(Choice C) Increased intraocular pressure is the cause of glaucoma, which does not typically
present with distortion of straight lines.
(Choice E) Visualization of straight lines is a task that requires fine visual acuity, which is
controlled primarily by the centrally-located macula as opposed to the more peripheral
retina.
Educational objective:
Macular degeneration is the most common cause of blindness in industrialized nations.
Activities that require fine visual acuity are usually the first affected, and patients with this
condition may report that straight grid lines appear curved.
The correct answer is: . Macular degeneration

97. A 21-year-old college junior requests an urgent appointment with her family
physician to discuss a "personal concern." During her conversation with the doctor,
the woman appears tense, speaks very little, and makes minimal eye contact. After
gentle questioning, she breaks down and says that her father is taking lithium for a
manic episode experienced two years ago. She had not realized this could have any
bearing on her own situation until she encountered an article on the Internet that
mentioned genetics appears to play a role in the pathogenesis of bipolar disorder.
Since then, she has had disturbed sleep and has been very worried that she too will
one day develop bipolar disorder. Based on clinical data, what is the likelihood that
she will have a manic episode during her lifetime?
Select one:
 . 1%
 . 75%
 . 60%
 .40%
 . 10%

 More common in single or divorced individuals and in the higher socioeconomic


classes, bipolar disorder has one of the strongest genetic components of all
psychiatric disorders. The college student in this case should be told that a child with
a first-degree relative ( eg, parent, sibling, or dizygotic twin) who suffers from bipolar
disorder has a 5-1 0% risk of developing the condition in her lifetime. A child whose
parents both suffer from bipolar disorder has a 60% risk of developing the condition,
and a monozygotic twin of an individual who suffers from bipolar disorder has a 70%
risk of developing the condition.
 (Choice A) For the general population, the lifetime risk of developing bipolar disorder
is 1%.
 Educational Objective:
 For the general population, the lifetime risk of developing bipolar disorder is 1%.
However, an individual with a first-degree relative ( eg, parent, sibling, or dizygotic
twin) who suffers from bipolar disorder has a 5-1 0% risk of developing the condition
in his lifetime.
 The correct answer is: . 10%

98. A 67-year-old male presents with a one-month history of dyspnea on exertion and
chest pain. He denies hemoptysis, cough, fever, night sweats or weight loss. His past
medical history is significant for hypertension and chronic obstructive pulmonary
disease. He has a 48 pack-year smoking history but quit six years ago. He works as a
salesman. On physical examination, the patient has a temperature of 38.2°C (100.8°F),
blood pressure of 128/72 mm Hg, pulse of 92/minute, and respirations of 20
breaths/minute. His complete blood count, chemistry panel, and hepatic function
tests are all within normal limits. A chest x-ray and computed tomography scan of
the chest reveal a right-sided pleural effusion and calcified nodules in both upper
lobes. The patient undergoes thoracentesis. Characteristics of the pleural fluid are
given below: Total protein 5.2 g/dl, Glucose 83 mg/dl, Adenosine deaminase 98.5
U/L. Cytologic examination reveals 600 white blood cells/mm3, 1% macrophages and
99% leukocytes. Of the leukocytes, 45% are neutrophils, 50% are lymphocytes, and
5% are monocytes, with no basophils or eosinophils. No neoplastic cells are found.
Which of the following is the most likely mechanism underlying development of this
pleural effusion?
Select one:
 . Elevation of hydrostatic pressure
 . Decreased plasma osmotic pressure
 . Increased capillary permeability
 . Reduction of pleural space pressures
 . Passage of fluid through openings in the diaphragm

Pleural effusions can be either transudative or exudative. Light's criteria compare pleural
fluid characteristics to serum characteristics to help clarify which fluid type is present.
Generally speaking, exudative fluid is more similar to serum than transudative fluid.
Exudative pleural effusions have one or more of the following:
1 . A ratio of pleural fluid protein to serum protein greater than 0.5
2. A ratio of pleural fluid LDH to serum LDH greater than 0.6
3. Pleural fluid LDH greater than 2/3 the upper limit of normal serum LDH
Based on the data given, this patient has an exudative pleural effusion. Assuming his serum
total protein concentration is at the upper limit of normal (i.e. 7 g/dL), the ratio of pleural to
serum total protein is greater than 0.5. Given his history, subacute presentation, and
abnormal chest imaging, pulmonary tuberculosis is the most likely cause. This etiology is
supported by the high adenosine deaminase concentration in the thoracentesis fluid, which
is characteristic of pleural and ascites fluid related to tuberculosis. Pleural effusions can
occur in patients with pulmonary tuberculosis because the inflammation increases capillary
permeability, which causes fluid to leak into the pleural space.
(Choice A) Elevations in hydrostatic pressure can cause transudative pleural effusions. This is
the mechanism behind pleural effusions that develop in congestive heart failure.
(Choice B) Decreased plasma osmotic pressure can cause transudative pleural effusions. This
is the mechanism behind pleural effusions that develop in cirrhosis and nephrotic syndrome.
(Choice D) In patients with ascites and portal hypertension, passage of fluid through
openings in the diaphragm can contribute to ' hepatic hydrothorax.' In these patients, fluid
tracks from the abdomen into the pleural space through the diaphragm. These pleural
effusions are transudative.
(Choice E) A sudden reduction in pleural space pressure (e.g., from a pneumothorax or lobar
collapse) can cause a transudative pleural effusion.
(Choice F) Lymphatic obstruction from tumors or fibrosis can cause exudative effusions.
Educational objective:
Pleural effusions are classified as transudative or exudative based on Light's criteria. Patients
with tuberculosis may develop exudative pleural effusions due to increased capillary
permeability. This fluid characteristically has a high adenosine deaminase concentration.
The correct answer is: . Increased capillary permeability

99. A 40-year-old male patient presents with a thyroid nodule. His other complaints are
episodes of palpitations, anxiety and sweating. He denies heat intolerance. His
weight and appetite are normal. He has a family history of thyroid cancer. His pulse is
80/min, and blood pressure is 160/100 mmHg. Examination of the neck shows a 4-
cm, hard, non-tender thyroid nodule. The urinalysis, serum sodium, serum potassium,
serum calcium, serum creatinine, serum PTH, TSH, T3 and T 4 levels, and the EKG are
all normal. The serum calcitonin level is elevated. The urinary levels of metanephrine
and norepinephrine are increased as well. FNA biopsy of the thyroid nodule shows
malignant cells. Genetic testing shows a mutation in the RETproto oncogene. Which
of the following abnormalities is also present in most patients suffering from this
disorder?
Select one:
 . Pituitary adenoma
 . Brain tumor
 . Pancreatic islet cell tumor
 . Mucosal neuroma
 . Parathyroid adenoma
 Multiple endocrine neoplasia type 2 is an autosomal dominant disorder. It is further
classified into MEN types 2a and 2b. Both subtypes are characterized by medullary
carcinoma of the thyroid and pheochromocytoma. The distinguishing feature for
MEN type 2a is primary hyperparathyroidism, while that of MEN type 2b are mucosal
neuromas and marfanoid habitus. Primary hyperparathyroidism in MEN type 2a is
due to parathyroid hyperplasia rather than parathyroid adenoma. Mucosal neuromas
are the most distinctive feature of MEN type 2b, and are present in more than 90% of
cases; these are present on the tongue, eyelids, lips and gastrointestinal tract.

 In this case, the patient's presentation (i.e., medullary thyroid cancer,


pheochromocytoma, normal levels of serum calcium and PTH) essentially rules out
MEN type 2a. Since the most probable diagnosis at this point is MEN type2b,
mucosal neuromas are most likely to be seen in his physical examination.
 (Choices B, C and D) pituitary adenoma, pancreatic islet cell tumor and
hyperparathyroidism are features of MEN type 1 (the 3 P's). MEN 1 is due to
mutations in the tumor suppressor gene called Menin.
 Educational Objective:
 Mucosal neuromas are the most distinctive feature of multiple endocrine neoplasia
type 2b, and are present in more than 90% of cases.
 The correct answer is: . Mucosal neuroma

100. A 62-year-old man with a prosthetic aortic valve develops fevers and malaise.
His valve was replaced 5 years ago because of aortic stenosis from a bicuspid valve.
He has a systolic ejection murmur but no other abnormalities on examination. Blood
cultures are most likely to grow which of the following?
Select one:
 diptheroids
 fungi
 Staphylococcus epidermidis
 bartonella
 Streptococcus bovis

S. epidermidis is still the most frequent early and late cause of endocarditis in patients with
prosthetic heart valves. The other organisms are seen less frequently in late prosthetic valve
endocarditis. (Fuster, pp. 2004–2005)
The correct answer is: Staphylococcus epidermidis
101. A 47-year-old male was brought to the emergency room with chest pain of acute
onset. The pain was associated with nausea, vomiting, and diaphoresis. He has a history
of diabetes, hypertension, and hyperlipidemia. ECG reveals ST segment elevation in the
anterolateral leads and ventricular premature beats (VPBs). The patient dies within the
first hour after the arrival to emergency room. What is the most likely pathophysiologic
mechanism responsible for this patient's death?
Select one:
 . Increased automaticity
 . Electro-mechanic dissociation
 . Reentry
 . Asystole
 . Full conduction block

The most common cause of death in patients with acute myocardial infarction (MI) is complex
ventricular arrhythmia. Acute ischemia creates heterogeneity of conduction in the myocardium.
Areas of partial block of conduction are frequently formed that predispose the patient to
reentrant arrhythmia. Ventricular fibrillation is a typical example of reentrant arrhythmia. Decline
in mortality of patients hospitalized with acute coronary syndromes is largely attributable to the
effective detection and treatment of reentrant ventricular arrhythmias.
Full conduction block (Choice C) may occur in patients with acute myocardial infarction,
especially inferior wall Ml, but it is a less common cause of death.
Asystole (Choice E) usually occurs in patients with severe complications of Ml (e.g., heart failure).
Electromechanical dissociation (Choice A) is typical for pulmonary thromboembolism and
pericardia! tamponade.
Increased automaticity (Choice D) is a frequent cause of arrhythmia in patients with glycoside
intoxication.
Educational Objective:
Reentrant ventricular arrhythmia (ventricular fibrillation) is the most common cause of death in
patients with acute myocardial infarction.
The correct answer is: . Reentry

102. A 23-year-old man comes to the psychiatrist with a chief complaint of a


depressed mood. He is very anxious and obviously uncomfortable in the physician’s
office. Which of the following actions should be used to help develop rapport with this
patient?
Select one:
 . Express compassion with the difficult position the patient is in
 . Tell the patient that you too are nervous when you see new patients
 . Ask the patient why he is so unusually anxious about seeing a psychiatrist
 . Get right to the patient’s complaint so that the patient can leave as soon as possible
 . Inform the patient that his problem is simple and easily fixed
(Kaplan, p 1.) A patient in this situation needs empathy above all else if a successful rapport is to
be developed. Informing the patient that his problem is simple and easily fixed might eventually
bring some relief to the patient, but if stated early in the interview process can sound
condescending, as if the patient should not trouble the physician with such trivial things.
Expressing compassion for the difficult situation the patient finds himself in is showing true
empathy with the patient’s current discomfort. Telling the patient that you are nervous about
seeing new patients too could be seen as an expression of empathy with the patient, but may
also make the patient feel dismayed, since he wants a confident and competent physician to
treat him. Asking the patient why he is so unusually nervous will only make the patient MORE
self-conscious, and it is NOT unusual for patients to be this nervous on a first visit to a
psychiatrist (especially if they have never seen one before). Finally, getting right to the patient’s
complaint just ignores the uncomfortable feeling the patient has come in with, and this will not
help the development of rapport (nor is it very observant of the psychiatrist).
The correct answer is: . Express compassion with the difficult position the patient is in

103. A 65-year-old woman complains of periodic headaches in the temporal region,


visual disturbances, and neck stiffness. Treatment is initiated early and biopsy of a scalp
artery is consistent with arteritis. Two months later, the patient presents to your office
with weakness. She says that her headaches are gone but she has difficulty climbing
stairs and getting up from a chair. Her serum CK level and ESR are normal. Which of the
following is the most likely cause of this patient's current complaints?
Select one:
 . Drug-induced myopathy
 . Symmetric polyneuropathy
 . Mononeuritis multiplex
 . Inflammatory myositis
 . Polymyalgia rheumatica

This patient has corticosteroid-induced myopathy. While the exact mechanism of steroid
myopathy is unknown, it is thought to relate to decreased protein synthesis, mitochondrial
alterations, increased protein degradation, and electrolyte and carbohydrate metabolism
disturbances. There are two forms of steroid myopathy, acute and chronic. Acute steroid
myopathy is relatively uncommon and is characterized by diffuse muscle weakness and
rhabdomyolysis during high-dose steroid treatment. The more common, chronic form has an
insidious onset with prolonged steroid use. It is characterized by proximal muscle weakness
without significant pain. Patients may complain of difficulty rising from a chair or brushing their
hair. For both diseases, muscle strength improves after discontinuation of the steroids but it may
take weeks to months to see improvement.
The patient in this vignette has a history consistent with temporal (giant cell) arteritis. This
disease is treated his high-dose oral steroids that are tapered over a long period of time. The
onset of her symptoms with the initiation of steroid therapy is consistent with chronic steroid
myopathy.
(Choice A) Polymyalgia rheumatica is the other diagnosis to consider in the differential for the
above case, as 50% of patients with temporal arteritis will also have PMR. However, unlike with
steroid-induced myopathy, pain is a prominent finding, ESR will be elevated, and symptoms will
remit with steroid treatment.
(Choices B-E) These conditions are unlikely given the temporal course of events.
Edu cational Obj ective:
Steroid-induced myopathy is a well described result of long-term corticosteroid use. It is
characterized by painless proximal muscle weakness. Steroid myopathy will improve slowly once
the offending medication is discontinued.
The correct answer is: . Drug-induced myopathy

104. A 38-year-old man is being seen in his physician’s office after being involved in a
car accident. He has a vague pain along his right sternal border, where he crashed into
the steering wheel. His temperature is 36.6°C (97.8°F), pulse is 80/min, blood pressure is
123/75 mm Hg, respiratory rate is 14/min, and oxygen saturation is 99% on room air.
Physical examination is significant for point tenderness over the right sternal border. X-
ray of the chest shows no broken ribs but a single, well-circumscribed pulmonary nodule,
1.5 cm in diameter, located in the left lower lung field. A search through the patient’s
electronic medical file reveals that he had an x-ray of the chest taken 2 years ago. The
radiology report from that time reveals that the nodule was only 0.75 cm in diameter. To
characterize the lesion, CT of the chest is performed and shows dense, flocculated
calcification within the lesion. Which of the following risk factors most increases the
chances of malignancy in this patient?
Select one:
 Nodule diameter of 1.5 cm or higher
 Presence of flocculated calcification
 Presence of discrete border
 Increased patient age
 Increased doubling time of tumor

A solitary pulmonary nodule is defined as a lung nodule, <3 cm in size that is discovered on x-
ray or CT of the chest. Solitary pulmonary nodules have a 40% chance of being malignant.
Review of old radiographs is crucial because they can give an indication of the nodule’s
doubling time. A doubling time between 20 and 400 days is more concerning for malignancy.
For example, small cell cancer doubles every 30 days, adenocarcinoma doubles every 180 days,
and welldifferentiated cancers have an even longer doubling time. Benign processes generally
have doubling times that are <20 days (infection) or >450 days (old granulomas). The doubling
time for this patient is not 2 years. On x-ray of the chest a pulmonary nodule appears as a coin-
shaped lesion, when in actuality it is best approximated as a three-dimensional sphere. Thus, if a
calculation is performed for the volume of the nodule/sphere, we observe close to an eightfold
increase in size. Thus, the lesion doubled in size three times during the 2-year period, meaning
that the doubling time was 2 years divided by 3, or every two-thirds of a year (243.5 days).
Answer B is incorrect. The likelihood of malignancy increases with age. In one study, the chances
of a nodule being malignant as a function of age were 3% in patients 35–39 years old, 15% in
those 40–49 years old, 43% in those 50–59 years old, and 50% in those >60 years old. In another
study, nodules in patients >50 years old had a 65% likelihood of malignancy, whereas those in
patients <50 years old had a 33% likelihood of being malignant. Given the patient’s young age
(38 years), doubling time is a much more likely risk factor for malignancy.
Answer C is incorrect. Nodule size is directly related to the likelihood of malignancy. Generally, a
nodule >3 cm in diameter significantly increases the likelihood of malignancy. In one study, 80%
of nodules >3 cm were malignant compared with 20% in nodules <2 cm.
Answer D is incorrect. Benign nodules tend to have a smooth, discrete border, while malignant
lesions have an irregular border.
Answer E is incorrect. The presence of certain patterns of calcification (best seen on CT) favors
the lesion being benign. Such patterns include “popcorn” calcification (as in this patient),
diffuse/ homogenous calcification, central calcification, and laminated/concentric calcification.
However, the existence of areas of calcification is not associated with a benign lesion. An
eccentric lesion having an asymmetric area of calcification should raise concern of malignancy.
The correct answer is: Increased doubling time of tumor

105. An 80-year-old female is brought from a nursing home with a two day history of
poor oral intake and lethargy. Her past medical history is significant for hypertension,
diabetes mellitus, coronary artery disease with coronary stenting two years ago and
Alzheimer's disease. She was hospitalized six months ago with pneumonia. Her current
medications are aspirin, lisinopril, metoprolol, hydrochlorothia zide, metformin and
memantine. Her blood pressure is 95/60 mmHg and heart rate is 90/min. Physical
examination reveals fine crackles at the right lung base. Her mucous membranes are dry.
Her laboratory findings are significant for WBC of 15,000/mm3, creatinine of 2.1 and
BUN of 61 mg/dl. Her creatinine was 0.9 mg/dl during the last admission. Which of the
following is the best explanation for the abnormal laboratory findings in this patient?
Select one:
 . Renal vasoconstriction
 . Renal inflammatory infiltration
 . Age-related renal function decline
 . Obstructive uropathy
 . Renal tubular dysfunction

The patient described is most likely suffering from prerenal azotemia (prerenal ARF). Prerenal
azotemia refers to acute renal insufficiency resulting from insufficient arterial perfusion of the
kidney. In this case, the decreased perfusion is most likely the result of dehydration caused by
poor oral intake of fluids. Prerenal azotemia due to intravascular volume depletion is rapidly
reversible with intravenous volume resuscitation, but persistent untreated renal hypoperfusion
will lead ultimately to intrinsic renal failure due to ischemia. The elderly, who have a diminished
thirst response and a reduced ability to self-administer fluids, are at the greatest risk of prerenal
azotemia. The risk of this condition is worsened when patients are treated with pharmacologic
agents, such as NSAIDS, diuretics and ACE inhibitors, which can blunt the renal response to low
intravascular volume. Physical examination findings consistent with dehydration include relative
hypotension, tachycardia, dry mucous membranes, decreased skin turgor and slow capillary
refill. Laboratory values reflect hemoconcentration and an increased serum BUN/Cr ratio.
(Choice A) Age-related renal functional decline causes a slow decrease in the GFR due to loss of
renal mass and thickening of the glomerular basement membrane. It would not be expected to
cause an acute or abrupt change in renal function.
(Choice B) Renal inflammatory infiltration occurs in various vasculopathies affecting the kidney,
in toxic insults to the kidney as well as in pyelonephritis.
(Choice C) Acute renal tubular dysfunction is typically the result of toxic or ischemic insults to
the kidney.
(Choice E) Patients with obstructive uropathy may also have an increased BUN/Cr ratio, but they
typically have a normal volume status and would therefore not exhibit hemoconcentration.
Postrenal ARF requires obstruction of both ureters or an obstruction at or distal to the bladder.
(Choice F) Nodular glomerulosclerosis classically occurs in the setting of diabetic nephropathy. It
is a chronic condition of insidious onset characterized by progressive proteinuria and renal
insufficiency.
(Choice G) Hypertensive arteriolosclerosis occurs in the setting of severe hypertension and is
often accompanied by headaches and papilledema. RBCs, RBC casts, and protein are usually
seen in the urine.
Educational objective:
Elderly patients with poor oral intake living in nursing homes and taking medications such as
NSAIDs, ACE inhibitors and diuretics should be suspected of having prerenal azotemia due to
intravascular volume depletion and poor renal perfusion.
The correct answer is: . Renal vasoconstriction

106. An 88-year-old female nursing home resident is brought to the hospital with a
one-week history of moaning, decreased oral intake and decreased ambulation. Physical
examination reveals a very thin female who is moaning and appears to be in pain. She
opens her eyes spontaneously but does not respond to questions. Her blood pressure is
75/43 mmHg, pulse is 105/min, temperature is 35.5°C (96°F), and respirations are 22/min.
Oxygen saturation (pulse oximetry) is 97% on room air. Her skin and oral mucosa are dry
and her neck veins are flat. There is no lymphadenopathy. On chest auscultation, there
are crackles in the right upper lung lobe. Abdomen is soft and non-distended. There is
2+ pitting presacral and lower extremity edema, and a deep pressure ulcer in the sacral
area. Laboratory analysis reveals the following: Sodium 155mEq/L, Potassium 5.3 mEq/L,
Glucose 88mg/dL, Bicarbonate 14mEq/L, BUN 151mg/dL, Creatinine 3.1mg/dl. Which of
the following is the most likely cause of her lower extremity edema?
Select one:
 . Increased plasma hydrostatic pressure
 . Decreased plasma oncotic pressure
 . Increased interstitial oncotic pressure
 . Renal water and sodium retention
 . Decreased lymphatic drainage

When a patient presents with multiple, non-specific symptoms and signs, it is helpful to
organize the relevant clinical data. Here we have an elderly, nursing home resident, with issues
including decreased oral intake, pain, and poor communication, as well as physical examination
findings of hypotension, tachycardia, tachypnea, decubitus ulcer, right upper lobe crackles, flat
neck veins, and 2+ pitting edema in both lower extremities. Notable laboratory findings include
hypernatremia, hyperkalemia, elevated BUN and creatinine, BUN/Cr ratio of 48.7 (ratio > 20
indicates prerenal azotemia), and low bicarbonate. Generally speaking, lower extremity edema
can result from increased venous or lymphatic pressure, decreased plasma oncotic pressure,
increased capillary leak, or recent trauma or infection. This patient has hypotension, flat neck
veins and laboratory studies consistent with hypovolemia, so venous hypertension is unlikely.
Lymphatic obstruction is a rare cause of edema, and the patient has no history of
lymphadenectomy, lymphadenopathy or lymphangitis. There is no mention of hemosiderin
pigment deposition on physical examination, so capillary leak is less likely, and there is no
history of trauma or infection of the legs. Thus, the most likely cause of her lower extremity
edema is decreased oncotic pressure resulting from malnutrition and a low serum albumin.
Educational objective:
Bilateral lower extremity pitting edema may be a sign of hypoalbuminemia secondary to
malnutrition. Decreased oncotic pressure causes the edema.
The correct answer is: . Decreased plasma oncotic pressure

107. A 47-year-old woman has new-onset transient right arm weakness and word
finding difficulty symptoms lasting 3 hours. She is also experiencing exertional dyspnea,
and had a syncopal event 1 month ago. Her echocardiogram reveals a cardiac tumor in
the left atrium, it is pendunculated and attached to the endocardium. Which of the
following is the most likely cause of this lesion?
Select one:
 rhabdomyoma
 fibroma
 myxoma
 sarcoma
 lipoma

The myxoma is a solitary globular or polypoid tumor varying in size from that of a cherry to a
peach. About 75% are found in the left atrium, and most of the remainder in the right atrium.
The clinical presentation is with one or more of the classical triad of constitution symptoms
(fatigue, fever, anemia), embolic events, or obstruction of the valve orifice. (Fuster, pp. 2081–
2082)
The correct answer is: myxoma
108. A 57-year-old man with a 40-pack-per-year history of smoking experiences
symptoms of shortness of breath on exertion. He has bilateral wheezes on expiration and
increased resonance to percussion of the chest. Pulmonary function tests confirm the
diagnosis of chronic obstructive lung disease (COPD). Which of the following is the best
definition of this condition?
Select one:
 It is caused by bronchial asthma
 It is due to destruction and dilatation of lung alveoli
 It is airflow limitation that is not fully reversible
 Is due to small airways disease only
 It is preceded by chronic bronchitis

COPD is defined as a disease state that is characterized by airflow limitation that is not fully
reversible. Emphysema and chronic bronchitis are closely related, and the term COPD is often
used to encompass both. Chronic bronchitis is a clinical syndrome defined as excessive
tracheobronchial mucous production severe enough to cause productive cough for at least 3
months of the year for at least two consecutive years. Emphysema is defined as the distention of
air spaces distal to the terminale bronchiole, with destruction of alveolar septa. It is primarily a
histologic diagnosis. Smoking is the usual antecedent for COPD. (Kasper, p. 1547)
The correct answer is: It is airflow limitation that is not fully reversible

109. A 60-year-old man with no past medical history undergoes upper endoscopy and
biopsy for an upset stomach that is worsened by eating. He is found to have
inflammation predominantly in the antrum of the stomach. Which of the following is the
most likely etiology of this condition?
Select one:
 Spicy foods
 Iatrogenic
 Alcohol abuse
 Cigarette smoking
 Infection

Infection with Helicobacter pylori is a cause of type B, or antraldominant, chronic gastritis. The
most common cause of chronic type B gastritis is use of nonsteroidal anti-inflammatory drugs,
but our patient lacks this history. The incidence of chronic gastritis increases with age and the
histologic appearance improves with the treatment of H. pylori. Symptoms of chronic gastritis
may include pain, nausea, vomiting, anorexia, and upper GI bleeding; however, this condition is
frequently asymptomatic. Diagnosis is confirmed by H. pylori antibody test and breath urease
test, as well as a direct gastric biopsy and culture. Treatment of H. pylori gastritis is with triple
therapy, which includes two antibiotics (metronidazole and clarithromycin), bismuth compound,
and a proton pump inhibitor. Type A gastritis is autoimmune-mediated, affecting the fundus,
and is associated with vitamin B12 deficiency.
Answer A is incorrect. Severe alcohol abuse may have negative effects on the gastric mucosa,
including the development of acute (stress) gastritis; however, in a patient with no history of
alcoholism and with inflammation confined to the antrum, alcohol abuse would not be the most
likely cause of gastritis.
Answer B is incorrect. Cigarette smoking may promote atrophic gastritis in H. pylori-positive
patients, but it has not been shown to cause inflammation of the gastric mucosa by itself.
Answer C is incorrect. Although there is some erythema associated with endoscopy, gastritis is
defined as the histologic appearance of inflammation of the gastric mucosa.
Answer E is incorrect. Spicy foods may lead to increased gastric acid production, but would not
be the most likely cause of the inflammation in this case.
The correct answer is: Infection

110. A 28-year-old primigravida is admitted to the hospital at 10weeks gestation. Her


right leg is swollen. Her BMI is 30 Kg/m2. Her temperature is 36.7°C (98.2°F), respirations
are 12/min and pulse is 96/min. Her ABG shows the following: pH 7.49, PaCO2 50 mm
Hg, HCO3- 44 mEq/L. Which of the following is the most likely cause of her abnormal
arterial blood gas?
Select one:
 . Pulmonary embolism
 . Hyperemesis gravidarum
 . Obesity
 . Aspiration pneumonitis
 . Normal phenomenon of pregnancy

This patient has metabolic alkalosis with respiratory compensation. This can be determined by
her ABG results showing a pH greater than 7.45, a primary increase in HC03- level and a
compensatory increase in PaC02. The increase in PaC0 2 does not indicate a mixed primary acid-
base disturbance because the compensatory increase in PaC02 expected given the metabolic
acidosis is approximately 52 - 59 mm Hg. Vomiting, such as in hyperemesis gravidarum, leads to
loss of gastric HCI and metabolic alkalosis. This is known as the generation stage of metabolic
alkalosis due to vomiting. The volume loss that results from repeated vomiting also causes a
contraction metabolic alkalosis due to activation of the renin-angiotensinaldosterone system.
This is known as the maintenance phase.
(Choice A} Hypocapnia is a normal phenomenon of late pregnancy and would cause a
respiratory alkalosis. It is caused by a direct stimulatory effect of progesterone on the central
respiratory center.
(Choice B} This patient may have a deep vein thrombosis (swollen leg}, but a pulmonary
embolus (PE} is unlikely. PE causes a primary respiratory alkalosis due to hyperventilation.
(Choice C) Obesity can cause hypoventilation (Pickwickian syndrome} leading to chronic
respiratory acidosis. While the patient's BMI is at the lower limit of obesity, she is also pregnant.
Additionally, she is not experiencing a primary respiratory acidosis.
(Choice D) Aspiration pneumonia leads to hypocapnia and respiratory alkalosis secondary to
tachypnea.
Educational Objective:
Hyperemesis gravidarum typically causes metabolic alkalosis with respiratory compensation via
hypercapnia.
The correct answer is: . Hyperemesis gravidarum

111. A 33-year-old white female comes to the office for the evaluation of weakness in
her upper extremities. She thinks she is unable to feel pain or heat, because she recently
noted some burn wounds on her fingertips, and does not know how she got them. She
denies weakness in her lower limbs, as well as any history of trauma, headache, bowel or
bladder problems, neck pain or facial pain. Examination reveals absent reflexes in her
upper limbs. There is absent pain and temperature sensation on the nape of neck,
shoulders and upper arms in a 'cape' distribution. Vibration and position sensations are
preserved. Which of the following is the most likely pathology of the patient's condition?
Select one:
 . Cord cavitation
 . Caudal displacement of the cerebellar tonsils and vermis
 . Disc herniation and cord compression
 . Caudal displacement of the fourth ventricle
 . Focal cord enlargement

The above patient is most likely suffering from syringomyelia. Areflexic weakness in the upper
extremities and dissociated anesthesia (loss of pain and temperature with preserved position
and vibration) in a "cape" distribution are classic findings of this condition. The pathology
involves cavitary expansion of the spinal cord, which may produce myelopathy. There is
destruction of the gray and white matter adjacent to the central canal. The most characteristic
feature is the presence of a cord cavity, which usually communicates with the central canal of
the spinal cord. The most frequent site of involvement is the lower cervical or upper thoracic
region. When the syringes occur in the upper cervical cord and extend proximally to involve the
medulla oblongata, the condition is called syringobulbia. Acquired causes of syringomyelia
include trauma, inflammatory spinal cord disorders or spinal cord tumors. Occasionally,
syringomyelia is idiopathic.
(Choice D) When the cord cavity occurs in association with a past injury or intramedullary
tumors, it does not communicate with the central canal, and in such cases focal cord
enlargement may be seen on MRI or myelography.
(Choice B) Most cases of syringomyelia are associated with Arnold-Chiari malformation, in which
there is caudal displacement of cerebellar tonsils through the foramen magnum. In such
patients, neuroimaging may show caudal displacement of the fourth ventricle.
Educational Objective:
Suspect syringomyelia in a patient with areflexic weakness in the upper extremities and
dissociated anesthesia in a "cape" distribution. The presence of a cord cavity is the most
characteristic feature. Caudal displacement of the fourth ventricle or cerebellar tonsils, and focal
cord enlargement may occur.
• Just know how to diagnose syringomyelia.
The correct answer is: . Cord cavitation

112. A 72-year-old woman notices progressive dysphagia to solids and liquids. There
is no history of alcohol or tobacco use, and the patient takes no medications. She denies
heartburn, but occasionally notices the regurgitation of undigested food from meals
eaten several hours before. Her barium swallow is shown. Which of the following is the
cause of this condition?

Select one:
 . Spasm of the lower esophageal sphincter
 . Loss of intramural neurons in the esophagus
 . Scarring caused by silent gastroesophageal reflux
 . Growth of malignant squamous cells into the muscularis mucosa
 . Psychiatric disease

(Fauci, pp 1847-1851.) The barium swallow shows the dilated baglike proximal esophagus and
tapered distal esophageal ring characteristic of achalasia. This is a motor disorder of the
esophagus and classically produces dysphagia to both solids and liquids. Structural disorders
such as cancer and stricture usually cause trouble swallowing solids as the first manifestation. In
achalasia, manometry shows elevated pressure and poor relaxation of the lower esophageal
sphincter. In classic achalasia the contractions of the esophagus are weak, although a variant
called vigorous achalasia is associated with large-amplitude prolonged contractions.
Medications (nitrates, calcium channel blockers, botox injections into the LES) or physical
procedures (balloon dilatation or surgical myotomy) that decrease LES pressure are the
recommended treatments. Squamous cell carcinoma would not cause esophageal dilation and
would be associated with ratty rather than smooth tapering of the esophagus. Achalasia is not
associated with gastroesophageal reflux disease. Although anxiety can cause dysphagiaand a
globus-like sensation in the cricoid region, it would not cause the anatomical changes seen on
this barium swallow.
The correct answer is: . Loss of intramural neurons in the esophagus
113. You are working in the ED on a Sunday afternoon when four people present with
acute-onset vomiting and crampy abdominal pain. They were all at the same picnic and
ate most of the same foods. The vomiting began approximately 4 hours into the picnic.
They deny having any diarrhea. You believe they may have “food poisoning” so you
place IV lines, administer IV fluids, and observe. Over the next few hours, the patients
begin to improve, the vomiting stops, and their abdominal pain resolves. Which of the
following is the most likely cause of their presentation?
Select one:
 . Campylobacter
 . Scombroid fish poisoning
 . Salmonellosis
 . Staphylococcal food poisoning
 . Clostridium perfringens food poisoning

(Rosen, pp 1209-1214.) Staphylococcal food poisoning is caused by an enterotoxin-forming


strain of Staphylococcus organisms in the food before ingestion. Most protein-rich foods
support the growth of staphylococci, particularly ham, eggs, custard, mayonnaise, and potato
salad. The illness has an abrupt onset, beginning 1 to 6 hours after ingestion of the
contaminated food. Cramping and abdominal pain, with violent and frequent retching and
vomiting are the predominant symptoms. Diarrhea is variable; it is usually mild, and occasionally
absent. Although often aggressive in onset, staphylococcal food poisoning is short lived and
usually subsides in 6 to 8 hours, rarely lasting more than 24 hours. The patient is often
recovering when first seen by a physician. The short incubation period and multiple cases in
people eating the same meal are highly suggestive of this disease.
(a) Scombroid fish poisoning results from the ingestion of heat-stable toxins produced by
bacterial action on the dark meat of the fish (eg, tuna and mackerel). The symptoms occur
abruptly within 20 to 30 minutes and resemble histamine intoxication, consisting of facial
flushing, diarrhea, throbbing headache, palpitations, and abdominal cramps. Antihistamine
therapy is usually curative. (c) Clostridium perfringens is probably the most common cause of
acute food poisoning in the United States. Most cases occur in fairly large outbreaks and are
caused by the ingestion of meat and poultry dishes. Symptoms usually appear within 6 to 12
hours but can occur up to 24 hours after ingestion of the contaminated food. Frequent, watery
diarrhea and moderately severe abdominal cramping are the major symptoms. The illness is self-
limited and rarely lasts for more than 24 hours. (d) Campylobacter is the most common bacterial
cause of diarrhea in patients who seek medical attention. The incubation period is approximately
2 to 5 days. Onset of symptoms is usually rapid and consists of fever, crampy abdominal pain,
and diarrhea. Constitutional symptoms are common. (e) Salmonellosis occurs most during the
summer months and is acquired by the ingestion of contaminated food or drink. Poultry
products, such as turkey, chicken, duck, and eggs constitute the most common sources. Family
outbreaks and sporadic cases are more common than large epidemics. The typical patient
presents with fever, colicky abdominal pain, and loose, watery diarrhea, occasionally with mucus
and blood. Symptoms usually abate within 2 to 5 days and recovery is uneventful.
The correct answer is: . Staphylococcal food poisoning

114. A 54-year-old male with a history of chronic alcoholism is admitted to the


hospital with hematemesis and abdominal distension. Upper gastrointestinal endoscopy
reveals the presence of esophageal varices. Continued workup of his condition results in
a diagnosis of cirrhosis of the liver. He is treated appropriately and strongly counseled
about the need to abstain from alcohol. He is then discharged home. After discharge, he
begins to work as a full-time volunteer for a non-profit organization that assists patients
with alcoholic cirrhosis. Which of the following defense mechanisms is this man using?
Select one:
 . Altruism
 . Sublimation
 . Reaction formation
 . Suppression
 . Denial

Psychological defense mechanisms are our unconscious means of responding and adapting to
different situations while still preserving our self-image. These mechanisms usually serve to
decrease anxiety, and can be classified as immature, neurotic, or mature. Immature defense
mechanisms are more commonly used by children or adolescents and result in socially
unacceptable behavior that prevents the individual from coping with reality. Neurotic defense
mechanisms are fairly common in adults and provide short-term relief, but often result in long-
term complications. Mature defense mechanisms are most often used by adults and allow for a
"healthy" and more constructive approach to reality. Altruism is an example of a mature defense
mechanism in which individuals provide a helpful, gratifying service to others as a means of
quelling their own anxieties.
(Choice A) A neurotic defense mechanism, reaction formation is the transformation of an
unwanted thought or feeling into its opposite. An example would be having a bias against a
particular race and then embracing that race as ideal and wonderful.
(Choice B) An immature defense mechanism, denial is the failure to accept a disturbing aspect of
external reality. An example would be a patient diagnosed with a terminal illness who refuses to
accept that he will likely die soon.
(Choice C) Sublimation is a mature defense mechanism that allows for unacceptable impulses to
be channeled into more acceptable activities. An example would be a man with a fiery temper
who channels his anger into athletic pursuits.
(Choice E) Suppression is a mature defense mechanism that involves intentionally postponing
the exploration of anxiety-provoking thoughts by substituting other thoughts. This allows for
the individual to cope with the present reality instead. An example would be a woman who
focuses on her children's needs instead of thinking about her father's diagnosis of cancer.
Educational Objective:
Altruism involves minimizing internal fears by serving others.
The correct answer is: . Altruism

115. A 34-year-old woman who describes herself as a "holistic healer" brings her two
children aged four years and three years to the pediatrician for routine physical
examinations. This is a first visit as they have recently moved from another state. The
medical records for the children indicate that no immunizations have been given. When
the mother is questioned about this, she proudly replies, "My children are wonderfully
healthy on their own and have no need for these artificial vaccines." The principles and
benefits of immunization are discussed at length, as well as the risks inherent in not
being immunized. The physician recommends that the children be given all age-
appropriate vaccinations today. The mother remains convinced that immunizations cause
more harm than good, and she steadfastly refuses to allow her children to be vaccinated.
What is the most appropriate next step?
Select one:
 . Request to speak with her husband
 . Document in the medical chart that the risks and benefits of vaccination have been
explained
 . Proceed with immunizations today
 . Obtain a court order for immunization of children
 . Inform mother that she will be reported to the local health department

In the medical community, the consensus is that all children should be immunized against major
diseases unless a specific contraindication ( eg, allergy to vaccine components) exists. It is
estimated that over 90% of children in the United States have received all major childhood
vaccinations by the time they enter school, and public health personnel continue in their efforts
to increase this number. Fortunately, however, the risk to those who have not been vaccinated
remains minimal. This is due in part to the presence of "herd immunity," whereby the disease
resistance of the majority confers protection upon the disease susceptible minority. Therefore,
the initial step in this case would be to explain the potential risks and benefits of vaccination to
the mother. If she persists in refusing to have her children vaccinated, her wishes should be
respected. Documentation of such a discussion in the medical chart is imperative.
(Choice A) There is no need to obtain a court order in this instance, as there is a very low risk
that the child will develop one of the diseases he is not immunized against.
(Choice B) Immunizing the children without consent from the mother would be appropriate only
if they were in imminent danger.
(Choice C) Threatening to report the mother to the local health authorities is inappropriate.
While health departments are interested in recording the incidence of specific conditions ( eg,
measles), they do not track or enforce immunization against such diseases.
(Choice E) Insisting upon speaking with the father is inappropriate. The mother's wishes should
be respected and documented in the medical chart.
Educational Objective:
If no significant harm is likely to result from withholding therapy, parental wishes regarding the
medical care of a child should be honored and the discussion documented in the chart.
The correct answer is: . Document in the medical chart that the risks and benefits of vaccination
have been explained

116. A 64-year-old man complains of palpitations and progressive shortness of breath


over the past several hours. He says that he also develops a choking sensation every time
he tries to lie down. His medical history is significant for hypertension for the past 20
years and medication non-compliance. He also has a 35-year smoking history. He reports
that his father died of a heart attack at age 70 and his mother suffered from asthma. On
physical examination, his blood pressure is 170/100 mmHg and his heat rate is 130/min
and irregularly irregular. Lung exam reveals bibasilar crackles. There is 2+ pitting edema
of the lower extremities. Bedside echocardiography shows a left ventricular ejection
fraction of 55%. Which of the following is most likely responsible for his symptoms?
Select one:
 . Diastolic dysfunction
 . Small airway bronchoconstriction
 . Increased lung compliance
 . Cardiogenic shock
 . High-output heat failure

This man has heart failure due to diastolic dysfunction, also called heart failure with preserved
left ventricular ejection fraction. Diastolic dysfunction refers to impaired filling of the left or right
ventricle, either because of impaired myocardial relaxation or a stiff, non-compliant ventricle.
Contractility (i.e. ejection fraction) may remain normal but diastolic pressures are elevated,
consequently reducing cardiac output. This leads to typical findings of congestive heart failure.
Systemic hypertension is the classic cause of diastolic dysfunction. Chronically elevated left
ventricular diastolic pressures cause left atrial dilatation, which in turn can lead to atrial
fibrillation, as in this patient. Treatment is with diuretics and blood pressure control.
(Choice A) Cardiogenic shock usually results from compromised left ventricular function due to
an acute myocardial infarction or chronic heart failure. Affected patients show signs of heart
failure but would not have the left ventricular function to produce a blood pressure of 170/100.
An ejection fraction of 55% would be very unusual for a patient with cardiogenic shock.
(Choice C) High-output heart failure refers to supranormal ventricular function that still cannot
meet the body's metabolic demands. Causes include anemia, hyperthyroidism, beriberi, Paget's
disease, and arteriovenous fistulas. The patient above has a normal ejection fraction.
(Choice D) Asthma is due to small airway bronchoconstriction and will not cause the signs and
symptoms of heart failure described here.
(Choice E) Lung compliance increases in chronic obstructive lung disease. COPD would not
cause this patient's pulmonary edema or signs of right heart failure. Lung compliance is
decreased in pulmonary edema.
Educational objective:
Diastolic dysfunction refers to impaired ventricular filling due to poor myocardial relaxation or
diminished ventricular compliance. It is usually due to hypertension and is treated with diuretics
and antihypertensives. In severe cases, diastolic dysfunction can cause decompensated heart
failure.
The correct answer is: . Diastolic dysfunction

117. A 72-year-old man with coronary artery disease and hypertension is hospitalized
after suffering a myocardial infarction 5 days ago. He suddenly complains of severe chest
pain. His blood pressure is 90/60 mmHg and heart rate is 65/min. Auscultation reveals
no murmurs or rubs. An ECG reveals sinus rhythm with an acute ST-segment elevation in
the anteroseptal area. Urgent bedside echocardiography showed anteroseptal, lateral,
and apical akinesis, mild left ventricular systolic dysfunction, and severe pericardial
effusion. Within 20 minutes he is unconscious with undetectable pulses and blood
pressure. What is the most likely cause of the patient’s sudden decompensation?
Select one:
 Pericarditis
 Free wall rupture
 Left ventricular thrombus
 Mitral regurgitation
 Ventricular septal rupture

Myocardial rupture is a sudden postinfarction complication that typically occurs 5–10 days after
an MI (peak at 7 days). During this time the integrity of the cardiac wall is compromised by
macrophage and mononuclear infiltration, fibrovascular response, and other inflammatory
mediators, as they replace necrotic tissue with scar tissue. Old age, first MI, and a history of
hypertension are risk factors. The clinical manifestations, as seen here, are a sudden loss of heart
rate, blood pressure, and consciousness, while the ECG continues to show a sinus rhythm.
Measures to prevent cardiac rupture include the administration of β-blockers, angiotensin-
converting enzyme (ACE) inhibitors, and the avoidance of steroidal and nonsteroidal anti-
inflammatory agents such as ibuprofen and indomethacin.
Answer B is incorrect. An LV thrombus can occur as a post-MI complication. Although an
embolus could result in stroke and subsequent mental status change, this would not cause a
sudden loss of pulses and blood pressure.
Answer C is incorrect. Papillary muscle rupture will lead to sudden pulmonary edema, shortness
of breath, effusions and crackles, and a new mitral regurgitation murmur on chest examination.
Answer D is incorrect. Pericarditis would likely result in a pericardial rub on physical examination
and it usually causes ECG changes. The acute nature of this decompensation does not suggest
pericarditis.
Answer E is incorrect. Although septal rupture could result in acute decompensation, a new
harsh murmur could likely be picked up on physical examination. The electromechanical
dissociation is more consistent with free wall rupture.
The correct answer is: Free wall rupture

118. A 25-year-old woman comes to the office for the evaluation of pale patches of
skin around her mouth. She noticed these lesions a few months ago, but they have
become more prominent now. There is no itching, burning, or numbness over the
patches. Her vital signs are stable. On examination, you notice pale white patches
symmetrically distributed around her mouth. The borders of these macules are well-
circumscribed and hyperpigmented. Similar lesions are also found over the areola of her
breasts. She denies any history of trauma or infection. Which of the following best
explains the pathology of her condition?
Select one:
 . Inherited absence of melanocytes
 . Infection with mycobacterium leprae
 . Superficial fungal infection
 . Destruction of melanocytes
 . Post inflammatory

Vitiligo is a specific form of leukoderma which usually peaks in persons aged 20-30 years. The
depigmentation has a predilection for acral areas and around body orifices (as in this patient),
and appears as pale whitish macules with hyperpigmented borders. The etiopathology is
thought to be an autoimmune process which causes destruction of the melanocytes. The
condition is slowly progressive. Few experience spontaneous remission.
(Choice A) Post-inflammatory hypopigmentation is usually associated with a history of trauma or
inflammation. There is a reduction in pigmentation rather than complete absence, as in this
patient.
(Choice C) Inherited absence of melanocytes is also known as piebaldism. It is usually noticed at
birth, and is confined to the head and trunk.
(Choice D) Infection with Mycobacterium /eprae causes leprosy. Areas of hypopigmentation with
anesthesia are characteristic.
(Choice E) Superficial fungal infection can result in pale macules due to a lessening of
pigmentation, rather than total absence, as in vitiligo. These macules can be velvety pink or
whitish. Sometimes, there are hyperpigmented patches.
Educational Objective:
Vitiligo is characterized by macular depigmentation that involves the acral and peri-orificial
areas. It is caused by the autoimmune destruction of melanocytes.
The correct answer is: . Destruction of melanocytes

119. A 40-year-old man comes to the physician because of increasing urinary


frequency and urgency. He has had these symptoms in the past, but they are more
troublesome now. He has also had urinary hesitancy and interruption of flow. His
temperature is 37°C (98.6°F), blood pressure is 130/75mm Hg, pulse is 76/min, and
respirations are 15/min. Physical examination shows no abnormalities except increased
tone of the anal sphincter and mild periprostatic tenderness. Urinalysis and urine culture
shows no abnormalities and expressed prostatic secretions show a leukocyte count of
four WBCs/HPF (normal is less than 10 WBCs/HPF). Serum prostate-specific antigen is 2
ng/ml (normal value is less than 4ng/ml). Which of the following is the most likely
diagnosis?
Select one:
 . Prostatic cancer
 . Non-inflammatory chronic prostatitis
 . Acute bacterial prostatitis
 . Chronic bacterial prostatitis
 . Inflammatory chronic prostatitis

Patients with non-inflammatory chronic prostatitis are afebrile and have irritative voiding
symptoms. Physical examination is unremarkable and urinalysis is normal. Expressed prostatic
secretions show a normal number of leukocytes and culture of these secretions is negative for
bacteria. There is usually no history of past UTI but voiding abnormalities may be present in the
past.
{Choice A} Patients with chronic bacterial prostatitis are afebrile and have irritative voiding
symptoms like urinary frequency, urgency and suprapubic or perineal discomfort. Urinalysis is
normal. Expressed prostatic secretions show leukocyte count greater than 1 0 WBCs/HPF and
culture of these secretions grows the colonies of causative pathogen. Rectal examination may
show normal or indurated prostate.
{Choice B) Patients with inflammatory chronic prostatitis are afebrile and have irritative voiding
symptoms like urinary frequency, urgency and suprapubic or perineal discomfort. Urinalysis is
normal. Expressed prostatic secretions show a leukocyte count greater than 1 0 WBCs/HPF and
culture of these secretions is negative.
{Choice D) Prostatic cancer is highly unlikely with normal PSA levels and with normal digital
rectal examination in a young patient.
{Choice E) Patients with acute bacterial prostatitis are febrile with suprapubic or perineal pain.
Irritative voiding symptoms like urinary frequency or urgency may be present. Exquisitely tender
prostate is noted on rectal examination and urinalysis shows bacteriuria and pyuria.
Educational Objective:
Patients with non-inflammatory chronic prostatitis are afebrile and have irritative voiding
symptoms. Expressed prostatic secretions show a normal number of leukocytes and culture of
these secretions is negative for bacteria.
The correct answer is: . Non-inflammatory chronic prostatitis

120. A 54-year-old female with a long history of hypertension and a recent


hospitalization for atrial fibrillation with rapid ventricular response now returns to the
hospital complaining of skin changes. Her medications include warfarin,
hydrochlorothiazide and metoprolol. On physical examination, her temperature is 36.7°C
(98°F), blood pressure is 130/80 mm Hg, pulse is 80/min and irregular, and respirations
are 16/min. You observe the skin changes pictured below. Her exam is otherwise
unremarkable. Which of the following is the primary cause of her condition?

Select one:
 . Antithrombin III deficiency
 . Vitamin K deficiency
 . Excessive platelet aggregation
 . Protein C deficiency
 . Factor VII deficiency

Based on the history given, it is likely that this is a patient who recently started warfarin therapy
for new atrial fibrillation. Warfarin is an anticoagulant that works by inhibiting production of the
vitamin K-dependent clotting factors II, VII, IX, and X. Administered to patients with atrial
fibrillation, warfarin helps to protect against embolic events. However, it also inhibits the blood's
natural anticoagulants, proteins C and S. While clotting factors II, IX, and X have half-lives of
around 60 hours, protein C's half-life is only 9 hours. Therefore, warfarin can lead to protein C
deficiency in the first days of treatment, resulting in a hypercoagulable state and placing the
patient at risk for thrombus formation and skin necrosis. This is particularly common in patients
with underlying congenital protein C deficiency.
(Choice A} Antithrombin Ill is a vitamin K-independent inhibitor of the clotting cascade.
Antithrombin Ill deficiency predisposes to thrombus formation.
(Choice B) Factor VII deficiency causes a bleeding diathesis characterized by bruising and
hemorrhage. In contrast, this patient has skin necrosis that is the result of a pro-thrombotic
state.
(Choice C) Excessive platelet aggregation is known to occur in a small percentage of patients
started on heparin therapy. This paradoxical pro-thrombotic state, known as HITT (heparin-
induced thrombocytopenia and thrombosis}, is autoimmune mediated and can lead to skin
necrosis as well as hemorrhage. Warfarin therapy, however, does not lead to excessive platelet
aggregation.
(Choice D) Vitamin K deficiency occurs most commonly in newborns and patients taking certain
antibiotics. Deficiency leads to a bleeding diathesis characterized by bruising and hemorrhage.
Warfarin is a vitamin K antagonist
Educational objective:
Warfarin is commonly used for anticoagulation because it can be administered orally. However,
during the first few days that it is given, it can have paradoxically pro-coagulant effects in
susceptible patients by inhibiting protein C production. Affected patients may present with skin
necrosis secondary to thrombus formation.
The correct answer is: . Protein C deficiency

121. A 50-year-old man wants to talk to you about something, "absolutely


confidential". After you assure him, he admits, "He is unable to get an erection and just
can't have sex." He wants to figure it out quickly because "he simply can't live like this."
He has never been diagnosed with diabetes and denies other complaints. He has a 2
pack/day history of smoking for 30 years. On examination, his BP: 158/90mm of Hg;
Temperature: 37.1°C (98.8°F); RR 14/min; PR 82/min. There is upper body obesity,
rounded face, increased fat around the neck, and thinning of arms and legs. You find his
skin to be bruised, fragile and thin. Laboratory reveals the following results. Serum:
Glucose 186 mg/dl, Sodium 142 mEq/L, Potassium 2.5 mEq/L, Bicarbonate 38 mEq/L.
Chest X ray shows a large mass in left bronchus. What is the most likely cause of
patient's condition?
Select one:
 . Pituitary adenoma
 . Ectopic ACTH syndrome
 . Exogenous steroid intake
 . Adrenal tumors
 . Familial cushing's syndrome

The patient has all the features of Cushing's syndrome: including upper body obesity, rounded
face, increased fat around the neck, and thinning arms and legs. Other symptoms appear in the
skin, which becomes fragile and thin. It bruises easily and heals poorly. Purplish pink stretch
marks may appear on the abdomen, thighs, buttocks, arms and breasts. The bones are
weakened, and routine activities, i.e. bending, lifting, or rising from a chair may lead to
backaches, rib and spinal column fractures. Most people have severe fatigue, muscle weakness,
high blood pressure, and high blood sugar. Irritability, anxiety, and depression are also common.
Sometimes sexual dysfunction can be the presenting complaint The patients also have
hypokalemic alkalosis.
Cushing's syndrome may be caused by a number of causes: including Adrenal neoplasia,
Pituitary adenoma (Cushing's disease), Exogenous steroid intake, or some tumors that produces
ACTH like Lung cancers. This condition is known as Ectopic ACTH syndrome. With long history of
smoking and radiographic findings this should be high on the differential diagnosis in this case.
Educational Objective:
Ectopic ACTH production by lung cancer is an important cause of cushing's syndrome.
The correct answer is: . Ectopic ACTH syndrome
122. A 60-year-old man comes to the physician because of worsening fatigue and
nausea. He had a carotid angiogram for the evaluation of symptomatic carotid artery
stenosis 15 days ago, and was discharged home three days after the procedure. His
medical problems are hypercholesterolemia, coronary artery disease, intermittent
claudication, hypertension for 18 years, and diabetes mellitus for 15 years. Physical
examination shows painless, reddish-blue mottling of the skin of the extremities.
Laboratory studies show: Hb 10.5 g/dl, WBC 10,000/cmm with 12% eosinophils, BUN 46
mg/dl, Serum creatinine 3.0 mg/dl, Serum C3 level Decreased. Urinalysis:pH
Normal,Esterase Negative, Nitrite Negative, Protein 1+, WBC Many eosinophils, RBC 1-
2/HPF. Which of the following is the most likely cause of this patient's findings?
Select one:
 . Contrast nephropathy
 . Acute allergic interstitial nephropathy
 . Diabetic nephropathy
 . Cholesterol embolism
 . Post streptococcal glomerulonephritis

Atheroembolic disease (AED), also known as cholesterol embolization, occurs due to showering
of cholesterol crystals from the aorta or other major arteries. It usually manifests following
surgical or interventional manipulation of the arterial tree, or during/following treatment with
anticoagulants or and hypocomplementemia. The definitive diagnosis is made by tissue biopsy,
which shows cholesterol crystals. Treatment is conservative. Anticoagulation should be stopped
since it may prevent healing of the ruptured plaques. Steroids have been used with little success.
(Choice A) Contrast-induced nephropathy (contrast is used for the angiogram) is usually seen
within 24-72 hours of the procedure. It is very unlikely to occur five days after the procedure.
Patients usually improve over a period of time, and do not present with skin findings. The
recommendations for the prevention of contrast nephropathy are: (1) periprocedural hydration,
(2) use of a low-osmolality contrast, and (3) limiting the amount of contrast agent. Patients with
borderline renal failure will also benefit from prophylactic administration of N-acetylcysteine
(Mucomyst) and fenoldopam.
(Choice C) Diabetic nephropathy is characterized by heavy proteinuria, and is not associated
with eosinophiluria and eosinophilia {which were seen in this patient). Furthermore, OM
nephropathy develops over a period of years, not months. Hence, if a diabetic patient presents
with a rapid decline (over months) in renal function, look for other causes of renal failure.
(Choice D) Post-streptococcal glomerulonephritis is characterized by a nephritic picture in a
patient with a history of streptococcal throat or skin infections. Complement levels are low. Skin
manifestations such as livedo reticularis or eosinophilia are not seen.
(Choice E) Acute allergic interstitial nephropathy is a drug-induced hypersensitivity reaction
characterized by rash, renal failure, eosinophilia, and eosinophiluria (Hansel stain). The common
medications that you should remember for the exam are:
1 . Antibiotics (most common is methicillin group)
2. NSAIDs (often cause heavy proteinuria)
3. Thiazides
4. Phenytoin
5. Allopurinol
Educational Objective:
Cholesterol embolization usually follows surgical or interventional manipulation of the arterial
tree. Renal failure, livedo reticularis, systemic eosinophilia, and low complement levels should
make you think of cholesterol embolism.
The correct answer is: . Cholesterol embolism

123. A 72-year-old woman had a pacemaker inserted 4 years ago for symptomatic
bradycardia because of AV nodal disease. She is clinically feeling well and her ECG shows
normal sinus rhythm at a rate of 68/min but no pacemaker spikes. Her pacemaker only
functions when the ventricular rate falls below a pre-set interval. Which of the following
best describes her pacemaker function?
Select one:
 ventricular inhibited
 atrial sequential
 asynchronous
 atrial synchronous
 ventricular synchronous

The ventricular inhibited (VVI) pacemaker functions when the heart rate falls below a preset
interval. If a QRS is detected, the pacemaker is inhibited. If a QRS is not sensed, the pacing
stimulus is not inhibited and the ventricle is stimulated. (Fuster, p. 909)
The correct answer is: ventricular inhibited

124. A 28-year-old man with a history of intravenous drug abuse presents to the
emergency department with a 2-day history of fever, chills, and shortness of breath. On
physical examination the patient has a new heart murmur, small retinal hemorrhages,
and subungual petechiae. Which of the following is the most likely causative organism?
Select one:
 Streptococcus viridans
 Staphylococcus epidermidis
 Staphylococcus aureus
 Mycobacterium tuberculosis
 Group A Streptococcus

In patients with infective endocarditis and a history of intravenous drug abuse, Staphylococcus
aureus is the causative agent in the vast majority of cases and is more likely to cause acute
rather than subacute endocarditis. If the patient has a prosthetic valve, then coagulase-negative
staphylococcus is the predominant organism. Bacterial endocarditis is an infectious process of
the endothelial surface of the heart. Symptoms includefever, fatigue, malaise, vascular
phenomena such as Janeway lesions, and immunologic phenomena such as Osler nodes.
Diagnosis is usually based on the Duke criteria, and laboratory studies include blood culture and
echocardiography. Treatment is through intravenous antibiotics and surgery for valve repair or
replacement, if necessary.
Answer A is incorrect. Infection with Group A Streptococcus, if untreated, can lead to rheumatic
fever, damaging heart valves and muscle. Rheumatic fever can predispose the patient to
subsequent development of infective endocarditis, but it does not cause the disease initially.
Answer B is incorrect. Tuberculosis can cause cardiac disease, but in this patient, especially with
the history of intravenous drug abuse, Staphylococcus aureus is the most likely organism.
Answer D is incorrect. If the patient has a prosthetic valve, coagulase-negative staphylococcus is
the predominant organism. Within 2 months of surgery, the most common organisms are
nosocomial: coagulase-negative staphylococcus, Staphylococcus aureus, facultativegram-
negative bacilli, diphtheroids, and fungi. Twelve months after surgery, the organisms are similar
to those that cause community-acquired endocarditis.
Answer E is incorrect. Streptococcus viridans is the most common pathogen for left-sided
subacute bacterial endocarditis.
The correct answer is: Staphylococcus aureus

125. A 35-year-old Caucasian male presents to the emergency department with two
episodes of bloody vomiting which occurred one-half hour ago. He has a history of
migraines. For the past two days, he has been having severe headaches and has taken 20
tablets of aspirin without relief. He then resorted to heavy drinking and forgot about the
pain. He drinks alcohol "occasionally" and has been smoking 1 pack of cigarettes daily
for the past 18 years. Which of the following is the most likely explanation for this
patient's hematemesis?
Select one:
 . Fulminant hepatic failure
 . Mallory Weiss syndrome
 . Acute platelet dysfunction
 . Acute erosive gastritis
 . Esophageal variceal bleeding

Acute erosive gastritis is characterized by the development of severe hemorrhagic erosive


lesions after the exposure of gastric mucosa to various injurious agents or after a substantial
reduction in blood flow. Aspirin decreases the protective prostaglandin production. In addition,
aspirin and alcohol cause direct mucosal injury (as in this case). Acute mucosal injury decreases
the normal protective barriers (decreased levels of secreted mucins and bicarbonate, and
decreased integrity of the epithelium), thus permitting acid and other luminal substances (e.g.,
proteases and bile acids) to penetrate into the lamina propria, thereby causing additional injury
to the vasculature and subsequent hemorrhage. As in this case, patients may manifest with
hematemesis and abdominal pain.
(Choice A) Esophageal variceal bleeding is the other common cause of upper Gl bleeding, but it
is seen in patients with liver disease. This patient is a young, occasional drinker and has no other
findings suggestive of liver disease.
(Choice C) Mallory-Weiss tears occur in the distal esophagus at the gastro-esophageal junction
after repeat bouts of retching and vomiting. No such history is given in this case.
(Choice D) This patient has no other signs suggestive of fulminant hepatic failure, which would
be characterized by hepatic encephalopathy.
(Choice E) Platelet dysfunction might be an aggravating factor in this patient's bleeding since he
took large amounts of aspirin; however, platelet dysfunction by itself cannot cause erosive
gastritis.
Educational Objective:
Massive doses of aspirin and NSAIDs can cause acute erosive gastritis and upper Gl bleeding.
Alcohol can aggravate their effect.
The correct answer is: . Acute erosive gastritis

126. A 36-year-old white male is brought to the emergency department because of


dyspnea, tachypnea, crampy pain and paresthesias in his extremities. He gives an unclear
history about how he "rapidly ascended to a height of 10,000 feet” His pulse is 70/min,
blood pressure is 120/80 mmHg, temperature is 36.7°C (98.5°F) and respirations are
24/min. The significant physical finding on examination is carpopedal spasm. At this
point, the suspected diagnosis is acute respiratory alkalosis secondary to
hyperventilation. Which of the following is true regarding this patient's serum calcium
level?
Select one:
 . Increase in calcium bound to inorganic anions
 . Fall in total plasma calcium
 . Increase in calcium bound to albumin
 . Fall in calcium bound to albumin
 . Fall in calcium bound to inorganic anions

Plasma calcium exists in three forms: ionized calcium (45%), albumin-bound calcium (40%), and
calcium bound to inorganic and organic anions ( 15%). Homeostasis of these forms is
significantly influenced by the extracellular pH level. An increased extracellular pH level causes
an increase in the affinity of serum albumin to calcium, thereby increasing the levels of albumin-
bound calcium, and consequently decreasing the levels of ionized calcium. Ionized calcium is the
only physiologically active form, which means that decreased levels of this form can result in
clinical manifestations of hypocalcemia (e.g., crampy pain, paresthesias and carpopedal spasm).
This vignette illustrates such a scenario, wherein the patient is experiencing signs and symptoms
of hypocalcemia secondary to respiratory alkalosis (increased pH level).
(Choices A and F) Due to the abovementioned shift in the forms of calcium, clinical
hypocalcemia can thus occur even if the patient has a normal total plasma calcium level.
(Choices D and E) The level of calcium bound to organic and inorganic anions remains unaltered
in patients with acute respiratory alkalosis.
Educational Objective:
Increased extracellular pH levels (e.g., respiratory alkalosis) can cause an increase in the affinity
of serum albumin to calcium, thereby increasing the levels of albumin-bound calcium, and
consequently decreasing the levels of ionized calcium. Ionized calcium is the only physiologically
active form, which means that decreased levels of this form can result in clinical manifestations
of hypocalcemia.
The correct answer is: . Increase in calcium bound to albumin

127. A 67 -year-old male presents to the emergency department with severe dizziness
and the inability to walk. He complains of repetitive vomiting and occipital headache.
The symptoms started two hours ago when he was playing golf in sunny weather with
his friends. His past medical history is significant for hypertension and diabetes. He
underwent coronary stenting two years ago for recurrent chest pain. His current
medications are aspirin, glipizide, enalapril and metoprolol. His blood pressure is
210/110 mmHg, heart rate is 78/min, temperature is 37.8°C (100°F), and respirations are
18/min. Muscle strength is preserved in all four extremities, and there are no sensory
abnormalities. Which of the following is the most likely cause of this patient's symptoms?
Select one:
 . Heat stroke
 . Posterior cerebral artery occlusion
 . Vestibular neuronitis
 . Cerebellar haemorrhage
 . Meniere disease

The clinical features described in the vignette are consistent with cerebellar hemorrhage. This
condition generally evolves over a few hours. Patients typically have an acute onset occipital
headache, repeated vomiting and gait ataxia. A small hemorrhage may not manifest with any
other neurological deficit. A larger hemorrhage may manifest with 6th nerve paralysis, conjugate
deviation, and blepharospasm. Patients with large hemorrhages often become stuporous in a
few hours. Immediate evacuation of the hematoma is required and life-saving for these patients.
Neurological recovery in patients who survive is usually near complete.
(Choice A} A heat stroke occurs when there is a failure of body thermoregulation upon exposure
to high environmental temperatures. It is characterized by a very high core body temperature (>
105°F}, dehydration, confusion, coma, and dry, flushed skin. The term "heat exhaustion" is used
for a less severe form of heat illness.
(Choice B) "Vestibular neuronitis" is a term often used for describing the acute onset of vertigo
and nystagmus without any other neurological deficit. Occipital headache and hypertension are
not the manifestations of vestibular neuronitis.
(Choice C) The posterior cerebral artery supplies the brain stem, cerebellum and occipital lobes.
Due to this widespread blood supply, posterior cerebral artery occlusion rarely causes vertigo
without cranial nerve, sensory, motor or cerebellar involvement.
(Choice E) Meniere disease is a labyrinthine dysfunction characterized by an increased pressure
of the endolymph, thereby resulting in recurrent vertigo, tinnitus and hearing problems.
Vomiting is not a prominent manifestation, and occipital headache is not a cardinal
manifestation.
Educational Objective:
The sudden onset of vertigo, vomiting and occipital headache in a hypertensive patient is
strongly suggestive of cerebellar hemorrhage. Other manifestations are 6th nerve paralysis,
conjugate deviation, blepharospasm and coma.
The correct answer is: . Cerebellar haemorrhage

128. A 46-year-old woman is hospitalized for agitation, restlessness and poor sleep.
She has been complaining of headaches recently and has gained 14 pounds over the
past three months. She denies any illicit drug use. Her blood pressure is 160/110 mmHg
and her heart rate is 90/min. Her laboratory findings are shown below: Sodium 142
mEq/L, Potassium 3.2 mEq/L, Chloride 98 mEq/L, Bicarbonate 26 mEq/L, BUN 12 mg/dl,
Creatinine 0.9 mg/dl, Glucose 205 mg/dl, Calcium 94 mg/dl. Which of the following is the
most likely cause of this patient's hypertension?
Select one:
 . Adrenal medullary disease
 . Renal parenchymal disease
 . Parathyroid gland disease
 . Hypothyroidism
 . Adrenal cortical disease

This woman most likely has secondary hypertension due to Cushing's syndrome. High levels of
cortisol enhance vasoconstriction, insulin resistance, and mineralocorticoid activity. This results
in blood pressure elevation, hyperglycemia, and hypokalemia. Proximal muscle weakness, central
adiposity, thinning of the skin, weight gain, and psychiatric problems (e.g. sleep disturbances,
depression, and psychosis) are also common in Cushing's syndrome. Cushing's syndrome may
result from adrenal cortical hyperplasia, ACTHproducing pituitary adenoma (Cushing's disease),
ectopic ACTH production, or from administration of exogenous steroids.
(Choice A) Renal parenchymal disease causes hypertension. However, parenchymal disease
severe enough to cause hypertension should also cause a decrease in glomerular filtration rate.
This patient's creatinine concentration is normal.
(Choice B) Renal artery stenosis is an important cause of secondary hypertension. Usually due to
atherosclerosis, renal artery stenosis generally presents with few systemic complaints and may
be entirely asymptomatic. It would not explain this woman's weight gain, hypokalemia, or
hyperglycemia. A bruit may often be heard upon auscultation of the affected renal artery.
(Choice C) Aortic dissections present with tearing chest pain that radiates to the back. They most
commonly occur in patients with hypertension but are not, themselves, causes of hypertension.
(Choice D) Hypothyroidism causes weight gain, fatigue, bradycardia, depression, and skin and
hair changes. It does not typically cause hyperglycemia, hypokalemia, or severe hypertension.
(Choice E) Hyper- and hypoparathyroidism typically present with asymptomatic hyper- or
hypocalcemia, respectively. Hyperparathyroidism may cause elevated blood pressure but this
diagnosis is unlikely in a patient with a normal calcium concentration.
(Choice G) Increased activity within the adrenal medulla would cause elevated catecholamine
production (pheochromocytoma). Symptoms include weight loss, tachycardia, hypertension,
diaphoresis, and anxiety.
Educational objective:
Cushing's syndrome is an important cause of secondary hypertension. In addition to blood
pressure elevation, high systemic cortisol concentrations cause hyperglycemia, hypokalemia,
proximal muscle weakness, central adiposity, thinning of the skin, weight gain, and psychiatric
problems (e.g. sleep disturbances, depression, and psychosis).
The correct answer is: . Adrenal cortical disease

129. A 60-year-old man comes to the office with worsening shortness of breath over
the last 5 months. He has had two episodes of bronchitis over the last 12 months. He
now has a mild nonproductive cough. He denies any fever, chills, hemoptysis, chest pain,
or difficulty breathing while lying on his back. His medications include an ipratropium
inhaler, aspirin, and amlodipine. He was hospitalized twice in the last 3 years due to an
exacerbation of his chronic obstructive pulmonary disease. He has been smoking 1 pack
of cigarettes per day for the last 40 years. His temperature is 99°F (37.2°C), blood
pressure is 130/86 mm Hg, pulse is 98/min, and respirations are 18/min. Examination
shows a thin man in mild respiratory distress with increased anteroposterior chest
diameter, diffuse expiratory wheeze and loud S2. Chest X-ray shows hyperinflation of
bilateral lung fields with diaphragm flattening and small heart size. Which of the
following is the most effective measure to decrease mortality in this patient?
Select one:
 . Adding a long-acting beta agonist
 . Adding systemic corticosteroids
 . Adding a short acting beta-agonist inhaler
 . Adding an inhaled corticosteroid
 . Smoking cessation

130. A 25-year-old woman is admitted with fever and hypotension. She has a 3-day
history of feeling feverish. She has no history of chronic disease, but she uses tampons
for heavy menses. She is acutely ill and, on physical examination, found to have a diffuse
erythematous rash extending to palms and soles. She is confused. Initial blood tests are
as follows: White blood cell count: 22,000/μL, Na+: 125 mEq/L, K+: 3.0 mEq/L, Ca++: 8.0
mEq/mL, Activated partial thromboplastin time (PTT): 65 (normal 21 to 36), Prothrombin
time (PT): 12s (normal < 15s), Aspartate aminotransferase: 240 U/L (normal < 40),
Creatinine: 3.0 mg/dL, Antinuclear antibodies: negative, Anti-DNA antibodies: negative,
Serologic tests for RMSF, leptospirosis, measles: negative. Which of the following best
describes the pathophysiology of the disease process?
Select one:
 . Acute bacteremia
 . Toxin-mediated inflammatory response syndrome
 . Exacerbation of connective tissue disease
 . Tick-borne rickettsial disease
 . Allergic reaction

(Fauci, pp 877, 872-881.) The disease process described is most consistent with toxic shock
syndrome, an inflammatory response syndrome characterized by hypotension, fever, and
multiorgan involvement. It can occur in healthy women who use tampons. TSST-1 is a toxin
produced by S aureus that is responsible for activating superantigens such as tumor necrosis
factor and interleukin-1. Symptoms include confusion, as has occurred in this patient, in addition
to diarrhea, myalgias, nausea and vomiting, and syncope. In addition to fever and hypotension
there is a diffuse rash initially appearing on the trunk but spreading to palms of the hands and
soles of the feet. Desquamation occurs a week after initial appearance of the rash. There are
many potential laboratory abnormalities as manifestations of multiorgan involvement. These
include azotemia, coagulopathy with abnormal aPTT, and electrolyte abnormalities, including
hyponatremia, hypocalcemia, and hypokalemia. Liver function tests show hyperbilirubinemia and
elevated alanine aminotransferase.
The disease is not a bacteremia, although it is precipitated by localized staphylococcal or
sometimes streptococcal infection. Toxic shock syndrome sometimes mimics diseases that cause
multiorgan involvement, such as systemic lupus or Rocky Mountain spotted fever. Serological
studies for these diseases were negative in this patient. An allergic reaction would cause urticaria
and would not account for the fever and the electrolyte abnormalities.
The correct answer is: . Toxin-mediated inflammatory response syndrome

131. A 48-year-old Caucasian male presents to your office complaining of progressive


exertional dyspnea. It has become especially bothersome over the past two months.
Presently, he becomes short of breath after climbing one flight of stairs. He denies any
significant problems in the past. He is not taking any medications and he denies smoking
or drinking alcohol. His temperature is 37.2°C (98.9°F), pulse is 78/min, blood pressure is
130/75 mmHg and respirations are 14/min. Chest examination reveals a harsh systolic
murmur that is best heard at the right second intercostal space with radiation along the
carotid arteries. An S4 is heard at the apex. Based on these findings, what is the most
likely cause of this patient's symptoms?
Select one:
 . Senile calcific aortic stenosis
 . Myxomatous valve degeneration
 . Rheumatic heart disease
 . Hypertrophic cardiomyopathy
 . Bicuspid aortic valve

This patient appears to be suffering from congestive heart failure as a consequence of aortic
stenosis. A harsh systolic murmur at the right upper sternal border with radiation to the carotid
arteries is a classic description of the murmur caused by aortic stenosis. The S4 occurs as the
result of left atrial kick against a stiff left ventricle. In aortic stenosis, the high resistance
generated by the stenosed aortic valve causes concentric hypertrophy and stiffening of the left
ventricle, resulting in the S4. The three most common causes or aortic stenosis in the general
population are senile calcific aortic stenosis, bicuspid aortic valve, and rheumatic heart disease.
The most important hint to the underlying etiology of this patient's aortic stenosis is his
relatively young age. A bicuspid aortic valve is the cause of aortic stenosis in the majority of
patients under 70 years old, so this is the most likely cause of this patient's aortic stenosis.
(Choice A} The murmur of hypertrophic cardiomyopathy can be easily confused with that of
aortic stenosis. Both entities can cause a systolic crescendo-decrescendo murmur and an S4.
However, the murmur of hypertrophic cardiomyopathy is usually best appreciated in the lower
left sternal border and it does not typically radiate to the carotids.
(Choice B) Myxomatous valve degeneration is the typical pathologic entity that causes mitral
valve prolapse.
(Choice C) As mentioned above, rheumatic heart disease can be a cause of aortic stenosis.
However, it is a much less common cause than either senile calcific aortic stenosis or a bicuspid
aortic valve.
(Choice E) Senile calcific aortic stenosis is the most common cause of aortic stenosis in patients
who are older than 70 years old. As people age, the aortic valve can accumulate calcium, causing
stenosis.
Educational objective:
The three most common causes of aortic stenosis in the general population are senile calcific
aortic stenosis, bicuspid aortic valve, and rheumatic heart disease. A bicuspid aortic valve is the
cause of aortic stenosis in the majority of patients under 70 years old.
The correct answer is: . Bicuspid aortic valve

132. A 65-year-old man comes to the office with a six-month history of a non-healing
ulcer on his right forearm. Physical examination demonstrates a scaling plaque with
central ulceration and 1.5 cm diameter. The biopsy shows polygonal cells with atypical
nuclei at all levels of the epidermis with zones of keratinization. What is the single most
important risk factor for this condition?
Select one:
 . Arsenic
 . Aromatic hydrocarbons
 . Chronic osteomyelitis
 . Sunlight
 . Chronic scars

The clinical scenario described is typical for non-melanoma skin cancer. This patient's histologic
features are suggestive of squamous cell carcinoma (SCC), which is the second most common
form of nonmelanoma skin cancer (after basal cell carcinoma). SCC is more aggressive than
basal cell carcinoma because it metastasizes frequently. Exposure to sunlight is the single most
important factor in the development of SCC. The ultraviolet spectrum of sunlight causes DNA
damage and is mutagenic. People with only a little amount of pigment in their skin (fair
complexion, blue eyes, tendency to sunburn easily) are particularly prone to sec.
(Choices Band C) Arsenic and aromatic hydrocarbons are other well-known factors associated
with SCC, but these have much less overall importance than sunlight.
(Choices D and E) Intrinsic factors predisposing to SCC include chronic osteomyelitis and
chronic scars, but these are uncommon causes of this condition.
Educational Objective:
Squamous cell carcinoma is the second most common form of non-melanoma skin cancer. The
single most important risk factor for the development of squamous cell carcinoma is exposure to
sunlight.
The correct answer is: . Sunlight

133. A 65-year-old man complains of lower leg swelling, fatigue and poor appetite.
His past medical history is significant for recurrent chest infections, wheezing, cough,
recent pyelonephritis, and arterial hypertension. He smokes 1 pack of cigarettes a day
but denies alcohol or intravenous drug use. His physical examination reveals a barrel-
shaped chest with bilateral scattered wheezes. His abdomen is distended and his liver
edge is palpated 4 cm below the right costal margin. You note pitting edema and
dilated, tortuous, superficial veins over the bilateral lower extremities. Manual pressure
on the abdomen causes persistent distention of the jugular veins. The patient's serum
sodium level is 135 mEq/L and his creatinine concentration is 1.2 mg/dl. An abnormality
of which of the following is most likely to explain his edema?
Select one:
 . Pulmonary artery systolic pressure
 . Pulmonary capillary wedge pressure
 . Urinary protein excretion
 . Serum albumin level
 . Portal venous resistance

This patient has recurrent pulmonary infections, chronic cough, a significant smoking history,
and an increased anterior-posterior chest diameter and wheezing on physical examination, all
consistent with chronic obstructive lung disease. The associated chronic hypoxemia causes
constriction of the pulmonary arterial system and, with time, leads to pulmonary hypertension,
right ventricular hypertrophy, and right ventricular failure. Right heart failure has in turn caused
this patient's elevated jugular venous pressure, congestive hepatosplenomegaly, hepatojugular
reflux, and lower extremity edema.
(Choice A} A low serum albumin level can cause anasarca as fluid moves from the vascular
system into the interstitium to equalize osmotic pressures. The leading causes of
hypoalbuminemia are nephrotic syndrome and poor albumin synthesis due to cirrhosis or
malnutrition.
(Choice C) The pulmonary capillary wedge pressure reflects the left atrial pressure. This pressure
is elevated in cases of left ventricular failure, but is not elevated in right ventricular failure with
pulmonary hypertension because the pathologic lesion is before the pulmonary capillary bed.
Crackles on lung exam suggest volume overload in the pulmonary capillaries and pulmonary
edema.
(Choice D) Portal hypertension is the hallmark of cirrhosis. This patient does not have significant
risk factors for cirrhosis (e.g. alcoholism, hepatitis C) and does not have other stigmata of
chronic liver disease like spider angiomata, palmar erythema, or caput medusa.
(Choice E) Nephrotic range proteinuria (greater than 3g protein/day} is an important cause of
systemic edema. However, this patient has more risk factors for secondary pulmonary
hypertension than nephrotic syndrome.
Educational objective:
Right ventricular failure will cause elevated jugular venous pressure, hepatosplenomegaly,
ascites, and peripheral edema, but is not a common cause of pulmonary edema.
The correct answer is: . Pulmonary artery systolic pressure

You might also like